CORNELL
UNIVERSITY
LIBRARY
BOUGHT WITH THE INCOME
OF THE SAGE ENDOWMENT
FUND GIVEN IN 1 89 1 BY
HENRY WILLIAMS SAGE
Date Due
JUN 1
950
TH^
^wr°z
$s& a .t
1?g$-MP~
1^72 F
iiUV 1
DEC. 1 4
497^
■titi^r&mf
Cornell University Library
GV1451 .M64
Chess gems: Some of the ( finest examples
3 1924 029 919 663
olin
a Cornell University
J Library
The original of this book is in
the Cornell University Library.
There are no known copyright restrictions in
the United States on the use of the text.
http://archive.org/details/cu31 92402991 9663
BY THE REV. HORATIO BOLTON.
White to play and mate in ten moves.
IYANHOE.
BY WILLIAM BONE.
White engages to mate with his King's Bishop's Pawn
(f5) in twenty moves, without taking the Black Eook, or
compelling it to move.
CHESS GEMS:
SOME OF THE FINEST EXAMPLES OF CHESS STRATEGY,
BY
ANCIENT AND MODERN MASTERS.
COLLECTED AND SET BY
JOHN AUGUSTUS MILES,
FAKENHAM, NORFOLK.
WITH THE SOLTJTIOIsrS.
" Qualis gemma, micat, fulvum quae dividit aurum,
Aut collo decus, aut eapiti."
Virgil's ^Eneid, X. 134.
"A thing of beauty is a joy for ever."
Keats.
Jjukenljam:
E. W. SOUTHWOOD, BOOKSELLEE;
AND THE EDITOE.
1878.
&
Price Eighteen Shilling s .
' WK'IVI Kur, y
I I : !• I, h V
/\.U^\4-\
M)|0
HU)
YI.IJ'.M'.-HMMU
TO
SIR WILLOUGHBY JONES, BARONET,
OF CRANMER HALL, NORFOLK,
THIS BOOK,
AS A TESTIMONY
OF
SINCERE EESPECT, AND HIGH ESTEEM,.
is trtbitaitb'
BY
HIS FAITHFUL AND OBLIGED SERVANT,
THE EDITOE.
PREFACE.
Eighteen years have elapsed since I made an attempt to collect
and arrange some of the finest Chess Problems extant. During that
time so many beautiful compositions have emanated from authors of
all countries, which are scattered among various newspapers and serials,
that it appears to me very desirable to collect and arrange the choicest
of these, and, at the same time, to make a better and larger selection
from the works of our two great English Masters, Bone and Bolton,
the decease of both of whom we have now to lament.
Messrs. J. and W. T. Pierce have given us an excellent collection of
Problems by English Composers only ; but this leaves ample scope for
a more extended selection, such as I have endeavoured to make from
the works of the old Masters, and of the composers of every nation.
In doing this I have deviated from the usual course of separating the
problems of each author, and arranging them according to the number
of moves in each ; and have, instead, placed them together in chronological
order, wherever I have found it practicable to do so : thus exhibiting
the progressive change of style which may have taken place in any
author's compositions,
I have given in Chapter I, a small selection from the works of the
old Masters of the 16th, 17th, and 18th centuries, in studying which
we are struck with their simplicity, and freedom from the depth and
intricacy of the problems of the present day : yet the conceptions
embodied in them are very fine. Ercole Del Rio (Hercules of the
River) was, indeed, well worthy of his name ; one of his problems
(No. 13) exemplifies two of the greatest beaiities of strategy, and in
the next one the modern style is foreshadowed in a most remarkable
manner. "With the present century commences a new era in problem
construction ; Mendheim in Germany, and Bone in England leading
the way, closely followed by Bolton, D'Orville, Anderssen, Brede, &c,
the great Masters of their day, during whose reign, so to speak, Chess
strategy made great strides. With the brilliant and forced sacrifices
and constant checks of the earlier composers there began to be mingled
the quiet coups de repos, which so eminently characterize the stratagems
of the present day. Bolton was, undoubtedly, the great pioneer in
this track, and it is worthy of notice that, in his later compositions,
these fine waiting moves occur more frequently than in his earlier ones.
Of these great Masters many retired from the arena some years before
their death, and were succeeded by another race, among whom we
find (facile principes) Bayer, Healey, Grimshaw, Andrews and many
others, the style of whose works is seen to be still gradually changing,
hard blows giving place to quiet manoeuvres. Compare, for instance,
the problems of Mendheim, the hard hitter, with those of T. Smith,
who seldom strikes a blow till he gives the final coup de grace !
Although many of the authors of this epoch are still living, some of
them, alas ! seem inclined to retire on the laurels they have won, and
to leave the field to be filled up by yet another race of strategists, the
heroes of the present day, whose name is becoming Legion, and whose
motto appears to be " Difficilia quae pulchra;" for the simplicity of
the olden time has given place to subtlety, depth, and difficulty.
Instead of the problems in many moves, in which the old school
delighted, and which, for me, will never lose their charm, we have
difficult problems in three and even in two moves. As examples of
extreme difficulty the following problems of recent date may be cited ;
No. 433, which gained the prize for the best three-move problem in
one of the tourneys just brought to a close; No. 581, No. 726, and
No. 732. A fine composer and strong solver calls this last the most
difficult problem in so few moves that he ever saw. Let these and
others such be compared (after solving them) with any of the problems
in Chapter I, and the enormous increase of difficulty in modern
compositions will, at once, become apparent. In arranging the
problems of William Bone I have followed the order in which he
arranged them in his own collection, and which we may fairly assume
to be that in which they were composed. They are selected from
many hundreds, and several of them do not appear ever to have been
published by him ; the fine stratagem on the frontispiece being one
such. The Eev. H. Bolton's are arranged in the order in which they
were published as nearly as I have been able to ascertain. The
beautiful position "Camilla" which adorns, the frontispiece would
have had its place near No. 160, but some years ago it was discovered
to be unsound, and I had to omit it from the collection ; I believe
however, that a slight alteration has corrected it, and I feel great
pleasure in being able to present Camilla to my readers, sound in
wind and limb, as the great huntress should be when she
" scours the plain,
Plies o'er th' unbending corn, and skims along the main."
I have inserted so large a number of the works of Bone and Bolton
because one of my chief objects was to make a fine collection of their
problems, and to offer it to the world as a tribute to the memory of
two dear friends. Indeed, without this inducement, I should probably
have hesitated to undertake the work at all. The problems of J B
of Bridport and of T. Smith must always take rank as highly finished
and beautiful compositions ; and those by E. A. M. M. of Mhow in
India are, in my estimation, gems of the first water. No. 245, which
I have named the Koh-i-noor, was considered by good judges at the
time of its publication, (about a quarter of a century ago) to be the
finest problem extant. E. A. M. M. is, or was, an Englishman : I
know not whether he is living or not, and I have therefore located
him between the last of our native composers who has passed away
and the first of the survivors.
Of the problems of other composers, deceased and Jiving, I have
made as large a selection as my space would allow. Of two-move
mates I have been sparing, giving, with a few exceptions, only one
such by any author ; and the same remark applies to suicidal problems.
In the selection of the problems of living English composers I have
been most kindly assisted by many of the authors themselves ; and my
thanks are due to Mr. H. J. C. Andrews of London for much valuable
help in sifting and scrutinizing the problems of Bone and Bolton : and
to Mr. John "Watkinson of Huddersfield for kindly placing at my
disposal his fine chess library and voluminous collection of problems.
The notation I have used
for designating the squares
in the problems and in the
solutions is the German
one, which I consider to
be especially adapted for
the purpose, being more
concise and clear than that
still in use in English
books, which I hope ere
long to see superseded.
This is now so generally
known that an explanation
is scarcely necessary, but for the information of young students I here
give a diagram.
Problems have been called, not inappropriately, the Poetry of
Chess. We might carry our fancy even farther, and say — they are the
flowers of Caissa's garden ; charming us with their variety, delighting
us with their richness and delicacy of structure, and appealing to the
aesthetic sense, in common with all else that is beautiful in Nature and
in Art. Viewing them in this light, I may say, with Montaigne,
"I have gathered a posie of other men's flowers, and nothing but
the thread that binds them is mine own."
a8
b8
c8
d8
e8
f8
g8
h8
a7
b7
c7
d7
e7
f7
g?
h7
a6
b6
c6
d6
e6
f6
g6
h6
a5
b5
c5
d5
e5
f5
g5
h5
a4
b4
c4
d4
e4
f4
g 4
h4
a3
b3
c3
d3
e3
f3
g3
h3
a2
b2
c2
d2
e2
f2
g2
h2
al
bl
cl
dl
el
fl
gl
hi [
Eakenham,
June 1st, 1878.
LIST OP SUBSCRIBERS.
The Honourable and Keverend Kenelm H. Digby.
Sir Willoughby Jones, Baronet. (Four Copies.)
Abbott J. W., London
Andrews H. J. C, London, (Three Copies)
Andrews T. J., Fakenham
Atkinson Rev. M. A., Fakenham
Baker C. T., Holt
Ballantine W. A., New York
Barclay H. F., Woodford
Beck Horace, Harpley
Birkbeck H., Stoke Holy Cross
Birkbeck W., Thorpe
Bolland Rev. W. E., Bedford
Bolton Mrs., Thorpe Hamlet
Bone Mrs., London
Boustead H., London
Bridgwater Walter, Birmingham
Brown Wm., Titchwell
Bryning J., Blackburn
Bucknall R. C, London
Carr Mrs., London
Chambers Thos., Colkirk
Chambers T. 8., Fakenham
Charlick H.. Adelaide, South Australia
Claremont, — London
Coates W., Cheltenham
Column J. J., M.P., Norwich
Conroy J. A. Londonderry
Cook B. B., Hoboken
Cooper A. H., London
Cowles W. B., Fakenham
Derby Free Library
Derry T. R., Belper
Dewing R. W., Fakenham
Duffy P. T., London (Two Copies)
Dyson Edwin, Huddersfield
Fakenham District Library
Finlayson W., Bridge of Allan
Finlinson J. H., Huddersfield
Fluck C. J., London
Fornival W., Wolverhampton
Gilberg C. A., New York
Greenwood W., Croskills, Leeds
Grimshaw W., Whitby
Grummett G. V., Fakenham
Gumpel C. G-., London
Gurney John, Sprowston
Gurney Reginald, Fakenham
Gurney Someryille, Lynn
Hardingham F. R., Fakenham
Hamond J. B., Stockport
Hoffiday Thos., Huddersfield
Horsley Chas., Fakenham
Huggins F., Hawstead, Bury St. Edmunds
Jones Rev. H. W., Sculthorpe
Kent Rev. F. W., E. Barsham
Kidson H. E., Liverpool
Lee Warner H. J., Walsingham
Lee Warner Rev. J. , Thorpland
Levien Rev. J., Bumham
Mallock D. M., Clinton, Ontario
Manby F.. Rudham
Manning J A., London
Mason J. W., London
Meyer H. F. L., London
Middleton E. P., Hindringham
Miles J. G., Saxlingham
Miller T. J., Jr., Fakenham
Murray A. K., Bridge of Allan
Nash Wm., St. Neots
Neame Chas., London
Onions G. F., Bradford
Overman John, Bumham
Page G. W., Fakenham
Pearson J. H., Field Dalling
Perkins F. V., Manchester
Phayre Rev. R., Raynham
Pierce Jas., Bedford, (Two Copies)
Pierce W. T., Brighton
Pope S. R., Colkirk
Ranken Rev. C. E., Malvern
Ridsdale Rev. G. J., Raynham
Robinson John, Cleckheaton
Roe T., Jr., Derby
Sewell P. E., Norwich
Shaw W., Montreal
Sheringham Miss, Fakenham
Sheringham W. G., Fakenham
Sillett Miss A., Fakenham
Simpkinson Rev. J. N., North Creake
Skipworth Rev. A. B., Tetworth Rectory
Slater G. J., Bolton
Smith J. D., Mayor of Norwich
Southwell Thos., Norwich
Southwood E. W., Fakenham
Steele Joseph, Croydon
Studd A. E., Oxton, Exeter (Two Copies)
Taylor J. 0. H., Norwich
Taylor J. P., London
Temple-Lynes C. J., Blakeney
Thomas S. H., London
Thompson F., Derby
Thursby J., Burnley
Townsend A., Newport, Mon.
Ward W. G., Mayor of Nottingham
Watkinson J., Huddersfield (Two Copies)
Watson G. A., Walsingham
Wayte Rev. W., London
West J. C, London
White Chas., Brighton
CHAPTER I.
TjHIIE -A-ZLvTCHEaSTT MASTERS,
To THE END OF THE EIGHTEENTH C/ENTURY:
Numbers 1 to 48.
GALAHAD
WHITE TO PLAY AND MATE WITH HIS PAWN IN NINETEEN M0VE8 r
WITHOUT MOVING EITHER HIS KING OR HIS ROOK.
: And there was one among us, ever moved
Among us in white armour, Galahad."
Tennyson. The Holy Grail.
No. 1.
From an Arabic Manuscript in the British Museum,
upwards of 500 years old.
1 1! B
mt
^W;
:. w I Hi ^-* ill
HI » HHi
i#a
ij H .1
?:--'%
m J mm vm ~~
HI
fa
\ . W/xwM W/////A
€ri mm
m
m ■ ■ ■
White to play and to give checkmate with his Bishop in
twelve moves.
No. 2.
From an Arabic Manuscript in the British Museum,
upwards of 500 years old.
a&
mm ^m
mm
W/a mffifo
4Mii
White engages to checkmate the Black King on the square
he now occupies in fifteen moves.
1
No. 3.
By Damiano.
tHHP _JL^ P
r*>rW
:Ti
M~^JStk
.&uUB am Jm,
MW'\u g *
■jy""-
....A
w mm
J
White to play and checkmate with his Pawn in four moves.
No. 4.
By Damiano.
^P#
Wm, y *JmmyJmm f I
m mJ m m
mm ml,
mm ill
White to mate in four moves checking every move, but
not to mate with the Bishop.
No. 5.
By Damiano.
J * i Z- d -9B, ■ -
"mm /
wm.
UiO
m 'mm 'mm m
t
6
^ 1
White will checkmate with his King's Pawn in five moves,
without moving his King.
No. 6.
By Damiano.
'm
C„„„„,«H fSSS
i
wm§^ mm i
HHP PHI P
JUL si
V-'..
tf i
mm
up""
3
White to play and mate with his King's Knight's Pawn in
ten moves, without taking the adverse Pawn.
No. 7.
By Captain Bertin.
mm. W
■IP— - .
r.
up
% w//M,
ill
HI M
■ 111 i
i-ft-iSl
"p^i s™
"White to play and mate in eight moves.
No. 8.
By Stamma.
BLACK.
^
i
M i
....M,^r„mA
m w J m ' "
w,
White to play and mate in three moves.
No. 9.
By Stamma.
m
.
iiL ; = :
iri i jBf
1 II ^ IP
m
mm,,,.,, WuA
mJL.gm
m m
*: !
I ■^J54
m
'.■■/. w, »s
H
"White to play and mate in five moves.
No. 10.
By Stamma.
///7/A '//////////A
mJt
Hi
lJLhi
asp
,^§Sls3^ ^HH %%mw s%§HH
w, 'mm, nm ^ mrn
^^S
White to play and mate in six moves.
No. 11.
By Stamma.
n IJIJ
IllJlill
11
mxwii. ~ az -' w/mM. ty/z/Jmy, tff%Z%>/,
musm. -///////////,. '////sZy/A. . m
"1
My//////;.
ife
r 1§ s up
^ vyy/////>
%t
White to play and mate in nine moves.
No. IS.
By Del Rio.
'mm mm «
|fl§ %f% mm
^-////////J , .....;■■"■
m m
/%. //////////:
msk
mwmm.
mm w%m w,
W.„ §»
^m
w/.-^^mm
wmr
^'liiteto play and mate in three moves.
No. 13.
By Del Rio.
\ W III
wmk ^ illl ,_„ , Jill llli
H © H i fll
«^^P * a
F2p
i „„.ftn
2^?
?
1 f
fwTl
White to play and mate in four moves.
No. 14.
By Del Rio.
wgk mm m
mm mm Jam
warn m. m
III
M
m^i,Wm
■w ^ m WM y0- wm
mmt ^^«^« J
White to play and mate in four moves.
No. 15.
By Del Eio.
; *11*I1 11........ II
•PUP A, 'WW? HHIP
w mm.,.
PI IB
w, 'mm.
ttmMM
' i wrTmm
'mi
iJZLMM
"White to play and mate in four moves.
No. 16.
By Bel Rio.
j -Hi /- > i
iii
W////Z
%%.,
9,
m.
m W wW
WHITE.
White to play and mate in four moves.
No. 17.
By Del Eio.
-W-LJJ1
f •
m X illl
■%%m&,, 4m!m4i„ m *^/z*.!„.TTL»l«8s*8P
^« Mil
PHI
! O ^41
iSI
White to play and mate in five mores.
No. 18.
By Del Rio.
«
i
1'
W
JlJSfJilllL ifa
IP I
■. HI »l.
P^ M^ IIP J»N S
ilil ffffil Ilil /5a HI
Hii ^§Hl tfH *£J m
m,„„„„„^Pi^_^^*^^^^^^
White to play and mate in five moves.
No. 19.
By Del Rio.
BLACK.
■
m «l mm
m wm^ >y-
W,
!«j*Hili JL
ill
m t/ JMM WM 7 WM
White to play and mate in six moves.
No. 20.
By Del Bio.
PPP P
■ if 1 b
f
^^^^ww^l
■'-^^P^'—M^-^^M
, yfo&
m wm mi mm.
WHITE.
White to play and mate in seven moves.
No. 21.
By Del Rio.
BLACK.
Hi§
'jmk wm
ill ill w /m w ,wm^
mm. mm,.
m—j*
m HH
B..1..I1
11111 "^ ifyl ^«
M MS
White to play and mate in ten moves.
No. 22.
By Contarelli.
i • 1
W"
^
W"'*
"White to play and mate in five moves.
No. 23.
By Contarelli.
WM W,
m.
f*1
Wm?
w/////// '-mrz/ //Z//// ■'////«'/■■'
L-Mi
.wmk '^^ / - ■////////// ^^^'^....Wifo
I
H&JP tw
up
n liH SI
y////y////fi /////// "'-////
m.
w ^h
H iHf H
^
White to play and mate in ten moves.
No. 24.
By Lolli.
m wmi mm 'm
m. mm, ^a„ .. fJH
Hi
ill
urn mm
nj mxffii
■i ■..^.■^„.bL
ii^ i
%
JML mm
B^^Hw '
I ill WB^\
\m^~m
mj±mm
w$, W; W>. wm Wlm,
White to play and mate in five moves.
No. 31.
By Ponziani.
mm mm mm,
4nA
mm 'mm, '■mm. mmt mm
— i mm k mm k
*m\rTm
m mm mm^/mm
mm. mm. m
m Wi/i
mm wm^=*wm.
White compels Black to mate in five moves.
No. 32.
By Ponziani.
w mm
m m m m
mm.
w mi.
'i
%^A,,,,,,,,,,fm3,„J^A j™ a
m. mm
' A m
mm mm
Wm 'mm
Wmb wml>,
mm
-...mm
WHITE.
White to play and mate in seven moves.
No. 33.
By Ponziani.
^w ^m il^#
fis ^jl8 ftp
White to play and mate in seven moves.
No. 34.
By Ponziani.
wf% 'WW}<
mm
WM wm
fit ill
m mm,
mm?,.
mm, warn
WM, 'WM HH1 H
w, mm.
■ '■;
^m mm, mm
WM, wm_
White to play and win.
No. 35.
By Allgaier.
|PI
;.;.! s ill
pip n^^
in
i "WV^W
,W/////////.„
in
OV////////A
« WM™ 4m z>:X,
Ill §§p
"White to play and mate in five moves.
No. 36.
By Senupah.
%%? 2%
pv ■>■ - »
■§ I
mm.
*L««J^«L«. ;
■
W?A
m ' ■•;
1 m§I1 tllll
White checkmates with his Pawn in five moves.
No. 37.
By Senupah.
■mm
WL
3m^m
flip '" a
A ^^HTB
4M, « r §,/fi f ^M.
■I
■Lr
ill 111 ■
jM| Jpn
WHITE.
White checkmates with a Pawn in five moves.
No. 38.
By Senupah.
HP
1
mm^Wm
w
-MiM,,,
m
m warn
■■ W fa.
il ^^l^^fc^,
i a
!8
MM
W//M
"White checkmates with his Queen's Pawn in nine
moves.
No. 39.
By Senupah.
w.„ ■*¥?■
m m
m y 'w&m, Ja^A
n -I it
^
w^wm. . ;
r ^ w <-> wk JL™ r ™
iHl ^B ^ Jill * i.
HI IBP * *
m i
WHITE.
White checkmates with a Pawn in eleven moves.
No. 40.
By Trevangadacharya.
y mm %
%HH W0W/, zs&gg? VBX&%
1 „■ I IB
iff
MM
1 Jf&^^St^im
'mm
warn m, m
II ^^
WHITE.
White to mate with his Pawn in four moves
without taking the Bishop.
No. 41.
By Teevangadachaeya.
ai.'
m&
i
^z^r^mM^^wmz ^ mm,
111
w v wk v mm.
mm,
%
wr
White to mate with his Pawn in five moves
without taking the Bishop.
No. 42.
By Teevangadachaeya.
iifei Wl
wmi.. =-„^m.
B Jfcjj
i
i 1 JL
ill
I
mm,,
■mm. m
mm
m mm. ^ ;
White will mate with his Pawn in five moves, or
compel Black to mate in eight moves.
No. 43.
By Teevangadachaeya.
BLACK.
Hi
'• Ww% A HI
IIP jl ill
■ 1
ml im .„ nm
~ ■ ■ B
y/M, IK, ^
^SPil
#81 ill ill fii
«
0//////>M
WHITE.
White to mate with a Pawn in seven moves.
No. 44.
By Teevangadachaeya.
'mm
m ww
6 i ''M'tiit
■■/■'
#////////%>,
wm ' "w
■v?\
ww mm wr&\
I
«
IBjIJilL ill.
iff" ;;,;,: Im
i» i?M Hif tiPi
^ W//////A ^H§y %^Si
Bil
'WvW wWW ^fe§S§?^„
:■■/■'■/".
ww. ww. wm *Wm
White to mate with a Pawn in eight moves.
No. 45.
By Trevangadachakya
W% Hk HH
lisp 8
fi
■ML
wn.
*L~JBIA. ill
fMk p
JP
m m r m
White to play and mate in nine moves.
No. 46.
By Trevangadacharya.
^^ Ml
HP
■1
HI
^
i ^»„
jdi HI
■ ^ ^p «p «p
jiji #1. ■
"V *■■
ISli
^, W//////ML
'**
,r;
,^^i v/y/Mwfr mh^ r : "
WHITE.
White to mate with his Pawn in eleven moves.
No. 47.
By Trevangadacharya.
: ^
« I*
m
wm m Wk..
<\MA
^^%^ M W ^M
'"^ %//■"■
m
"HP
■iW^A vxwM
W W
i
Wf
wM
mm
^ - B
/,..'/.
WHITE.
White to mate with a Pawn in 'snLteen moves.
No. 48. Hercules.
By Trevangadachaeya.
W WM,
■ mm .
m mm
vmz. §§§i
HS5H , . 'fiGZfr. IfiSI Mm?/,
if im mt
\ i If i B^\
7//,.,,, 'A
..»,/;.
..III lU M In
(*) i
■ ■■
^^
11 (
White to play and mate with the Queen's Bishop's Pawn (c2) in 82
moves, without taking any of the Black Pawns, or allowing any of
them to move. Given in 81 moves by Trevangadacharya. Solved
in 89 moves by Rev. H. Bolton, and in 32 by J. A. Potter of Salem.
Mass.
CHAPTER II.
EUGLISH COMPOSEBS :
Deceased Numbers 49 to 248.
Living Numbers 249 to 448.
THE PROPELLER.
BY THE REV. HORATIO BOLTON.
i i & f«I
• #S5^ ^fe^i ^^M ^™ %fe«M
H a fl ill
£/^F.'
Mi ffi^?///i
§ H mm
-
Mi
■WHITE TO PLAY AND MATE IN TWENTY FOUR MOVES.
" monumeutum sere perennius ;
Quod non imber edax, non Aquilo impotens
Possit diruere, aut innumerabilis
Annorum series, et fuga temporum."
Hobace. Odes, III. 30.
No. 49.
By William Bone.
.'■■ ^
1IIF
mm mm x lip §»
mm
White to play and mate in four mOTes.
No. 50.
By W. Bone.
WT
mm,, Jm
ill
il
7:
m M ^mK^ Bm §Sml s ^m
JL
03/A
ft if
^5»ft -fi^m Mmffl-,
"White to play and mate in six moves.
No. 51.
By W. Bone.
Ill
was* vJ^'-mw ^ / w / w^'ww
/,y MWA . '''VMM. ''WilMZ"' "" / ' / MW/,
I * ill * ™L w
nU^iW ■ ™ fe ^
H W'M.'^~W//Jm
V"
White to play and mate in six moves.
No. 52.
By W. Bone.
mm
i PPI ^
n
I i
m tm wm.
m fill
w
PW '1
I
m si
M
WHITE.
White to play and mate in seven moves.
No. 53.
By W. Bone.
BLACK.
mm. mm.
m
»l m #1 ■
■
W
fi
111 111
WHITE.
White to play and mate in nine moves.
No. 54.
By W. Bone.
m
i§P\Ui/
mm
•/////////A
PUP k
JKmJLj
HP 111 6 HP
«1 jB_JHL 9, Jili
iAI iPM
i
wma
mm. jflL m ■■
White to play and mate in ten moves.
No. 55.
By W. Bone.
PUPP W
WM.
m WWA el
mm
*% m
J. wMm. vEm,
■
WTdbP*
=— ^gS^rt®
I A £1 ffll IB.
mm
White to play and mate in eleven moves.
No. 56. Agincourt.
By W. Bone.
i i ail
/« VZr////ss/.s
4m
III-
''WOffl', W%%%?, W//itf//, ?//,
.if M 1^ *"
wry
Mitpfi
H
'^
,-f^™^-™
^» ^mm//.
White to play and checkmate in twenty moves.
No. 57.
By W. Bone.
■
£3 ■
Pi flB
ws/m. '-mm. wm
In
mm mm *%m A Wjk A
fifi i ■
HP
i
^ Jill
8
sell jiw
% /■
White to play and mate the Black King on his
own square in seventeen moves.
No. 58.
By W. Bone.
W3 ¥
PV
i Ill
^
m mJ m mm
m Wm. , wmk,,„, i
wr"
m ;wB,,,ymm
^ m J Km mm
M,.,.,..,,„?mm
£Q)A
White to play and draw the game.
No. 59.
By W. Bone.
ill
'mm.
JSy--"^"'"' '
4a ■&■/////&„
f
■f-^a
i HI lifi
■ B
i ra #yj
m
A 1111
White to mate with either Queen's Pawn (d2) or
King's Bishop"s Pawn (f2) in fire moves.
No. 60.
By W. Bone.
w, w I
S 'w/w/FT"m
wAMJsLjm. mm
ffsAf mm
mm wm, mm,
White to play and mate with the Pawn in five
moves without taking the Queen.
No. 61.
By W. Bone.
BLACK.
W&w/M
mm
i i i f '
W£W%
mm.,,,,,, wmM,
m
i r
— I »1 ^^
White to mate with the Pawn in twenty-seven moves
without taking the Rook or Rook's Pawn, or compelling
either of them to move.
No. 62.
By W. Bone.
Ml _fc
m -J Km m
11 B
■ ////,9 / ////
dm pf
WW mw WW
I £§
mm, i
White to mate with his King's Bishop's Pawn (f5) in eight
moves without taking the Black Queen.
No. 71.
By W. Bone.
BLACK.
"5ZZT/Z v/y// / // // ////////
isli
%a_m&sm
Mm*
m
rf*ili3
wa m
m
White to mate -with the King's Bishop's Pawn
(f2) in eight moves.
No. 72.
By W. Bone.
^, ^^H, Wow,., , v , VmmM
™ "■d^aJf^B
^ V////////M
PI 11
i§ gi* lMi Uli
iHH. «T
^ 9m,,^wm.
w . mm.
■
White to mate -with the Pawn in eight moves
without taking the Bishop.
No. 73.
By W. Bone.
BLACK.
■ «■ Mi I
i
B
— w
in'
§§ip
^8, WttZM
''warn. ''mmd'"' ' m m
y/ ^^m,, ^A m
White engages to mate with his Qneen's Book's Pawn
(a2) in nine moves, without moving his King or the Knight
at Queen's Knight's third square.
No. 74. The Triplet.
By W. Bone.
PmMvf
mmj m m
iH
jmj \
m%
m mm?/,
wm, „„JMm
V wT 11
jjjl P
^ Jill J
White to mate with the Pawn at h4 in nine motes, without
taking the Black Pawn or allowing it to move.
"White also engages to mate with the Pawn at g3 in 10 moves,
checking at the 9th with the Pawn at h4. And to mate with the
Pawn at h4 in 11 moves, checking at the 10th with the Pawn at g3. .
No. 75.
By W. Bone.
1 mm
wm W"
ia /mm.
White to mate with his Queen's Knight's Pawn (b2) in ten
moves without moving his King.
No. 76.
By W. Bone.
Hi . wm^'wm, i
* JgLj^fji ; '- j
w, JB
■*"**
mum J^ tKt ^J^^J^
mm
J
m m
■L
M
™ „„$/.
ft
w
White to play and to compel Black to checkmate
him in ten moves.
No. 77.
By W. Bone.
wm„^J4Bm. ill II H
e«
■ H ffli
up
HI X ^^
™.„, ^p
I »
■ Hi ■!
HI « ^
White to mate with the Queen's Bishop's Pawn (c'2) in
eleven moves without taking any of the Black Pieces.
No. 78.
By W. Bone.
Ill
F
I HlM
White to mate with the Queen's Knight's Pawn (b4) in
eleven moves without taking the Black Queen.
No. 79.
By W. Bone.
1 W,
'WW M
H^r*
ww, » mm
*L0mL*JHM
warn
% JNM
I
111*111
^1? WW/
warn. %
iM*mL-M
''WW' r~^ HP
w.
■ft
iiil t—* WW,
WHITE.
White to mate with the Pawn in twelve moves.
No. 80.
By W. Bone.
m W"*
I s
'///////'A '/////////M tmo
mm mmm jmm
ww. w
M
m ik e.
HH §■! H§H §111
WtM, a UTaH 'WW
mm fi fit
§
W/W /\ w,
-'mm 'WW, %
White to mate with either the Bishop's Pawn (o3) or Rook's
Pawn (a3) in twelve moves without taking the Bishop.
No. 81.
By W. Bone.
m m
wm..
%....
*«"«■
%i#„ ^^r^ i
B
1..
r ^^m r .^ > W^
M
"White to play and to compel Black to mate him
in twelve moves.
No. 82.
By W. Bone.
% w.
■JLPW".
m
m.
bJLs
'ill
fSl HI I
^L.'^L ^
White to mate with the Queen's Knight's Pawn (b2) in
thirteen moves, without taking the Knight, or compelling
it to move,
6
No. 83.
By W. Bone.
up
m mm.
ytfe
^
wnjfwm
11 |§J
I ^H ijBB
White to mate with the Queen's Knight's Pawn (bi) in
fourteen moves, without taking the Black Book, or compelling
it to move.
No. 84.
By W. Bone.
'S'
m
111 . 'Mm,
W warn
&-wm m
IP
mm
mm
White to mate with the Pawn in seventeen moves without
taking the Queen or Book's Pawn, or compelling either of
them to move,
No. 85.
By W. Bone.
^ill . mm mm, mm.
warn
P
•■
Ill
i
an*
mi
mm mm
'/ A V //.
- HH w
It
White to mate with the Pawn in eighteen moves, without
taking any of the Black Pawns, or compelling them to
move.
No. 86. The Ambuscade.
By W. Bone.
BLACK.
s "'- npfi ' i
■
s a
I p 1
; ^^^.„„„„„ r ^^^,^^ / J^ ! ^'
W"
it
W", rrrr . r ,S' / '"""^
White engages to mate with his Queen's Pawn (d3) in
eighteen moves, without taking the Book at e3 or compelling
it to move.
No. 87. Richard Coeur de Lion.
By W. Bone,
mm*- V////////A m
m i£!.:
HI
m m^ m L
^"%
; W 2£
HP, iHH ^H " sum
R g%«
W, ess i^ ess i
wok
■
m w
White engages to checkmate with the Pawn which now
stands at his Queen's Knight's 2nd (b2) in nineteen moves,
without moving his King or taking either Queen or Eook.
No. 88.
By W. Bone.
/ 'y////i///// . ip
■iA 4
WW
m 1*..:
til
K^&7%, W/////X/,
elJ.i
H ^^J «=S=. L^
1 ?? 8^8 H«
iH Hi Hi
White to mate with his Queen's Bishop's Pawn (c5)
in 2 1 moves, without taking Queen or Pawns.
No. 89. The Tower.
By W. Bone.
m. mma iHH iHH
IB
%i%S<2 *^ s= ':,.~7%ii {f777^r:.. ,
wm, mm
m ■
^MaH^ r*yJB6m r
White engages to mate with the Queen's Pawn fd2) in twenty-four
moves, without taking the Rook or the King's Pawn, or compelling
either of them to move. "White further engages not to interpose any
piece between his Queen's Pawn and the Black Rook now attacking it.
No. 90.
By W. Bone.
White to play and mate with hia Queen's Pawn (d3) in
thirty moves, without taking the Black Rook at d4 or
compelling it to move.
No. 91.
By W. Bone.
BLACK.
r'.'n
* If
H
m ,,,,,, ,,wm.
% warn
w
mm
W„ mm
i
^^ ^ €Hl
White to mate witli the Pawn at Queen's Knight's second
(h'2) in thirty-eight moves, passing the adverse Pawns
without taking them, or compelling them to move.
No. 93.
By W. Bone.
■ llSi
II il r ■'■--■-■
mwMM 117Z&7&. W/////MM
WHITE.
White to play and mate in twelve moves.
No. 93.
By W. Bone.
ySl *"
m mK J Km
■mm V"™ 1
mm.
*3
I mm mzdi. ilH
White to mate with either of the Rooks in fourteen
moves, without moving either of them.
No. 94.
By W. Bone.
m
m #jS ,,„iwi
ill ■ mm ii
si 4k
■ !■ ■
ii a tt&f
HHP m
m, A
iM^i,
White to play and to give checkmate with his King's
Knight's Pawn (g6) in thirty-nine moves, without taking the
Black Queen.
No. 98.
By W. Bone.
'WM W
k g§§|| f§§|§§
m mm..
m mm
SHHl vw 'jam
mj± mm,;j-^;w,
m 'W mF** fc
White to play and mate in eleven moves.
No. 99.
By W. Bone.
pH wMm w& HHy
''/MM. 'W$M>. W%M. %
/;
^\il w ww
Wa M ;
WW /J\ WW
WW
WW WW,
W WW,
m
White to mate with either Pawn in fifteen moves at most,
checking the previous move with the other.
No. 113.
By W. Bone,
'WW 'WW WW PIpW
•11 wB, ww mm
h ww, mm. ., la
i« ^^
It.
m-jm^^^^mKk
% v///Wl
WwJ
1 WW,
m SJ mm 'v
W&i
/////////s.
'■W///M
V',,^,,'™*"*". — '///////////,
W.
WW, W/W,
White to mate with a Pawn in twenty- one moves,
without queening any Pawn.
No. 113. The Challenge.
By The Bev. Horatio Bolton.
,,,,,,,
■■/'■'■
i II
w-
■wm A
P""a PP"
Ife
v//////////, y ^,^0mW/,,
-■A„ — „^^ JIW -
White to play and mate in six moves.
No. 114.
By Bev. H. Bolton.
mm. ^^
m
i
« I i {fig
i
ffiil mm. wmm, mm
'/y///y/JM r ,
iSl
'■warn w>,
m i
■■-'/////a m
White to play and mate in six moves.
No. 115.
By Rev. H. Bolton.
s ■ in r
' wmm T"wm
ill
■^Sb^mJ^h
H fen.
181
MP
MM WM///M
White to play and mate in seven moves.
No. 116.
By Rev. H. Bolton.
HH
^^■/////////// / '"'" '/// Mw%%
MAm
w,„ mm..
m_m
&MJm€m
mm
White to play and mate in six moves.
No. 117.
By Key. H. Bolton.
■
.mi fc
-%^™_~™~ ™
ifip i H
_1 7 11 i 1
11 ■ ■ •
« 4 ill
«
■ i ■ I
Iff ■
% y fe«>i ilii
!&■
White to play and mate in eight moves.
No. 118.
By Bey. H. Bolton.
IBf 'mm. Ill
m m PI L
Jf^J--^" **
ill
&i
n. «!„..._ ^B
pp
1 «n
WHITE.
White to play and mate in seven moves.
No. 119. The Escalade.
By Rev. H. Bolton.
BLACK.
'■"■iTw p
■ III i iS 81
"3 H ■ P
HI mA-%.
.....
P
Ml Hi
'#$3,
Wa .^w
White to play and mate with, the Pawn in
thirteen moves.
No. 120.
By Rev. H. Bolton.
j IS
» , w^L^m.
/////j V//S//////A
Wa,.., warn
m ^^ WM VM
White to play and mate in six moves.
No. 121.
By Eev. H. Bolton.
MB
mm »1
JM±M m
flLJ"^.(
i
9
MM H»
H
White to play and mate in seven moves.
No. 122.
By Bev. H. Bolton.
-mm. -=C-*"-
§iffl H
H
JL™ii
im
n Mi
=*!&;
mkMkM
"WHITE.
White to play and mate in seven moves.
No. 123.
By Key. H. Bolton.
%H
H
•bfli
ilk
. ill
WSKT
wamt m
w. 'mm..
/m %,
iB Ufa
'wm
White to play and mate in twelve moves.
No. 124.
By Key. H. Bolton.
BLACK.
m
1SI —"mm/-,
HK
III
ymm. i^ :.
iff A Pfl ' 'P
« mmJ±M
fM
IE
m wJPLM m
White to play and mate in three moves.
No. 125. The Arabians.
By Rev. H. Bolton.
—j™^..™.
HP
■AJtePL '
m''' A wd"' A wf m %
^P ill Mil i
HI P
■mm,w'"
™_P"«
m m ^- jm
M .mm
Wliite to play and mate in fourteen moves.
No. 126.
By Rev. H. Bolton.
»™t**J*™**»*^M«
« MM
tfr/M w////////. vrzzr.s
'„/,-/.„■:£
WM W.
HP IIP k IIP k
I mm. mm. * ™1 *
wm&m
W0M WV if PL
w 'ww ww '
'I ■_^™^^"^
1
4m,
4M,,p.:wB,
...m
i
Ws, ^ .
'HI'
^ ^^ fill
White to play and mate in four moves.
No. 13S.
By Bey. H. Bolton.
"iiW™"
, *f : IrI
mm..^Z...j„
...m
mfflk
4^1 SI
J III
pip
- « ijBI.
i%tmi %■//////?/.
White to play and mate in ten moves.
10
No. 147.
By Rev. H. Bolton.
BLACK.
', W/,
A^J m '— / P
Jm
1
Warn wvwi.
i
ISP HP
Vi&&.
mm
White to play and mate in eleven moves.
No. 148. Ghuznee.
By Rev. H. Bolton.
■
iJB IB
iM
WML WMLWm mm
Kt^wm
m i m^L
W®*WM
^^^ IllP ^MM* W
ill mm mm
White to play and mate in eleven moves.
No. 149. The Projectile.
By Key. H. Bolton.
fill ill
mmm i
1
WB x MSB /s »i ^ HP
ill i H^fii 1 fif
I ^P Iflli
*JP PUP Pw
HH HHI §111,,
«
|1 p|
VwssssSJSt / //////////V.,
w
m
p
mm//,,.
mm
a , r £l l
jh B bi
§i #iii
While to play and mate in fifteen moves.
No. 150. La Chasse.
By Rev. H. Bolton.
■...\^±.mm.^T,. PH
m ^F m ^
lill & IS
■I ip
ftp llf
WSSk . wm, IS
pp§ j i i
iff! * Ml HI I. . /;
?>:^ HHSgj ^H §
»
White to play and mate in seventeen moves.
No. 151. La Pucelle.
By Rev. H. Bolton.
wm, mm
P
wm w
Wm WB &\ HI IBi
' ill iH «i ..
«S i III
B. * ..si
1H^
m 'mmFFm
mm j&m
fSi^ill 8 iiiw
j£i
m
Hi H|
White to play and mate in eight moves.
No. 152. The Torpedo.
By Rev. H. Bolton.
*~w™»
i Hi*!. A
m *m
Y//////M
fm Hi 1 4y
r ' ^
iUSi, .JHH1..
m «E1
I "
H HHI s iHH
White to play and mate in nine moves.
No. 153.
By Eev. H. Bolton.
«&& JEHU.
111 H 4m
i . mm_wm,
V6 - 9,
. H
'/■■'>.
White undertakes to compel Black to checkmate him on his
Queen's Book's square (al) in twenty-five moves.
No. 154.
By Rev. H. Bolton.
W - i
fc
P§|U
H & B » 4m
P
jilt hi i i
H ^H mm.
White to play and mate in five moves.
No. 155.
By Eev. H. Bolton.
WWM pp
mm^WM_
mm
mi.
?&&>,
w
mm mw,
§a £&f Warn
fiBjpl
ilflll 4MM> fi osL'*
^p as
■■'*% *■
I iBf »lf illl
White to play and mate in six moves.
No. 156.
By Eev. H. Bolton.
BLACK.
m
km MMA i
i^ei ■^ISI^
§1 ill
1 #Jli H5i ^m
V////////// //////////// W////////S, o
If *
ni ■■ »«
1^1
«f
m ww M
WHITE.
White to play and mate in seven moves.
No. 157.
By Rev. H. Bolton.
vW-
mp n
mm. mm.
P
wzk~\ %~~\u~\
'Z ■- .■ . ''
™«£=LiK»J:
m„ wy/////fa
III
■_*■
m 4mm
^^p
J
wB,
White to play and mate in eight moves.
No. 158.
By Rev. H. Bolton.
""% 77Z.. %:■ "■ - ■
wm 'W§r^n t: &
.. - . ..-.■/.
y///v////^^^'''My^Z //, ^^'Z///"//y
flM^^j/m 0A
m
I Ml
^1 ^H SI Ill
IflP H
White to play and mate in ten moves.
No. 159.
By Rev. H. Bolton.
P
w>. Uta
mm
m mJ m
Wmk
- mm
fH. 111 wA
-mm w
m w Fm *
«
i
bJ^VJW"
i ^P jfej « i&J
White to play and mate in twelve moves.
No. 160. La Finesse.
By Bev. H. Bolton.
wet *t* ^P ^ ii
II ^B
^
m .„. . i^
rii
ML*. HH » « Mi
£ 1
**JkM iL,
Hi
■
■ . |»! H Hi
™t^> y/ /,,~,., < ^ / " i W///// ™^W///W ™™™-
W///M&,.
mm.
'WM& WW>,
White to play and mate in nine moves.
No. 161. The Revival.
By Key. H. Bolton.
■mm
wm_ am * i
III H&'
mm, £11 w
HP 111
mm. mm.
ill WB.
WHITE.
White to play and mate in five moves.
No. 162. The Enterprise.
By Key. H. Bolton.
'jIB
m
■ IB
III
wzMi..Q..,0m.
m 111
w
mm (■
ill
"ip p
mMi
,m 9
White to play and mate in ten moves.
11
No. 163. Moultan.
By Rev. H. Bolton.
%& ■ Hi
mm
i $ mm
111 Iff iP Hi
JUl iO Lwl mm
i^p p^^ ^^ ^
m^^Jmm,
in . Hsfe
2^
YA—Jmm..,
m.r
mm
White to play and mate in seven mores.
No. 164. The Attack.
By Rev. H. Bolton.
'//A ■//.
mm.
Wfe»iff US i !
'/by///////, yy////////,.
W 1 ,
W^jfll
111 mm
i
j^» " r „„„„„ v// "" //M ZJ*^«:
* ^H.
m 'MB.
WHITE.
White to play and mate in ten moves.
No. 165. The Rivals.
By Rev. H. Bolton.
I 'W3, WM
H wmk
■twm. ... ip
J
nw m' mm, •
i M
lifl
I mm,.*mM
&®
ph
MP
HHH
X
IP A •
^» ##i
White to play and mate in ten moves.
No. 166. The Propeller.
By Bev. H. Bolton.
m
x W* WW' mM fA
I of
^ iUB „ mm.
II i
HP I
HPf ^HP :
■ i In
m, „,^5^M
White to play and mate in twenty-four moves.
No. 167. The Electric Battery.
By Rev. H. Bolton.
w -ww j^B,^ Jwm.jMm.
"mm hi wm a
mm 'W i» .ML
i
I ^ Mi wmj*jfflm
'MM?,. W
'wm& x *g®
Y///WM
m *M m W>
lis :,„. «1 Isi
White to play and mate in fourteen moves.
No. 168. The Research.
By Rev. H. Bolton.
m *«"*i £1 Wi
■
m Hill
»J^J8B»
^^ H IHP
ill
1
..„.§!I Q lill
White to play and mate in six moves.
No. 169. The "Warriors.
By Bev. H. Bolton.
"§=■=* ■
w
ill
S ill^lSJ
M ™ — ™ *
H
wk WJk 'wmk
?;'?■■% . ^^^ ^?%% ^-^
IIP fiPF
Wm mi wot
fUH
g;S ^
White to play and mate in sixteen moves.
No. 170. The Leviathan.
By Bev. H. Bolton.
■d™« ™-
^
».
HP
^ m^
A-
m
yz.
Mm.M-
mm
fli & III
111 lb
m& W/foms/, l
White to play and checkmate with his King's
Knight's Fawn (g2) in twenty-four moves.
No. 171.
By J. Brown. (Temple).
w, & iHH #JH§ warn.
WW,
AiU m -jM W JM
Hi s
U -mJF m m
m..u.ww, p
in
m, %&&
WM 'Wk%
m
m "w,
m^^,& WWy? '=' WmW, WWW
White to play and mate in seven moves.
No. 172.
By Elijah Williams.
■
mm
JaiUlI
WrW, WW, *§H* WWW
£Qi, mm . ■■. mm..
mm m mm.
WW
mm
WWW ?/////,
i „„„.iSI,„,*../^ kl wm.
' W *** m
WWW. &. WrW
an!
i„,xr„ ^S^
White to play and mate in five moves.
No. 173.
By R. A. Brown.
BLACK.
WMA ^^ WM
'Wm ... m
mm. warn
WHITE.
White to play and draw the game.
No. 174.
By J. Barnes.
'y/, '#/,
m
£#J fgJi illt
■All
fift ffliJI
■i
m W"w
1 & |Sj
m^ nm
White to play and mate in five moves.
No. 175.
By W. Lewis.
m w,.
^8BL '
Hi * ■ III s «l
§lsi , ppf| ^am
•Ml i 1111 »l U
"-"it-*
^
pil ■ m l
«p nm 1
White to play and mate in four moves.
No. 176.
By W. Lewis.
WWM ts=s 'WZM
W/ ''ww-
■
i
|H§§
m.
ifftii
o
"IB
i * KB *
P. ^S^J
^^5
t«
■ : til HH
White to play and mate in eight moves.
No. 177.
By W. Lewis.
W
HiP'
iMik 4m,
ml i
£ uBj ill si IS
■ B i B fi ■
HrrrfTL
mm
.my/M4fr,„
w = «i
White to play and mate in four moves.
No. 178.
By W. Lewis.
2,
Jill
"UJ^V
HI I
mm,.
HHP" ^^^ ^
h^—jwk
' 4^ WW/////// V//TX////
Wliite to play and mate in five moves.
12
No. 179,
By W. Lewis.
WZMA
tH m mm ; m , <||> p
mm. Bit!
Ill 4
m ■§ §§ §3^
% Mm c
m~B-wm
mi n»
i§
m£A wmk mmk ^ 4^A
m HiB
P 1111 HH ^ IIH
White to play and mate in eight moves.
No. 180.
By W. Lewis.
■tW&l
HHf HH
-mm, 'rim-<
f
WHITE.
White to play and mate in five moves.
No. 192.
By T. Smith.
wm
fa
"W
1
i
• TaTL
M
White to play and mate in four moves.
No. 193.
By T. Smith.
i§§§§ g§§§| p
!■ *m IIP ' P
"■L, mm.
■■■'■■'■■■■■
fm
\
iaPW :
^W/jmT' V m
III
■p*"-
w % r ,Jm ill
White to play and mate in four moves.
No. 194.
By T. Smith-.
/ft* Sib
m.
w,***Wz%m..
\ S"
^.,'~J™"' wy////////. r
mm & m
§lil
I Pis pi
HP '^B 3 W
////////////. 4?/////M
WHIIE.
Wliite to play and mate in three moves.
13
NTo. 195.
By T. Smith.
ty////////?.
rri&^^////f''Z / '''WA :■
HP
mm,
m9, ft
m wmtfm^
i
!i
% * Mill ^J
p «1 MM H ';
White to play and mate in five moves.
No. 196.
By T. Smith.
BLACK.
ML-
A.
w mm,
«L~. #>
mmm
m m
III
?//////%.
ill
Pf
'/7//SSJ/SAH
WLJbL
%^/^ m
r wwm mm
m mm HI
White to play and mate in four moves.
No. 197.
By J. B. of Beidport.
'WW B I
W0""i""W,
mm —mama
w mm
M7M WES!?/}}?.
mar
■UFL
^„„„,„jiiiL„„ Su
1 If
IIP
MM
White to play and mate in three moves;
No. 198.
By J. B. op Brxdport.
"■i! 1 " 1 ■
WW/
m mm.
M mm P^l W-%
W, Wm. " mm,
'mm. w.
m p
M w^'
mm
WHIIE.
White to play and mate in three moves.
No. 199.
By J. B. of Bridport.
M
fci
VlH-flJ"
m
m & wM „,.„„mm,
mm w
^ v £!L$™A,,,,,,,J®^ m
i m
"WO0""*' 'HIP
if w ij§ij^j
4w< mm
nm §1
Ict mam,,,,,
mm. mm. m
b « n
White to play and mate in three moves.
No. 200.
By J. B. of Bridport.
fe
Br
y - , mm, mm, m
HI
mm ',■ M4
Wmk tllll *
^ ^« . ..
I mM, HH llil
WHITE.
White to play and mate in three moves.
No. 201.
By J. B. of Bridport.
'mm. nn ^ «
mtfrnSJ^
im.
km
]■
i ! ^
«1 * SUi"^*
i
jjgywLjMj^
ISI
m Wffl8^^W ^^
White to play and mate in three moves.
No. 202.
By J. B. of Bridport.
BLACK.
■p ■ s ■
H iKi
^^"^ ^tlflilP ^i$HH
WHITE.
White to play and mate in three moves.
No. 203.
By J. B. of Bridpokt.
■ -««f«
■„-4-P.
P
It ;
H %
WHITE.
White to play and mate in three moves.
No. 204.
By J. B. or Bbidport.
m ■
i ml
i
H 111
w/////;yZ>.,
m Mb
mwA
wk mm mm
Whito to play and mate in three moves.
No. S05.
By J. B. of Bridport.
M ij H
4™^—™= «
Pf2f mssm
m m
i msm
■Usf
m ■
■mmiw-
WHITE.
White to play and mate in three moves.
No. 206.
By J. B. of Bridport.
1'
WW*/, w,.
m w J Jm m
itiiiC
mw, WW? ^ i
i
r WW ^ iww
m
// 'yMmw-J^ /////// ^'-
iwB,§.
WHITE.
White to play and mate in three moves.
No. 207.
By J. B. of Bridport.
mvsmx, »^- V////S7///,
M
m wmT m m
i si
^ „«t
mm
Hi
my, w////////f.
HI
WM €l§J
m mi
»\1
aL. fat
W/,
"White to play and mate in four moves.
No. 208.
By J. B. of Bridport.
BLACK.
P; #J^ WM% .. :
m
w v/ mm,,.
W£':%
H Hi
•,—■■1
^J^JiiilL
W
tli ii ^
WHITE.
White to play and mate in five moves.
No. 209.
By Silas Angas.
BLACK.
y ///>. vu
W, y//////////,
mam — - «^
W"~
i r 'i
'^'WW^^''W7///"~
WHITE.
White to play and mate in three moves.
No. 810.
By Silas Angas.
i iH§
MP *
PS
IL 111!
w 88
10
WHITE.
White to play and mate in three moves.
14
No. an.
By Silas Angas.
■i i
r iy w iM
'■P &^mf7i IIP wM
WHITE.
White to play and mate in three moves.
No. S12.
By Silas A.ngas.
P
i ib.
Hi
w, ^e
E
w= r
■ 4™"^
%^f^ M 'r^.
White to play and mate in four moves.
No. S13.
By Silas Angas.
m mm,.
pif mm x
mm x mm
m...^.MmM. 2^1
i„ ei^M «
[ mwMMiz^Mi
Jr**
»f±*—
WHITE.
White to play and mate in three moves.
No. 814.
By Silas Angas.
mm tBi mm
M
\S\,
'Hi
mm
w mm
ram
mi
WHITE.
White to play and mate in four moves.
No. 315.
By F. H. Deacon.
»\1
%t
III Wl
m 3~m
i im
1 i ill
ill xl „ jhSL. *: . ...
mm ^ mni g *
mm, HBHL
; ISP
i3|
.;.;; d«
White to play and mate in four moves.
No. 216.
By F. H. Deacon.
:%is W////////X
y/s, wWa ^ , III
wk,.. m
W, #1111
mm.
WHITE.
White to play and mate in five moves.
No. 818.
By F. H. Deacon.
„ """""■„ / . ///// ,,y // " / ^ / -r // , / - / ., / -, / """""'■ /"zztT. mvmmv.
M»
4m
fMB Wm. W%M„ ,^JH,
vytXky/j %r£\4 ySsSmxi wrJtxi
^11^ '///.LtI?, W^,'. '///.L^i
5 ^^«»«««^^ :: ^^j««««^^ // ^^^>«««»^^^^^
wmr* w
pi
OV/////////.
1
■
White to play and mate in five moves.
No. S19.
By F. H. Deacon.
mm
^■AII m kmm
11
i§ IHIJ
% ■■
White to play and mate in five moves.
No. 220.
By F. H. Deacon.
ML, IB! SB
p?^ ^^^ «m
Pff P
ill ■ 1BV
i. ™
iii
iJLi
i®
■/////////// /y/ 7
%_^_wm mm.
White to play and mate in seven moves.
No. 221.
By R. B. Wormald.
BLACK.
■
til < iij.il
Wv
tip
.Mr//////'/. W/////M&,,
*w
White to play and mate in three moves.
No. 222.
By R. B. Wormald.
k §§§ k
'WW
Mi
-
H mm
.■//, '///,///////,
WHITE.
White to play and mate in three moves.
No. 233.
By R. B. Wormald.
BLACK.
m y
'W^% HIP f$M 1HP
H
s jmm
W//////M 1 ,,,
mm
1 mm-
■W*" 2
WHITE.
White to play and mate in three moves.
No. 324.
By R. B. Wormald.
i m Biwiq
m III
WHITE.
White to play and mate in three moves.
No. 225.
By K. B. Wormald.
H I§1
WjgtM
m
w
w"V
WHITE.
White to play and mate in three moves.
No. 826.
By B. B. Wormald.
mm
i §111
* --Tip ■
m
... Wt,
-. •..••;
Ill 1
an
■WHITE.
White to play and mate in. two moves.
No. 228.
By R. B. Wormald.
BLACK.
.. Ill w fc
^. iSa, ^Sfei <^^2
'mm
W mzm,
HH H
;t> 1ra m/
fflf :f #l i Pi
■I EljK
■mwJ^^^Jt^m
m i
mi HII II-
^ fc«^ .i§
P «l
White to play and mate in three moves.
No. 229.
By R. B. Wobmald.
■ m
®- J*
v'/mv/x: ir^rrr', r.
1 HJ MjmAJm^J^
White to play and mate in four moves.
No. 237.
By E. A. M. M.
■PtfM
mm M mWk
4 wM,=-„mm,
jBL^ljhnhl
\%
mm
; fi 111 ■ i
W%M Wwk
White to play and mate in five moves.
No. 238.
By E. A. M. M.
im
-mm. m
wm
mmi 8 !
^ a vB?A c
A Si lite ww,
, fm wm% W%
I
wSTETM —— „w//////m
ii#Hsl!
. PfWlO 8 pi
wmd. vrnf '
II
WWM. MM W
fTj ^P IS
A I
to
■ iptf FBI
White to play and mate in six moves.
No. 244.
By E. A. M. M.
ty/' 'm-'-S'-
<4 %%
WA.Tr'wmi
M"J wmU 'mm
m ww^mm m
WKK^^mm mm y ~~
WHITE.
White to play and mate in six moves.
No. 245. The Koh-i-noor.
By E. A. M. M.
flat
''■mm
S#///S/SSA ""'" V/////S/A
Wk
W/Z&>//. %WAW///a 'Py^'ffi WZWffi
^jftwm Si^il
11
\tbM 111
WW WW V
. ... j
WHITE.
White to play and mate in six moves.
No. 246.
By E. A. M. M.
4M
Wa)
'%j9%fr 't^&as?'' z*w5g%
m &7//J, ,„„,„„ %///////£.,„
'WW, r WW
,//s////7777/,„
mm
mm JmWL
WHITE.
White to play and mate in three moves.
3fo. 247. Mhow.
By E. A. M. M.
m,
ifcf mm A Si
mm
Mm
m Hi
W//////ZU,,
m
Wm
WHITE.
White to play and mate in four moves.
No. 248.
By E. A. M. M.
iW™
"W if w
mM>, m%M wm.
w> mzm. fen wwm
mm. a 'mm, _r_ m
",
_-i | ^?« l- « K m
White to play and mate in five muves.
No. 249.
By F. Healey.
WM v ; "'"'%
mm /A m
im I
-vFT'w,
wSk mm.
mm
w
White to play and mate in five moves.
No. 250.
By F. Healey. t^jy /
BLACK. '
■ PI
% w ;
^^ ^p 1M
ill fli
a
i *
i i
^ i
my. ,„„„„„^M^^
White to play and mate in five moves.
No. 251.
By F. Healey.
A w,
M wmF
mm, mMM,...^!, ,,.■?//,■/,/////. 85^^.
a
H
m I#I ^P ^ P
ra III ■MfcL
mm, mm,
^u-
iO
White to play and mate in three moves.
No. 252.
By P. Healey.
*.mm, \
m m Ill %
I 3 P P
m ^P * .1
8
WHITE.
White to play and mate in four moves.
No. S53.
By F. Healey.
w
m -mT m m
KXOMMi „ . S^H^g.,
^ jSII ^^
^„.:r,,,.^%^
b
White to play and mate in four moves.
No. 254.
By F. Healey.
AiL
wm^mw.
WM-mw
wd^'m
wmrTm
m m^m warn
mm mm
w — mm//, „„„„ wwM.„„ r '&%&>}.„.„
wsm. . WM. m§ II
_„0M j^^
mm„,,,„,,,,fa ;.
'■i H
^IP p
II »lll«j^yyj
39 fy\1
mm.
I
White to play and mate in four moves.
No. 255.
By F. Healey.
w> ww3...
W. m
mm.
w. mm _ mm , mm,
i
1 * mm
H, #iBSi
ilBl
m.
m m m^mi m
■f ■ H iiii 1 ™
# Hsif p3"| iUPP
3% ggggB^ Vy/////Mt, twwZy,
WHITE.
White to play and mate in four moves.
No. 256.
By F. Healey.
"I I ■ w
pi 4^ pi
III A
i#Jls illl r ,
^
PPf ^
'WM
WW-
White to play and mate in three moves.
No. 257.
By F. Healey.
H PP
"WB, "^
iHi
■t
- - isi B
■ ■ ra « M w
1
i
■I ■
b Isl am,,,,
White to play and mate in four moves.
No. 258.
By F. Healey.
-mm wm
iHf ill
'W% ■
Mm y/ w/////////., aw.
s & ?
,-^u
I
^$
I « 111
wm
White to play and mate in two moves.
17
No. 269.
By P. Healey.
#1
m %
%-PVi
§1 'WM
wm tmfr,„wm,,
jjpijji i
*■ iH ft
a£jBS»
BjfcS
WHITE.
White to play and mate in four moves.
Wo. 260.
By F. Healey.
HI
n
II 1
.WW//.
VA
pit
mm m
, %mm. ^^^
B
W, gffa
WHITE.
White to play and mate in three moves.
No. 261.
By F. Healey.
Hi
WPWi
&\i
s' e =».«
§0
/ '-v m - //////// -
Warn. * %wm m
'/m fy/////;y/?.
mm
ran
mm %
m
(*) i
ri, b
White to play and mate in three moves.
No. 262.
By F. Healey.
'wm w
A ftM m
^™ — —^
"p. ■■«
H Wm.
White to play and mate in three moves.
No. 263.
By F. Healey.
BLACK.
^rnJ^m
M **. WW, VMM
up
i
i „„„^*..;
"'■> ''//////////, ■ yy/////y///^^^
m ww, y/
to 'ww, v ww.
WHITE.
White to play and mate in four moves.
No. 264.
By F. Healey.
'■SWiffllt. '////,
w
m v ww,
m
m in
Wm. WM. WW/, WW/,
B, , m o **-* WW, -s- 1
P ,t, ^^ & MM
W/mw/,J±%mi
, mm ^j^jb^jB^^i
ifll 111 gl^
White to play and mate in four moves.
No. 265.
By W. Grimshaw.
"WB I
||§§| As
^ m
?'■••■;
m mm it
White to play and mate in three moves.
No. 266.
By W. Grimshaw.
m« ^
••£--•/
■ ifi
fl »»
v^m.
m r/ m mm /y/
n illy
Pi
*%
»/////////. ^^ mMM ■»/////////, ggassfei
WHITE.
White to play and mate in four moves.
No. 267.
By W. Grimshaw.
- HP Iill„
■ f— ™ ap J" 111 |"«
Hi 4W™^
?i
A mm //////A w?"
% w//M,
i
mi i;m * in
■ri ■
MB. ^ SI ##Jf H
^■^ ^SS>% ^^^ iT
^ '^j JBJ JHHf HHH
Ml «
White to play and mate in five moves.
No. 368.
By W. Grimshaw.
\i\k\ ;•#
WM A &
MM w^Sziz.
■■'"■"A
mis.
W 111 : -
m
i,o ill
JHL JH
pi tm
White to play and mate in four moves.
No. 369.
By W. Grimshaw.
™H*_«
W m ,,„....0i00. %B^
i
9
LU
f'wmfw.
* mm, ill
i fai iB
ki m i
r^ i mm
■m0 §p ip
WHITE. .
White to play and mate in three moves.
No. 870.
By W. Grimshaw.
■ i
IK *■ W
\m
V
,„ ft/////////. 't%MWV, WMM/:
in w
Mm,.,, %?,. ....
ill
I
Ml PPP P¥i P
B « p f
White to play and mate in four moves.
No. 271.
By W. Gbimshaw.
mm %
■I Bi
y/mz ^'wzzzzzzz^^'m-
w mm, wm^/ww,
mm, a mm, ^ '-mm, * y
WS/S////Y/.
Ifl
i fan
VV'
pi pi H
^ W//////A Z///////Z. SSSfe^
"White to play and mate in three moves.
No. 272.
By W. GrKIMSHAW.
'W'ZZZ^ "-"-f W&$%!>
W///////&
fa is
i
i 1553
pj*i
i
.,,,///A '/////S////A
pi x k
IB
I
m iHl
White to play aad mate in three moves.
No. 273.
By W. Geimshaw.
%Wa mm.
m a m
if (S WBSr
mm.
mem
*y//////AY/. W/TT; ;;///,
ppsp
ffpl m
mm iHI mml
wm. wmf'^'''Wm
//ssjs,/A V7////Z'.y/. '/////////A
. W4W,
W/, %Mm
■A M iBJBi
Smi^^wA
HI Wkwmm mm,
■ III
i
¥ M PS
*H €sl„ Jill
™„„__,„,w//////fa 'oiTT'..,
m m i
Hi
W'mwZ- ■ww/ ////// ^wfmr^^w&
White to play and mate in four moves.
No. 282.
By H. J. C Andrews.
m. 'mm
W///////A ty////////i »» , ■X^t^/':-
y.&£% warn. 'VMM; ^ WM
I
PI
I
V/////S///S> ,
'■mm w.
Hf
^1 4%zfA
White to play and mate in four moves.
No. 283.
By H. J. C. Andrews.
J""*
w,
£» ^B,...§....^B. IS
i ■
m
■I
fej
i'*H HHI
■ $1 ■ § IP
||f§t #2_ 1| if™ ,1, ^»|
White to play and mate in six moves.
Wo. 284.
By H. J. G. Andrews.
±s "■%&% y MW% WZM
m ?Mi mm Wm^m
y/////////, ,
v/yy/7// „y/sts////%>.
m wm mm''"'
fmwm
m til
ii Jt «L ™l
, 1 ^mm fe#i ira H
fe*l ™ 4MvA, Wmk, mmk
Wm mm ?m
VmTZZi. WW/0, <£t%SZ&.
White to play and mate in seven moves.
No. 285.
By H. J. C. Andkews.
■
w m "
m w^r-
'■ZWM
*m
m „H§ mi f#J
v mm.
PI i
mAJmW.WB_. B
ISif «§! Jill
P^M Ji^AWf& y/zy/J^^', ///////// ^^
White to play and to compel Black to mate
in eight moves.
No. 286.
By H. J. C. Andrews.
//-/A
¥M vm WM„,,'%m,
v/m iJiii „,,,...ii§ii mil
i
jy ^ ■(- §■ , jiii
'V/1'.L'V'/, v'/l'^'/J'/, ■///''''>-■
W,
'"A,
■'■
yam
A//////Aii
o Yflks''.
WyW, a WZW/,
mim
m a WB,
■/, r— •? w//A//i
White to play and mate in three moves.
No. 287.
By H. J. C. Andrews.
m mm.
mm
^ W//////A..
III 181
^ww-
mm mmJ am m
"Wliite to play and mate in three moves.
No. 288.
By H. J. C. Andrews.
II.
mm
1 fHH
up ■?»"— "™
ill
^M W//////Z&
w$% ^ Hi
iMI
'fa'
i 181
WHITE.
White to play and mate in four moves.
No. 289.
By H. J. C. Andrews.
BLACK. .
%®4 mm m
tmmf™mmJp*™»*mJ
'^^
JSm. 8 KM.
ill up a m
WM mm
%^ ciL.
wM/7/,
WMt;
Ml fHf
WHITE.
White to play and mate in four moves.
No. 890.
By H. J. C. Andrews.
^B. isky
Wm k wlm^M^ k I
^3i * ^^^ ms Warn. * *
. %mi*WM. w Mm,
iJLil _■_ BL
H Us?
White to play and mate in three moves.
19
No. 291.
By H. J. C. Andrews.
^ISii W//M/-, ^ ^^ 8
1
, w/w/-,
//////ft.
mm
i ■ mJkm m
ill
^^; wp
ill
1 fill
p
n . i§n§ ^ ^^^^^j^^'
Wlri*e to play and mate in four moves.
No. 292.
By H. J. C. Andrews.
H
M ■
ill I
WW;
■-J"
M*
1 ^ si
> ill ffif
© H M Wm\ A H#r
iJJfl^eJ
ill IP
■■ / //7////////. '//,^* / /.
M, iS
White to play and mate in five moves.
No. 293.
By H. J. C. Andrews.
■"■■
IP
mm...
m
<&„„„ i //////////. v/Mmm
m&M
^sf'J X ■ H : ^X ^H' #-^
jfl! ffl
■S
m
H vwm,
White to play and mate in four moves.
No. 294.
By H. J. C. Andrews.
ill
:k^ y/!55!!?M.
™%
mm -*- wm_, mm,
mm ^wJ^'mwF m 'mm.
m^m m i
%8„ 'sfTffsTTXZ I/////////:., W//////M,,
#1
m S^^'m^ m w""'- "
111
H
kJ±Jm
WHITE.
White to play and mate in four moves
No. 295.
By J. C. Roll.
^
'WW)^x,Mw3.
mm.
wm
y////////^^^y/^////^
WW, WW. v££ WW. «h_ H
_- m^bk w jm^ w jmm
m*
m ww,
M, mW,
White to play and mate in four moves.
No. 296.
By. J. A Coneoy.
i
W
m
ww, ww, mm ww.
~M „,.,,,..., Wm,
S3,
81
mm mmm jB8m
»j^.i
Jill 4mw,^..ww,, // ill
WHITE.
White to play and mate in four moves.
No. 297.
By J. G-. Campbell.
fawm w
■m. x m
:;-%—• m muf l ' m ^gg^
%
f|p ## ; HE (f
■mm. i
tM
m
f "if" l
«| ill.
White to play and mate in three moves.
No. 298.
By J. G\ Campbell.
MP M mm/ hm ^n
WW>.
1 M
-*■
M Pi i pi pi
%7&7?7ti, V////S/M..,. ~ ,,■%/////////.„ -»//////,//..
' /M Wt
^Ifil M
White to play and mate in three moves.
No. 299.
By J. G. Campbell.
1
i ■ n
^ mma. iffm^/..
wSffity,.
s ^ im Jmm^ Mm.
\\ i
lAlil
mAm(M^M
m w/w mm '^^ m ^'
White to play and mate in five moves.
No. 300.
By J. GL Campbell.
H§J
ma mm. ©
a
mm
'W&& 'WM WM
^™ - iii
mm, WJm,
4M
J**J£J*
. ^SsfM W777777A Mv/awj '■Ztt^FtttIl
WHITE.
White to play and mate in five moves.
No. 301.
By J. G. Campbell.
PP| pip pll
lih
Mom
m
t-jmzsm^jm
m^ mm *
A ;
X '-'.JSm
"HI""'
m
WWM
WHITE.
White to play and mate in three moves.
No. 302.
By J. G. Campeell.
BLACK.
HP ^ r^i
m% a if
km h m
V/////////>.
'mm wm
fll ill 11
W&& x 1111 tit SiP H '
m Aim
iff 9
m i
■21' '■
a
iKJ
& wmr^m
White to play and. mate in five moves.
No. 303.
By J. G. Campbell.
mfimm '/StZtSiz
w,„„„ /M mm
H
ill
HP
mi n
"«t L.
WHITE.
White to play and mate in three moves.
No. 304.
By J. Gr. Campbell.
mm I
1
? a
H, ISJ
b.
■«v~
illl. wMk„. i£m " bSJ
%„„. jsam
White to play and mate in four moves.
No. 305.
By Rev. C. E. Ranken.
m
wm m
JL III ■ mt
I ill
I ill iSi
HHf^. ^ ^^■mw/^^^w//^/J^ s ^'w
%
^^ ^*s»
WHITE.
White to play and mate in four moves.
No. 306.
By H. E. Kidson.
. V/////S//M,,.
1
wm «
.iH li III
*9p i :
■ ■ ■
wm wm
w//////',
IIP
m
WHIIB.
White to play and mate in three moves.
20
No. 307.
By H. E, Kidson.
nm
M mmTTm
'///,, '////////yy/.
m
WiM. W.
i pi & p| IB
>
* m ■■-
H
■flfisffl,
White to play and mate in three moves.
No. 316.
By H. Turton.
Ussf
Jfil
III * #» . Ki gfeafa
H.
v/s///ss/yh
j^^fw^?> ■%/%%% JvAtCCwvs
H. ill
H III
White to play and mate in four mores.
No. 317.
By H. Turton.
J^mm/^ggJ^i^™
mm w.
ft , w///////;
WM WMk WMM y iW?A
1 fli
w.
wk PI
KS«v^%a timy/M?.
m
ii
si m
imi wm vm ww, §;.:./
WHISTE.
White to play and mate in five moves.
No. 318.
By H. Tueton.
..4«" ■"
^•? ^S
mm
m
mm, mm, mm,
w f FBI
"if b
mm,, .
mm, «i m
ill 111
1*
1L till
White to play and mate in five moves.
No. 319.
By H. Tubton.
WW. mm>A
WW4
im fm
#1 WM.fBA^P?^
A
if W* 4
wm
mm WsB
m
WHITE.
White to play and mate in six moves.
No. 320.
By H. Tubton.
J HP IP
m l», ^—i Mi
i
WW "WW P^
ill mm ffl i
w////// ,,imm 'w-v/// '' m ^\^//// m//m ''y /
»%*p^ '"^w/maf^^vsm
H Hi
White to play and mate in four moves.
No. 321.
By H. Turton.
Jft,
Mb m. V///A
m
nm mm m
r K ' « — IT " ;
W//////Z4, ■xy/////zZ>, ^HH
W&M §
H
„1W1 i^i ^^ JB
pi n "r
B mi
« ISS IHf
WHITE.
White to play and mate in three move3.
No. 332.
By H. Turton.
WW4
IP
* i^n
m M*m.,, #HI
w
If
ill
^wsm r , n
1
Pugf
H %^si
a rfl
WHITE.
White to play and mate in three moves.
21
No. 323.
By C. W. of Sunbury.
■ r «■
W"if
if -<
J MJPtJBL
«1
White to play and mate in three moves.
No. 324.
By C. W. of Sunbury.
i§ iai
w £11 w
m ^ m
J"%
^ (22iiSK2
m m
w
i„. h
White to play and mate in four moves.
No. 325.
By C. W. of Sunbury.
■ /Wi
WZM
■P IIP
%%23 tf777A^7*7.
<&
mm
Mm.
mm %m%
White to play and mate in four moves.
No. 326.
By C. W. of Sunbury.
m mm
mm
mm
§|||p
wk t™' i
mm
wm.
H^HL 'mm,,.._.,..M^m.
V/////////.
il * J
B
if Wwm,,.
i.
■r
WHITE.
White to play and mate in four moves.
No. 327.
By C. W. of Sunbury.
im mm...
HI mm&
W"
m y mm.
^~- : P%^ ^
■ k HP ■ i
m^M
m JBWiZ£±jBBM
m m
mm
111
2% Vsss//ss/y/.
White to play and mate in three moves.
No. 328.
By C. W. of Sunbury.
III HI
M . ^_^^«_—
''7777/7777Z V/////?/?/*
White to play and mate in two moves.
No. 335.
By I. 0. Howard Taylor.
HP
ill ill ifa " i
mm,
*mv ■>, w t wm
m
I
.mm «= SSs i^f
JSLJam^^Sm^. ^.^
■ ^^^
HI
White to play and mate in two mores.
No. 336.
By I. 0. Howard Taylor.
/
Wk-.M
l5W# f^M §ffl§ I
j ny im
M m m
m
■ * a
White to play and mate in two moves.
No. 337.
By J. W. Abbott.
W.
\J*JHk iSJ.
ww' m
imm ^^ ... Ml e -
WWA
Wwm
^^^PiP--^
HI *t\ Hill 1
W
wmy-iwm mm .■■
i»
;'-T';l »M pw WM
m m i ■
White to play and mate in three moves.
No. 338.
By J. W. Abbott.
i MM
m m m
llBlli Hi
WM
■V -1
mil H
i
wB, W.
wm
zmi ~ — ~ %•/.'////£ ~ — " vxfiwA,
% A Si
V-E-- ^P 1
^p HP 2
J lib "^- j i
White to play and mate in two moves.
22
No. 339.
By J. W. Abbott.
39 Wm PP IP
ill «S> WM.,,,,,,,,'WM,,„„WM.
mm, wm i
■ (Hi
1 1
m,.m.wM
m m «^--
*^^
V/ '
%m^k
mm mm, %mkj*Jmm.
i
I ft »
®i * HI
1 I
1
%8 '///////////,
4M y/ JB,. m
Wliite to play and mate in three moves.
No. 340.
By J. W. Abbott.
'wm m
m ww? m W -
Wm
HI
HP, & w
mm
WM Wi§ PPP*
Hi Hi A
Wmk WMk WWk
fSi I
isll
^m^-jak.
White to play and mate in three moves.
No. 341.
By J. W. Abbott.
musJm^Jm. m
wm-\u/^,
1 Hi
sb i wmi
\ mm X wMlk
w/////m W///////A
Bl
A
:a
ili . Si . (■ iImI
^M
w rwKT m
■-■■11
■■"■/'///, u %?%J:
k 111 *
rs^
H llli
%fei?J
White to play and mate in four moves.
No. 342.
By J. W. Abbott.
'wm ms Ws
■ %
Mil
i i lllli k'A lllll
ffif W:M
''/////////A WW"'- '^^
Ml ^^„
..im iii
H IS
til
% y JW//M,
mm,.
wmm mm.
White to play and mate in three moves.
No. 343.
By J. W. Abbott.
HI n
..iHH WWS/, Wmm
WM% Wm WiM
(m mm m
W»M %5fegKrf UMM .
V/////////.
"White to play and mate in four moves.
No. 345.
By James Pierce.
MP
ML
mm m
m, ////////M ^m. Ill
iiii — .
H 1
I . ag«
— B r r - "■
W,
White to play and mate in three moves.
No. 346.
By J. Pierce.
H mm... mW,
- wr~iV
WW '^ Wf
HI Iff ' ™*
.wwm M^%
#18 A m
•i. • ^as. *
III
i ijm.
HP
White to play and mate in three moves.
No. 347.
By J. Pierce.
an m
m
^ '///.
i
a ^ «« * m
Yy
11 a 1
^
WHITE.
White to play and mate in four move3.
No. 348.
By J. Pierce.
— IP ll
is
IIP
ill _ilil
3 mm wm
il ^ w/M,
"White to play and mate in four moves.
No. 349.
By J. Pierce.
::^HM
mm ' mm
ism* m
mm *mm
mm
a wm wm a
a kVi
wm wHm
"White to play and mate in five moves.
No. 350.
By J. Pierce.
"pTgTl 11
^ M 'mm M wr
WM 'WB, W1M
■ ■'////%!', s£f5zH ?
*W J
White to play and mate in three moves.
No. 351.
By J. Pierce.
hi mm
mm^jm .
■■Tfr-i
I H I
■r •
» til iH
!l«i
mm fAl '*w>
wmfim MWZM y/////y//Z; '■/////////■
w%m. o ^^^ 111P ^^
.....1 ^ iHI llil BHi
1 gp mm «* «r-
White to play and mate in two moves.
No. 36jf. Z-
By J. Pierce.
?%
I§PeJs3§§P
y/M&..,^/mA MH..
If
■■ ■ ■
'VAvtttt,/, tfiSBSSSit
iiti
it
m *SJ m wtJ® m w''" M
Eft ■■
'mm
*§ a mi up
i:^«IjJh Hi
lllf ■
I lUl HI
White to play and mate in three moves.
This Problem has not been published before.
No. 353.
By Rev. W. Wayte.
IIP HIP IIP IP
warn, , "W
WSB WM.
" WW' WM'
flbi mm. mm.,,.,,, \
w, J....JSI
^HH w//////.... r/
zzz/zy/Zb.
IIS hp ^sn
lay «
■/■y.: %:.,z -%y//zy//): wfiffim zZXY/////.
wm
* * m i ill III
i,-
White to play and mate in three moves.
No. 354.
By Bev. W. Wayte.
WM
I' C "Wi
Zz\wM L — ' Z7/M/M
* W"
V">,////,%
^ppi A ^p X
% *
I
,'yyy/////i% .,
:.,.... J
I J J
V
lill i§il l^P «*» ^^
^ IP IB HP
i «ii g #11
White to play and mate in four move3.
23
No. 355.
By W. Timbrell Pierce.
iwffti ■!
' 111 mm
W
III ■ Ik
^L
1
,
i JLfHi -
"H ■-^H fc
M
"w^aHr
^^
..MAmLili
«i ^^
W////M" ''WMk %m
White to play and mate in three moves.
No. 356.
By W. T. Pierce.
illlJ ilil
ill
MimmP^waJ^m
m " ^w^
.*. i
,-/,.;. .v, A
Wm
i a.
WHITE.
White to play and mate in three moves.
No. 357.
By W. T. Pierce.
m m m s
?1 Iil§
f: ■lg|
«
H P ■ ■
'<&%
'mm §Pi| «
^^^. IfeM fe^l _ Jite
u
™^- mm pM wM x
mm,
lb
WHITE.
Wliite to play and mate in three moves.
No. 358.
By W. T. Pierce.
MA
%S^ -J**- #~^ L — ' W%m 1mm.
mmWiM
H HP ■
^ W V# rl^
WHITE.
White to play and mate in five moves.
No. 359.
By W. T. Pierce.
%-P.A ■■
fl .M ,H JML m
m w///////f^ m m
W I
in
111 ■ B
%-/////////.
'd/OOzm.
mm
mm,
m
i ill
White to play and mate in two moves.
No. 360.
By W. T. Pierce.
mi m
Br
i 4
»* w™^
A w "--
^ ===i t/////////k y///////// W////MM^^^Tfa/#ASffl f V/W7??///
m wt/ m -
,•&•/////.
W '//WW,
Www? f~^ W"M>
WHITE.
White to play and mate in two moves.
No. 361.
By W. T. Pierce.
§WI
I
W£M
-,
WW W
i
m
111 10
j» ill jM iSl
m*tt& i^*** wmtmfa ft£vS5xAa
White to play and draw.
No. 362.
By John Watkinson.
m
, 4>wA ....warn..
iff
?W.
i H^jdLjHl
White (Mr. Watkinson) having to play, won the
game.
No. 363.
By Rev. A. B. Skipwokth.
mm m , w*m fm
i
I * HI
WM -
■ : S i<^fli III
*Ji ^ IM w mk
*m mm m
4. 4%zm Li
us
W^^=^=' '/>//////%i
wm m
mm
LSI 11.., H
pip
mm
wm wm.
^m^^Y-- ■"■■■"■"■
til
J§p f&f ¥
White to play and mate in three moves.
No. 370.
By Eev. G. Mc. Arthur.
'MM. WW4. W,
W£%?,.
mi
Wk mm WMk ^H
■ Hi
'mm. mm. -mam. wem
i
W^'-i" 1 - s
H Mill "5^-» ^^
Pj
P
H Hi
»
Hl^ wm%,
mm.
m
White to play and mate in four moves.
24
No. 371.
By Rev. G. Mc. Arthur.
m 4/ wi m
w §m
w mm »8^r
. - Ill fill
p/^A Jt^Szi. W//////&, .„&m5mi
i mm
i m
White to play and mate in four moves.
No. 372.
By W. S. Pavitt.
v m> m
■ "111
JP IIP
"ail
Jem. Wb
ww iHHf
fi__rai__il§il
WHITE.
White to play and mate in three moves.
No. 373.
By W. S. Pavitt.
i ■ I ill
m^jmmmjtm^
■ JE.il I
Hi 01
n* 4Km r ^ mm
WM m m Jm\ mrn^m
n mi
WHITE.
White to play and mate in three moves.
No. 374.
By W. S. Pavitt.
MJ
m as
jj 11 ■&!!
^ g;**...
White to play and mate in four move3.
No. 375.
By W. S. Pavitt.
'|jp k Pil^^P
i 111 wii «y
131 i4t,u
eH .. mm mm
111
§111 1111
1 S
White to play and mate in three moves.
No. 376.
By W. S. Pavitt.
A m J^< w -.jMm.,M
w, mm,.
1
i •■
i IBI H.
^^ SBEbb.
11 IB ^^
WHITE.
White to play and mate in three moves.
No. 377.
By A. Kempe.
w//. w/;;y/'//.
H i ■ ■ ■
'w//.'//A
M wmT
«i
w,
I i «
wmamt
; %km
B ■ Bi-I
-mm -ww
!R III ^ dfll Mi
White to play and mate in three moves.
No 378.
By A. Kempe.
BLACK.
WM MM/A . =L^llii..
wm
kwm Wmk
. ^^.— MM— ,0^1, iii
pip mp
S b n fli i
IHt tsi
i
^ A
PHI tsa ill
m
Ji !M.
WHITE.
White to play and mate in five moves.
No. 379.
By A. Kempe.
J I8J f «J^
I i
WW4
«T\f
^ „,%?/////,//>.
Wm rMe mm mr
'wv/WP X fillip
■ Sir
1 PI ■
i
tM
WHITE.
White to play and draw.
No. 380.
By S. H, Thomas.
Ed f»cS/i
K W//////A '^//////A. ^HtH
1 fi Wt
Mi
». ^K„
^ n
1» A
El
'T m ™— M — J
"White to play and mate in four moves.
No. 381.
By S. H. Thomas.
'm ft i
\\i •■•
m m
m m ',
w. ' 'mm- "
m ^^ m 'mm mm 'mm^^ m '
m
Up
f4£
"White to play and mate in three moves.
No. 382.
By S. H. Thomas.
■
waum ftei "wm -^ %s?i
%MW zmmz wm&, mm:
i
tri,
111
m, %//////A
0S%
w mW
m_m
White to play and compel Black to mate him
in five moves.
No. 383.
By S. H. Thomas.
mm pn
m
m. mm.
I
-*|HI
wk ■ Wk » v *
8mh« -mm Z M 'w
I
Iflff ^W • fiHI I
im ail * ^p
-■Wwi warn us wm
IIP i IIP iff P
: * i^ io ife
nn
V///s////JZ>
mm, wm. a s
in © ^
WHITE.
While to play and mate in three moves.
No. 384.
By S. H. Thomas.
IIP
■
jfB.
HH fill ,1111
Ml
a
mm:
rf
i mm
mm mM fjp
"White to play and compel Black to mate Mm
in eight mores.
No. 385.
By A. Townsend.
w
H Wmm
I I
■Pi
WW
n§ wml , mm
mm *
m mi
i
m,B...wM L;,..J
i
"m
H wm.
I
m
w
w mm
H I^J
White to play and mate in three moves.
No. 386.
By A. Townsend.
§M, mm. m
S%^^% %%%%%%>. vwMM
9m.
Lifi
^P ^^ ^^ ^P
"White to play and mate in three moves.
Conditions — That the White King is to be in check, bat able to
escape by any of the three methods, viz. interpasing, capturing tha
checking piece, or moving out of check.
25
No. 387.
By A. Townsend.
mm warn
Ws fc
« i i m m
■f ■ ■ ■
.as, I * ^Si
I a ||||§
*^ Jul wm%>
White to play and mate in four moves.
No. 388.
By A.. Townsend.
«2
■ Mi
• ess-, WWM ^a W°W
//////////A
Jiil if IB
a , , ist
1: P
Bl
...mm.
mm, hi
^/mgummwm
mm
WB% W^M ?mm
White to play and compel Black to mate him in
six moves.
No. 389. /Li/u*(- crvtcMtj
By A. Townsend.
warn nn 'Wm\
w. WMM
*■ HWH
J ■ ■
pi *
i i
mm *;wm..
Wb,
mm, wm m
m mJ m
White to play and mate in fourteen moves.
No. 390.
By A. Townsend.
X
&
P IP !
wM,„ ML fc;„
m ww MM ww ™ "
HI z — ' vh»0/fi ^feSal mmw.
WHte to play and compel Black to mate him in
fifteen moves.
For the first correct Solution of this fine Problem a prize was
cffjred by the Composer, but no solution was sent in.
No. 391.
By G. J. Slater.
BLACK.
»W ess WMk HP
m
-mm
~'wm£ /jm '
41
wF^rn^
H wm& „,„psk
TpFW p
J ill 111 ill
ip §§§ g pi
■%
m wm.
White to play and mate in four moves.
No. 392.
By G. J. Slater.
I ^]
it! ■ A ■
" iMk
M y Wm%L ^88%^ ** Warn,
^§ Ifejl \J~,....MMw.
White to play and mate in three moves.
No. 393.
By G. J. Slater.
BLACK.
'■W,
m J^ m JB\-& — «l
HP »
^m % 1
ggug
B8 ^ I
k mm mm
i« « an i
«f ~wm
■I * Ci HI
i
^ ^.„.™..^iii
i ■"■
in m ■
. . 777777. 49///////A <%//.
H
WHITE.
White to play and mate in four moves.
No. 394.
By G. J. Slater.
WW-
'WM0, BH mm?,
WHOM
■ Bill II
if 19 B Bi
my//////:, / /V//////'//,
■r~B~ 0TB
m ■ ■
1 ^ HSI ^^^
up.
White to play and mate hi five moves.
No. 395.
By G. J. Slater.
m. mm.
wmk Hi
m. -
wm». wwm mm.
ffl W£
mm
m mm
mm
'T-m^?~--
^-^ye^ "»* mm mm.
W.
- if^ill^^
"up*
"White to play and compel Black to mate him. in.
six moves.
No. 396.
By Edwin Dyson.
White to play and mate in three moves.
No. 397.
By C. W. M. Dale.
YW3 WM P
■—■L ' ■
m
AA gilf || Ml i i
ffl
m
Hr--
^ i Qfe"< Hi
iff
&„„„„„!».„;
B
p W"g^p" —
CtHl; 0" ! '
i
1811
White to play and mate in three moves.
No. 398.
By Fked. Thompson.
i/' K
WMMWM,. ■
HI ■ 111 HP
mm, s&f fli
In
m
■ Hi
mm. Wm-*~
White to play and mate in three moves.
No. 399.
By Feed. Thompson.
#
IP
il
■*J1 1. .11
i
v*
<%%%
mm, Us m
B ^ ^» ^H M
"White to play and mate in three mores.
No. 400.
By Feed. Thompson.
""wI /A
I
1
I
■I
mm
m+*
C\lll ? ^ i ill
■
■ «... Ill
III (Hi
Jwl£jOL^-jl
§mm—_Jmsm
WHITE.
White to play and mate in five moves.
No. 401.
By J. Paul Tayloe.
mm m
Wffl% Ww/
mm mm>
1 * '"prp™
- Y mk^mk r
mm, mm. mW:
WHITE.
White to play and mate in two moves.
No. 403.
By J. P. Taylor.
m* W/ZT??.
H ^ .mm.
I mm
.wm 'warn w/m,
W/Wi W2M lal
H
m
m «
i m
mm,, wm mm_
rJ\ mm, wSxt
%m
f*i
■iijsJw
WHITE.
White to play and mate in two moves.
26
No. 403.
By J. P. Taylor.
wm m
wm ww,
I ww, 111 ww,
wm wl
a.
'S/, '4s/.>,&& =,.;^
m i III
M^wm WW, WW,
White to play and mate in two move3.
No. 404.
By W. Greenwood.
**l - #
^l:^-WW
Mstjijl
«il
W^
^ww,
Jill
^^ iiip ^
H. Jill
m
White to play and mate in four moves.
No. 405.
By W. Greenwood.
■";■/
i
HP
lilt.
VMifPA W&jsysi
fliflL 11
dSM wMfr,.,,,, ISI
a fM, mm.
w §H^
?A'<
«T
J»i MH MSI u
p HP iff HP
—■^JsUfL-^
WHITE.
White to play and mate in four moves.
No. 406.
By W. Greenwood.
, ■ ■ if
W ilf •'
if 1 pI
m -wm § 7 A m^\-
m i t^MA
nn
;.
K'/zy//////. /?}—//.//,■
pi
lb
» ,,0JwM
White to play and mate in four moves.
No. 407.
By Rev. A.. Cyril Pearson.
iH wWk. ww3, * lii^i
"]» IB
'w9 r-\ Pi# c
i
■
is
Pi £
SwWw? WSz> mm
WHITE.
White to play and mate in four moves.
No. 408.
By Rev. A. Cyril Pearson.
WMfr W$$& WM
I ill
i is i
<\ii
White to play and mate in three moves.
No. 409.
By R. W. Johnson.
BLACK.
wm. / jm% _t™ mm.
v/^y^osy ± 'fiy^z&s f\ W/, Is/
PI A @#jfi
W/////Z& ////////////. t2/////M, / mmy/a
§llf„ ilii Jill ,„„0m.
I n«i
mm, mml " iy!
r-
II
M^ HSB 8BI ^§i
■ lH: ■
■WHITE.
White to play and mate in four moves.
No. 410.
By R. W. Johnson.
BLACK.
' "**. '1 Jk H-'"^
&«r# ^P f€m; » ?
I ... nn ... till J*. Mi
aril
i \
- 3 WW/
IS 36m Jtm
t*
'Wm mm
i
■ in ft
* »AW IPil IktW
id §a jM
IP
pni
White to play and mate in four moves.
No. 411.
By C. Callander.
K y< ^^ 'W^ '^^ & WMfi
.„,* JiHt x fell w// mw/
wm, wm wm aSI
White to play and mate in sis moves.
No. 412.
By C. Callander.
fcH
Va ... ?%"/
IP i
'■/.T7T.,,. ',..
■ A ■ i
J Wl UP
fpHI km,: fiBXi '■ (£» ;
■ ■&■ H
■Y///.// /.'. ty/////////. OY//A////>. :-■- -,-*/.
0£-
&% w mm q wm m
; .
II
;: ,,
"White to play and mate in four moves.
No. 413.
By C. M. Baxter.
ISP 'HP
m mm, * §§11
*■« -J^ wm ******
U'/M////'. W////M&
"""■■ 5g%g?
■ #IH ^^ _^^
White to play and mate in three moves.
No. 414.
By C. M. Baxter.
^ H
bH -si 111 ifti
iHI iiifl
rmi
w i
r mm « ^» is ip^
1 I
i 111 !§P i
w,„ im
WHITE.
White to play and mate in four moves.
No. 415.
By W. Furnival.
A
i mss
mm
.!■ IB
^P HP. IV :
P 5 HP 6 H1F
^Itl
i^ifi jh iBL
-^ !«'•
mihmw. mm m
p
& ■ p'T'p
H §mi ippf
rat 111 Hi
M iM
White to play and mate in four moves.
No. 416.
By W. Weatherston.
ir^nA. i%HI%.
b: :
i wm.>B~wm. tin
tt
^R B
i ^JN iff Hfl §i§
m^wk Jm mm Jm
mm %y\.^§h> \ Q uw
mm, , mm j** mw.,^~ i8I
Wm ft ill *
iMjlOEj(-i
WHIIE.
White to play and mate in two moves.
No. 417.
By E. Ormond.
IMP • «^
HP i ?
wkjks
mm @ i*
111
j til ■
White to play and mate in four moves.
No. 418.
By R. Ormond.
m
%W™W"fi
W7V
n nyi *„i§ii
- %"# - ii^
iHf ffl mA
■Z:
H 'Warn „, fflSZ:.,, w.
HP PPP
is mk
m if*' 1
WHITE.
White to play and mate in three moves.
27
No. 419.
By W. C. Spens.
^
maw.
■fflawM. x. imi,m
Wt. „ 'tCWA
fH mm
wfflm fe=s??i
''' i ''
M Ill H H.
til ill i
White to play and mate in four moves.
No. 420.
By W. Coates.
'WM W
-a ■ Hr mm. i X
"W*"
,W™W
m wmT m m
sfeggp^, ■/
i 1h
illi IP3»
« 4
■ i
11.
>A
111 A ■
WHITE.
White to play and mate in three moves.
No. 426.
By J. Menzies.
'/////sss/A
:
mm mm.
||
1 »«
'mm
£ fBllP^i^ Warn'
'''-mm. 4 wr~
mm | _^ | jy"i |r JBai
H ill
I
Us i^M s^
^H sW""
White to play and mate in four moves.
No. 427.
By F. W. Lord.
mm mm mA^^JLi
iPj% HP 1111 k fill
bI H MAm
'WZM 'W$y/ Pa# ^^^
H 4M JM,
, — ^
WHITE.
"White to play and mate in three moves.
No. 428.
By F. W. Lord.
HHP PUP W%m W^m
mm
^t ^il * khb
'mm. fSi
* 111 £ Iff
Pi IP A IP i
....J«
fgBk
m
'^BtM
mm'JmAfmi^m
WHITE.
"White to play and mate in four moves.
No. 429.
By J. H. Finlinson.
m wm
4% 'W;
£*■ ■
WW W
S88 ly//////M
■ f m&,-
i m
WHITE.
White to play and mate in four moves.
No. 430.
By J. H. Finlinson.
Sffil
'W/Mfi, Vv7i77T&
is;
PI
WM
mm iH
r ^., r ,.
|f: £ 1(1
mm
Warn vow/M vSzEsa
ip ph »**»
» -^ ill MM; q Hm
w
White to play and mate in two moye8.
No. 431.
By J. H. Finlinson.
Wm wim WM ^P
tmmit
* w r«
W/W/> Ji, 'W%W/ S3 ^#2^ a
fiJt§l# jy fi n
£SS5>;3rf - wl - W//Wa vmmA #W?M
■
,.w//////i„
^ == '/////,/////.
;S :
^s ^« Si si
g
Ai
WHITE.
White to play and mate in three moves.
No. 432.
By J. H. Finlinson.
mp up
"1
i
'ZZZV^ZZZ-
WaaSli
1 IHM?- SP^
iSl, feU i«
«# zzzzzvzyz/
'wm h wm
A 'JtU i;
"White to play and mate in four moves.
No. 433.
By J. H. Finlinson.
IIP wJw
■^^'^^"w^ m r wm '
m HH mm mm.
„^^.,f*r.M
fa ;w m
i
m m m
fyy, -//////M//. mm/////, feai
"*■ ■ >
y/y//'///^ %
White to play and mate in three moves.
No. 434.
By J. H. Finlinson.
mf
'W/M WyWk WyW>. W
\ mm. mm.
a hA^L"I.
up
Li
#8#
m " Ik JSBM.^.JKwi
IB H :
"White to play and mate in three moves.
28
No. 435.
By A. E. Studd.
W0M Ws
1 HH Wm
mm W% #k mm mm
mm mak ** mm mm
wim\
mW Wm mff\
-«m- fci
White to play and mate in three moves.
\
No. 436.
By A. E. Studd.
jh
wm ?ip «p™
y Jm. JET Wm,
i wm * ; f# mm $ ^»
M^Jj^m^mmjL M
mJzh. mm,,^ mm.
mm
Jm\ wm
* &
m mm m %m mU m
m mm 1
WHITE.
White to play and mate in three moves.
No. 437.
By A. E. Studd.
§111 ilHl A H^ ■
u
H
4L_» IJI.J1 if S
,J«|ii ^
WfM% -WSW; WiW%
'w. r\ W/.
Y *^ %M/»A y ^-* mMj. r
WHITE.
White to play and mate in two moves.
/ -7'
No. 438. Studd's Challenge.
By A.. E. Studd.
1HP HI
..— ..:
i
%/////A. #^^i
ill
1
\^J^^ W - "
ni i
?
Z: VmwXA
White to play and compel Black to mate him in
eleven moves.
No. 439.
By Colour Sergt. H. Woods.
■■
....11 mm I
www, ' 'W4W>, V'V'W/f'. ft
m
im
,if
ill S
'WW/?',
<* ^ »* WWW-
WWW
HUP Wv,
wJA — M^. w// „jmm. i
c&
Ml
"White to play and mate in three mores.
No. 440.
By Colour Sergt. H. Woods.
HI M 1
bL.
$\§BP
wm^wM. £bl
III
^111 ft
fH
W
X
WW??/. . ^
-//■■
1 fe
* ' B Bi
i
111 ■■ El
WHITE.
White to play and mate in three moves.
No. 441.
By J. A. Miles.
'Ok "7\Wm mm
mm.., mw^. mm
WW,
WW* a
m mt m w
==■ WW,
White to play and mate in three moves.
No. 442.
By J. A. Miles.
•*. : '"■■:■■
\ mm mm
m if *m ta '
111 Hi
111 WM
#%%%% ^
WHITE.
White to play and mate in four moves.
No. 443.
By J. A. Miles.
eg
n wm mm
Www
WHITE.
White to play and mate in three moves.
No. 444.
By J. A. Miles.
:WB, WM, WW, mm
Bf B 111 i
s. 1
Y/y//////:.,
f/A 'Jfimw,,.
'HP Pi- ;
ill n
»j
iiM ,;^-^
vim
*—m*
WZzM p
B 181
White to play and mate in four moves.
No. 445.
By J. A. Miles.
M„^,dMm£*^ wwA .. iH
fc
n " mm, 'wm
■ Si
WW "^ '
w 4M.1 r r
White to play and mate in five moves.
No. 446.
By J. A. Miles.
Hg p"p m
9W
jam
m
wb, m?///////.,
HI HH.
b
White to play and mate in three moves.
No. 447.
By J. A. Miles.
61* „
m M M '
"SW ' "fMTw-
a
m wm mm *»
mm
§
W...
Km
mi
Wm
mm
White to play and .mate in. four moves.
No. 448.
By J. A. Miles.
?;;^=y
■p
pp if
i ^« »s
3 ^
r ; A";
mm
mm W3 w Mil
LSI m?dZ. ^ ^51 ,,,,
iwm, WM 9m w*W,
wkT m m
J WQW
46m §Bm mm
§§i a jpni
White to play and mate with his King's Knight's Pawn
(gC) in twelve moves, without taking the Black Queen or
compelling her to move.
CHAPTER III.
IFOIRIEIGKET COMPOSBES :
European and Indian Noa. 449 to 640.
American and Australian. . Nos. 641 to 720.
Addenda, Various Nos. 721 to 736.
Dedicated to CHAELES A. GILBERG, of New York.
BY
HEINRICH F. L. MEYER.
white to play and mate in twenty moves.
Edle Kunst, die sich bewahret,
Wachst im Stillsten unvermerkt,
Bis, gesteigert und bejahret,
Sie des Freundes Fest verstarkt.
Goetue's Artist's Song.
Wahrheit sei im Spiel des Schonen.
Hermann Lehnek.
No. 449.
By Julius Mendheim of Berlin.
Wmm wmM WM& wwm &
HI
mm
iH 111 Mm^mm
m mJr m wM^ *ir
W" 3 ^^^
WHITE.
White to play and mate in seven moves.
No. 450.
By Mendheim.
4 4 Hi l'i ;:
i i III III
i
HIP d P
4wtA
■ H ■
».
White to play and mate in eight moves.
29
No. 451.
By Mendheim.
wm
TmvsT^m
iWw
WB B HI
mm mm mi
tzmfflM, mfiwwito, vtvpTZMt.
M mi
■■■MJr ^ mm &A v/ *MM.
White to play and mate in seven moves.
No. 453.
By Mendheim.
m§ w /M mim
I
n. §3
mi
i
W *i
#
#1
w...~M..
■B
i..
^„ asm, <2 ■■■
m ^ tiiii
WHITE.
White to play and mate in four moves.
No. 453.
By Mendheim.
mm///, '==' mm// *==> ssbJbsss ,
mm wm & p
ill wmp> wM„ ^^
f HUPP K ikr
•§ 6 fill
w,
..ill , Wm3.'0m%
w,
'WW'
mm P»
r;
i till Ml if
White will give checkmate -with his King's Bishop (a2) in
twelve moves, without taking the Black Queen.
No. 454.
By Mendheim.
— Wwk
!^§y^
Pi BMiP
m///y/.„„ ///////m.„„ rr ,>.
'HPIP
IIP Willi #£# §yy§
ill to =tH l»l fOl
^ IP 1
MJBLjil
HI
mm,
W4WA ££ ( W4W, V/6WA a 22^^
fell fi
susses 'WM/."^... v ^^M.-^^W/m»-
\ WmmWfM
mm mm
Worn
m wmW/ WMW/
White engageB to mate with his King's Bishop (g2) in
ten moves, without taking the Black Bishop at hi.
No. 455.
By Mendheim.
BLACK.
ML
wm
■ HP
My%%9%, 't/MS/M ,„ty//////M:
■ Kl
H II H
„.JH „„Miy/////. rr v//M/,...,
wsiWt. WW-
B
HM Is
•Ml I 4
^^"i"^ iilj?k
m vm JRm vm m m
fit
l
White to mate with the Knight at King's Hook's
8th (h.8) in eight moves.
No. 456.
By Mendheim.
BLACK .
i
1JMM *■»
nasi . ^^ mm
f s| ill i n
&s^ ZZWM„, M?m»t. Wmm
HI PI WB
v//',<»/ ////// "wy//y///'" /////// *- mmXm..
W//S///JW.,.
H US
^^ ^^ ^^ HHMI
ilil
m mm Wimk
"White to play and mate in nine moves.
NTo. 457.
By Mendheim.
wm mm _■_ mm
mm
mm
ij^ ill mm,
m 'wm
■ ■ ■&■
HP *
m wwF m w- ™-
%Jm . mm
m mm mm
WHITE.
White to play and mate in six moves.
No. 458.
By Mendheim.
n K IB! H 1
*^ mi
mi „ „1M§., #»
■«
^»
i "<
""m„
W Ml
IM1 «4
m EMM.
PPf
;i« Lfe. ' .'
^^
«g ^
P 1111
White to play and mate in seven moves.
No. 459.
By Mendheim.
HI
is m ■■■■
ill
^
mCwm
P gwH I-
WHITE.
White to play and mate in five moves.
No. 460.
By Mendheim.
$% M M M
my//M ',//////M. wmf& % g "aa
II§1 ~gu r "i^ i HHI * f||li
#±11 illl IIP
p if fi
.'i'J :.^
^ J: -J
Whito to play and mate in sixteen moves.
No. 461.
By Petroff.
11 i
mJ...,Tr. ■:.:;;%%}..
^■ mw mm - ///
mm
i
JR4
:„,~;,.A 'M7////M
wk
White to play and mate in five moves.
No. 462.
By Petroff.
»i Ufa jhl iBI ^B
i» * ^^ mm 'mm,
■r"i""K =""'^'1 i^ if
«■ 'm w §
m mm
WHITE.
White to play and mate in eleven moves.
No. 463.
By Julius Bkede of Altona.
w
mm tm
Jmwm
I m
HP a HP
z%Mm v/MW/ wMu?/ >< ■ W '■
mB, w -mm jmfr.,j^ ?M§,
g™p mT A W m
"White to play and mate in three moves.
No. 464.
By Beede.
BLACK.
MM
I %B,
Wm
H HI ■ i ;
'/////.//«■. ■%%%?%. Wzam %&&?
. mm §§p mm Tn
p,
mm ■
WHITE.
White to play and mate in three moves.
No. 465.
By Brede.
W,
'm
^K -"* *"
iiit ^
m m
-■mm
■ i'
V/y^jwww.,,— ««. / warn
Wm, mm „,„„ mm.
WHITE.
White to play and mate in four moves.
No. 466.
By Brede.
™9a
%%W%.
....
WW/,,,
m
m mm
JBm
WAkmm
wm ■mm
Pi
W WwJ
I
,///M w
I^i
'■www
I
White to play and mate in four move3.
30
No. 467.
By Brede.
<&*WM WB HP WB
wffl y/Wwi Wzmfc Wmy// imEki
mm, .mm ,„i» w i«
iJL
; pa
^1 „ V.W5A
M «l
#M# #111
i lis II
wm,
WM *
'//////////.
White to play and mate in four moves.
No. 468.
By Brede.
1 f»
WS.
w, ww//// w/////////
m ^P ^P
li
&'1
Hi"
V//////M
m iH
pi
WmfM,,,, HHH
illl 1^
i
WHITE.
White to play and mate in five moves.
No. 469.
By Brede.
'mm
wm
Jk mm-
: <&4
— I: :;
in
W 1 *
mi m !#i 4 i
am el 1^ * €M
W ds PIP Pii i
■i H H A M
«i.
^
f A §
(#
m m&m
Ill Ill jib
I Slip
White to play and mate in five nioves.
No. 470.
By Brede.
yy/z/s.'/,//,
1
n.
til
.'//////////. V/////////A 9.
" ill w
HP in
■///////M
:■ Bl
Wixtm
i HLJHI « fi
Wm
111
p ^^i
WHITE.
White to play and mate in five moves.
No. 471.
By Brede.
Iff
""" J wm A wwi" / "" A WM
wt mm Wm
'^■„,,„„„P^K„„„„f^A M ^*
$0m'%0' ' 0/// a §^j
^mmj^'"
W, . iJH
White to play and mate in six moves.
No. 472.
By Brede,
HUPP . wmk Wim WM
White to pla}' and mate in four moves.
No. 480.
JtL
By D'ORvitTE.
Mi
%>./■
■V- **
Wi ~^— '%/W/m -ZZZTA
m ■ m
""iL
%"'::".
m wKmF m m
WB
m
m
JMRTmr
Wm% HP WM U
wW/?/ wm% Wtw WM
Avt 4
wM. v ^ mm w 'mm Jmm
iHf mm
White to play and mate in four moves.
No. 481.
By D'Orville.
^i?
mm
111 !«l «
HI A 111 iM
IP
4m,
i
^^ ««„
^jmm^.
White to play and mate in four moves.
No. 482.
By D'Orville.
HIP
1 illl
'•,/?$?. ' $/////$. %////;/,//.
1
^
IjI; ■ ^
"si -
? J
il VvMrf/. WftMi 'J?/////////,
WWi 'WW
m..B..3Mm. 'm/.
% fm fm
P
mm
m,
m mm,.
White to play and mate in five moves.
31
No. 483.
By D'Orville.
""S*
.////My///?.,,
% %/////////.„ ,„,(»«!
m m Pi m
«p mm
i
i mm wm
-V0S/0A V/SSS/SA
m m m.
White to play and mate in six moves.
No. 484.
By D'Orville.
1 MM
i
...iff '^ wSb. WMk, HSH
i 4 HI Hi i
" W Iff'
al
flH ■
» ■ ■
White to play and mate in six moves.
No. 485.
By D'Orville.
S/////S.'///.
wM mm
WM6
m
y.
■ B
"**■ wm, pa
§m mm ^
air a 11
..-.^ •==!=» %JS^ %///M r
■wyV
i in
imp A mni
.■W/////ZV.
■white.
White to play and mate in six moves.
No. 486.
By D'Orville.
w 'fgf"" A 'WB
wM^4Kk
m mm mm
m w ■
§111 lilii *=LiMi,
JO ill ill 151 f
if£!
HfiH H
48HI1 "//Mm W//////A
m *■ i-
White to play and mate with his Pawn in ten
moves.
No. 487.
By J. E. Kies of Stuttgardt.
-ill 111 S
y /i&ay fr v;.?'??', i''.'/" / '"^ ■,-■■."■-, "^ ^^
■ i ill iii I II
mm
'.■/.-'
wm wW mm,
mm mm, mm
x W'"''y *^WW4 1
■ ■ ■ ■
■
...... /////. %/////!%;
White to play and mate in four moves.
No. 488.
By J. E. Bies.
P
thHbG"
W El
^--w—W™
m mm
w""" A w
Wi Y ,
■ 11 i
m
» ^P c
fif
i«„„„ wm a£s, ^m
I.
WHITE.
White to play and mate in five moves.
No. 490.
By J. E. Ries.
: W%
J
,f#l |fi
. s=g S_- #2_K I1I§
V/////M ~'— i HlH iHI
^3 '-'/A '; A <
^^...S„^H„
w
i
L
11
l El
1 IlH
i Ws> wm wmk
WHITE.
White to play and mate in six moves.
No. 491.
By J. E. Kies.
!§■ * wmi. Wm„, 4m
'/'/■'/Kg?/ W
m mm.
ml. ™ §BA mW,„„,„„„
-mm 'mm mm
Ki mm.
m 4m,
■WmF m m
p w
mm
w wmf,
White to play and mate in four moves.
No. 492.
By J. E. Kies.
a ww, mm,
Wkw w
Jm
'■/,/'■■ ,
mm wm i
in
m
m^%. 'Jmm. wmm . W:,//s-6.
■ i
WM HIP A ;
X. 1 lilAi ■
White to play and mate in five moves.
No. 497.
By Kieseritzkij.
<0^A w,/A//, %7^A ^^ Wm&
<
W3 ww% '%%wi wzm
M M til M
mm
LssJ^L^j^m
ms& i mm
0jy& m J0i mm , H
wm
WxaM WtWfi.
WHITE.
White to play and mate in sis moves.
No. 498.
By Kieseritzkij.
_
HffC
I i t i ;• gj I
mm,.^,&A Jm^>, y/// jhHL
I
i
8
§
ill
a "job , m
White to play and mate in four moves.
32
No. 499.
By J. Kling.
BLACK.
w///// 3 WM WM
®® ™p^"
"White to play and mate in four moves.
No. 502.
By Kling.
■^J^-HpA'
I
1
illl i
WW WW?'
m -wmf m m
...WW,
W WW,
Li
WW,
WHITE.
White to play and mate in five moves.
No. 503.
By Rling.
^mm wi mi mm
!
mm ~mfr^
fta pIx
JliJt|S|
JiH Jin
White to play and mate in five moves.
No. 504.
By Kling.
^ ,,,,,?i2^&.„„,, r ,„JZW%%y,
W%0
m nm
mm
#282? '4%
Wi 'WfflMk. Y <^yM, y #mwM<
mJ**WM..
WWa
l.. T B ■«■
White to play and mate in four moves.
No. 505.
By Kling.
«-" W
^_»
4.
W£$ * Hip
fiy * ill
JHHftJJW
White to play and mate in four moves.
No. 506.
By B. Horwitz.
WHITE.
White to play and mate in five moves.
No. 507.
By Hokwitz.
m
m m
m^ mm
V/////////.
«ii» m
¥M, I*
I 4M. mm...
PW WsM Wfo
m m
w%m
.11111 mtvA mW/,
<1 wM mm,
WHITE.
White to play and mate in four moves.
No. 508.
By Hoewitz.
>=■ ,
m -^Jf^- m
w mmi / ^4&m^.,.mm.
■mwr-a
m mmF m vmm
Km "■*■
White to play and win.
No. 509.
By Kling and Horwitz.
IK
I ,,,,„,,wm mm, W//////M
wmm, 'Wm WM W
mm,
"-'; ,
Wff ■
WMi, t ^fH8%[, '/?////%#//,, %%W//A
■P W ifi f^-jjjj**
m§A mm
w. mm,
WHITE.
White to move and -win.
No. 510.
By Kling and Horwitz.
WKk_
m w/M.
mm p
mm,
™,±±.J
mm m w j0k m jm u
m wmf m m
...m mm, mm,
m 'W§
WHITE.
White to move and win.
No. 611.
By Kling and Hoewitz.
" **?! MWi.
wm„
7?'"'rz-.
m -mi? m mgF m ma^
fi irt
WHITE.
"Wliite to move and win.
No. 512.
By Kling and Hoewitz.
Vs.
■P I
^ ^ «11
m mK J am
■■"■^ M 'wmf m
m
m. Warn
p
fa
"PPf 4
w$ 1
4 '///////Mi,.
m
mii
W//;/////j
WHITE.
White to move and win.
No. 513.
By Kling and Horwitz.
;
m *
i
Bl
s. IIJI^Hi ^tiBk
W"%
4k
m iM^ f im m
mm mm
mm
WHITE.
"White to move and win.
No. 514.
By Kling and Horwitz.
WHITE.
White to move and win.
33
No. 515.
By A. Andeessen, of Breslatj.
Wm
■ Hi i
m *Jr-
m wJ m m^ m ^
mm mm.
White to play and mate in three moves.
No. 516.
By A. Andeessen.
fill w |Bi JI
H lii ill
m -,J^^ mm
w.
W//s////yyh
H warn, iill
%
WHITE.
White to play and mate in three moves.
No. 517.
By A. Anderssen. ^ , ,
BLACK.
illl 9 §111 iHI
iHI X HH mm.
'W!W;. W/Mb
■ x 1 1.... 8 i *
fat
W&3 Wtw WMfr
w mM,.,
•M4 Y%V///////.
m:zz, m
TVHITE.
White to play and mate in three moves.
No. 518.
By A. Anderssen.
m>
■ H
ii ii mil
mi y/ , w mM r mm,
1 ■ m m
■ mm _JBL
White to play and mate in four moves.
No. 519.
By A. Anderssen.
g8*gi
1
%
ill 11 II fc„
- ' J8jL
pi pqJ
i§H§ H
w 'mm, mm.
m.
w mm
..mm. * II3J
b, mwi.A..dM
MM
White to play and mate in ^ve moves.
No. 521.
By A. Anderssen.
M ] WLifo\
wm*,,.,wm , mm
; M_4^M»
111
% Q I
'/y/////
- lr~.i&^
ill ill
ill i ill
im # m *jra».
wm,^ mm.
WHITE.
White to play and mate in five moves.
No. 522.
By A. Anderssen.
3 '%%%%?%
ii
iff
1 J
./'
II. „ if i_ ju *jB
M
H *J8w.
Hi ■■
*WM. A W\
S
H "^ HH * SI
White to play and mate in five moves.
No. 523.
By A. Anderssen.
■
mm,.
lb
flJLpJ#fli
w HP ■ HP
m w J
ii
. fc^
S
JT^%^
White to play and mate in five moves.
No. 524.
By A. Anderssen.
W$M WW, 'MM
mm mm mm
% V/////M HIH
HI imm
mm, I
III g| [jj
wm.
j
White to play and mate in five moves.
No. 525.
By A. Andeessen.
JS& is Ill
iS tilll HHH HHi
^ ,„,, ^«
mm... in.
H 8
i 4
HP
'_ ;
»/?,,'~^'%MY/%i, i
...m y/ w,
WHITE.
White to play and mate in six moves.
No. 526.
By A. Anderssen.
,,,.,.
i
^^V^
« mi ^ mem
Wa Jill jillL.
m
m
White to play and mate in eight move3.
See No. 98, by Bone.
No. 527.
By Conrad Bayer, of Olmutz.
mm
JH mam
wjm. mm,
Wk w, mm'
. 1m **, mm m9. m
v//y/,7"'.
i wm A iifJ
mm m
'mm ff"
mA III „ . WMM, •
PP IIP
§1
mi mm
WHITE.
"White to play and mate in four moves.
No. 528.
By C. Bayer.
BLACK.
w^SlfiJIt,
m
w////m W//////A
1mA mm. mm, Q ifSi
mm 'mm , p
'mffl. y//fflyyy% X «?
HI ^^ " «§I
«1
rn^^wM 4M
WBi
WHITE.
White to play and mate in five moves.
No. 529.
By C. Bayer.
'////'..'.■■//■'..'.
;...-..... ,..
. si;;........:
White to play and mate in three moves.
No. 532.
By C. Bayer.
«
»s, ^ *«:
\ k\4m |K (f
/ m„„„„„, $&^//Zi.
Mi
• P
w mm
1 HH!
HI
WHITE.
White to play and mate in four moves.
No. 533.
By C. Bayer.
Ill
& ill «
i i
/.•^r^^i—*^ !"->■, ■■■////*/
S^ „ HH AH!
BLJBUbii
JH «
^ fli ^^
«« HP
WZ&'A
WHITE.
White to play and mate in four moves.
No. 534.
By C. Bayer.
H
i HI 111
■ ■ ■
fMf §K Hi ^ peg
M m
"«... ~..,.^^/'...tZZ,,w"M'M mr////y//. .
"- i ^
i^iM Hi
"White to play and mate in five moves.
No. 535.
By C. Bayer.
. '/W%
S wuJF m wm*
3 P
^«W rf P rf ^
41 S im i
m
mm a €m
m
i
m mk 1 jfSA w jtm
fm fa.
White to play and mate in four moves.
No. 536.
By C. Bayer.
m HH
Hi
nm i
m 'mm, m nil wmb,
m m ■
i. H
iiIj5?LilSi ™
v iai -
^W™ ^
P?
WHITE.
White to play and mate in four moves.
No. 537.
By C. Bayer.
BLACK.
hhp ww> www www
. ~*i 4////////A WfflMMi VXMZiZ,,,
'warn. mam wwz. %
WWW, iSi.
i III H
i
w
■:
■ww ///A
mm ^WWWW,
WHITE.
White to play and mate in four moves.
No. 538.
By C. Bayer.
nsr
WW
\ \ i
% WW i
mam w%""
I fit §■
a * 111
wtm wow, „ ^
d
whtW W,W:
A IP
■ Mi
Mm
WHITE.
White to play and mate in four moves.
No. 539.
By C. Bayer.
HP HP 1
- 1 — m^-M
m
«
m mm.
mm w^m , JJBBBSi
%?M%% illlllP ^sUP^
Wf WW 1
w tf"mw a wm' i''S
iM if ill i
■mm/' A fflffi A " ^fflm*
m II P
WHITE.
White to play and mate in four moves.
No. 540.
By C. Bayer.
::.'.'.;•>'; 1
m
M wd:
m
i IBS
^ i^j
...M ml
mm mi^
m * HH ^.mm. . wsak
mk mm,
** y SL\m mm
HP
mm, m
White to play and mate in four moves.
No. 541.
By C. Bayek.
WmW
WW's Wb
•I.i.i.:.: M
. fe :
S*/SSSSS/Ai.
mm. .,.#S§
mm ill
ill ill ii
JK '%m. r/ ,.^m^ mm
White to play and mate in five moves.
No. 542.
By C. Bayee.
¥M M
msm -mam mk% mWzk
m
HI
m
■ mm
§^§»:
ml wm
mm. Hjl.1
'■:,
if
W^ — ~™|^
White to play and mate in three moves.
No. 543.
By C. Bayer.
il
mi
■ ■ Al
m * is at
fAl
<%* W//////Zfr
WHITE.
White to play and mate in five moves.
No. 544.
By C. Bayer.
■I
mm.
y mm mm,
m mm, mm,
dj\, i am i
wm, mm mm
mm i i
"White to play and mate in four moves.
No. 645.
By C. Bayer.
*mm *. A WB® A A 'WM i**
M mm m i
■ in. HI
i III M fl
m^Wf^W^W
i
n nn
WHITE.
White to play and mate in four moves.
No. 546.
By C. Bayer.
TBI ^
p ^ 2 "
w mm
W/////A M. ■%
mm.
HH * ILl
-fli
i«i
*-j-
%»*>
1 Hlb
^H e ^»
iM
White to play and mate in two moves.
35
No. 547.
By C. Bayer.
.Ill
wm.
w.
■ ' I
y/Jm «-*- Hfedy JmH
WM,
mm ..wm,.... mm.
r
^
mi * is »J, n ;
Hi^ HBP <** ^
Oy////S/7Y/.
mm wm.,,^www,.
mi tm
m
wm "mm
WW%. /\ WWw. WWm. » 8%
White to play and mate in three moves.
No. 548.
By C. Bayee.
ft wm, 7/ mm. mi.
m:^wm.
im,
m mm
ww,
HU, WM,,,*JWWa^,'wS...
\ i
= ^AJ^WB,,
W, '— ' WW?/, BSO WWZ,
nm warn, ^ » --:\ ,* m,
WM
White to play and mate in four moves.
No. 549.
By C. Bayer.
mml » wim mm%
f WMfr '%mW W%%
wm mm
wm% k W$xk k 'mm\
JmAJm ill
^M x 1
S i Is**' £
■aL i#.. J ^ •
H 1 1 V
I ^H ; ■=* ilii
!
I^Pi
IB*'
^ ... .
White to play and mate in five moves.
No. 550.
By C. Bayer.
■ j jUB£i(B
= i i||p ilP^^IP
> J 'wm J '
■WW-
iA» i ffi
Vl pi i s ■
IF
mm '
y ' '■ sis
'mm ~«
m. Mi
"White to play and mate in five moves.
No. 551.
By Rudolf Willmees of Vienna.
WSB PUP
Pi S
If
W/ f/r^ m
«l.Jf ■
m w^wm mm - *■
€j i ill
wm. WM W9
■" "
i ill gft
*«• iiii
White to play and mate in four moves.
No. 552.
By R. Willmees.
v: _ '''///a
w W III
■ mi
m mm
i i
mm mJ^m
mA
m my/M
^
■■■r"W
w -
White to play and mate in four moves.
No. 553.
By R. WlLLMERS.
HP n ap
Hil |§lll 1 : =:.
^■■'■■-y/sJ™^'.,
ill lil i
V////////3.
W I*...
si
WHITE.
White to play and mate in four moves.
No. 554.
By R. WlLLMERS.
«... ,* iiii ?.^ .<
1
p
. ..mm 'm
IP i =
r ^ W"" J "'
^ wm.,,,,,,,,,'^M„ HH
B mm
m mm
mm x A wwM
mm mm
i 111 iBl
WHITE.
White to play and mate in four moves.
No. 555.
By K. Willmers,
I Mm Wm ife III
m WP
Hi 111 Hi i n
n§ m in
iiij ,fc
w
m ..." m.
: 4v///////A
P
W>/%& V//////M& mmH&
|f| IK
V///S/SSML
.*».
i
WHITE.
"White to play and mate in four moves.
No. 556.
By R. Willmers.
^;:
■ B ■
I p
K^ V/M/Mi
WiM W*W, W?M> W&Wa
§1 §1 *■ **
pi fil Hi"
"^■A^JL**
■^
■
^\
m ■ i
"White to play and mate in four moves.
No. 557.
By E. Willmees.
JiPPI ww- HI
mm mm.
ifl
m*mm.
M
..mm s mm. WSsfc 1W^
m m
1 i mm\
mm
WHITE.
"White to play and mate in four moves.
No. 558.
By E. Willmees.
'wsm
H h Hi RIaI
i
ill!
P. ill ,iBL 1
§|^^ ^^^ ^5%%%
#1 Jfeff
WHITE.
White to play and mate in four moves.
No. 559.
By E. Willmees.
V///A Z'/A
**.m H
" J ^mnsd"*'
m^m, mAM.
JBf all fijf
WHITE.
White to play and mate in four moves.
No. 560.
By Ivan Kos of Vienna.
*/ tSS 'SSS//s
4rJBm.
mm
i
ff/SSS/SS/M
M mJ mm mm mm g-
^ ^gSgifej MM%M
WHITE.
White to play and mate in five moves.
No. 561.
By Adolf Rothmaler, of Nordhausen.
m MdH
mm wm wmrsx'i
■ "™J Wmw. . V~M-
■P — *
A HIM ■
I ^m]j
B^ i &A/MW// ^/79A// ■////'' ■ ^K 'S3!'
^^ ■yWsmM %ym»y%, WxWZ w/mmw.
HI tJlli :. e3 WkW,
mm
■ iff HP ill
9 §o iw is
*%HHI %////////>.
m mJ m '-mJ m ' m
WHITE.
White to play and mate in five moves.
No. 565.
By Anton Novotny.
JBL
..PH. WMi
m
jBSl
W/////y/A /$y////////j ///Zv7?///,
wwg. v-^ wm
-,i
■ m wm mm ip
WHITE.
White to play and mate in four moves.
No. 566.
By H. A. Stavenuter.
"%Mw/'"' ip^p
ft.
mm
i
m
HUJi ■
Hi- A Hi
JLra H Hi I
V///s /.-//, A '///s/s/s///
■I ■ ■
^w ' ai
mm mm^ mm m
.
1 fil
^^ p»~ '»» «p'
WW/, tpMmyS,.
«§
White to play and mate in three moves.
No. 567.
By J. G. Schultz.
mm
ww, ww.
HI HP ■
i ill Si liL
WW WMfr W9
mm Wm w
i m * ■■
'/■Tip'!!
HH * la Q HH,.
^ M Wf M M
il . .. ^SJ
H IO.
WW
mm
White to play and mate in three moves.
No. 568.
By P. Klett.
BLACK.
MP HP
SI ■
w///////,r"'"'\M
ww, ww,^ ww.
i
Wi
I P 1
£ g -sir
'mm,
1. 111
lira
fii
"-Sip m ^f M - w
,,,JM Jm...
w^wm
White to play and mate in three moves.
No. 569.
By P. Klett.
Ill Ili
m HHi.. mm.
PUP Wm
fm wm
% M.
w #§H1 x HHi
i§ i — 1 mw/%:
I ^
^§•1 I
\£hw
wm^ mm mm mm.
White to play and mate in four moves.
No. 370.
By P. Klett.
H
Wl
1 * III
° IF
I Ill
i ^/wm * ^^
■%y^%a r %?////////k, .......mmM, «s§
...:m
■P "
il nn
White to play and mate in four moves.
No. 571.
By S. A. Sorensen.
I B Wm. m
wm II ■ •
'mm mi i
iWW> 'WW>, WWi W%
tim .mm mm,.... Jm
Pif WM
i, s HU
J
^p B'I'B ■"
«K. M
w"'"" /M WM A *W"'
M M M ii
wmmm i] gj
Wi m ,„„„, WM
hhp M'
"White to play and mate in five moves.
No. 575.
By J. Plachtjtta.
«»
wm 'wm.
i
-mm v&m
m m fr^' 9////
#JH„
¥ \J w f / ''mJ m 'm
i • » i»
White to play and mate in three move3.
No. 576.
By J. Plachtjtta.
'-:'/,
m», „,„„,„ W//////M,
© tm
'W.,^. mm «i.
'■W"W '%W' / /%' "/■''jffl?/ '■%%■
nm lui
■ 61 M HI
.„;
White to play and mate in four moves.
No. 577.
By H. F. L, Meyer.
HP HP H P
m
Hi . , Hi r „ Hi„ H r „
IB HiJi B
^^ ^^ ^^ n^f
• *l 7 H ■
H ill §=..
ill
in
n ^»
WHITE.
White to play and mate in four moves.
No. 578.
By H. F. L. Meyer.
I WM
111.
9,,,„„wm
\ wm
y mM mm
i
mmi*zMm.
m
H ^ «lf JH1
WHITE.
"White to play and mate in three moves.
37
No. 579.
By H. F. L. Meyer.
WW
w.
jm ■ ■
m
m
w: y/
m wsm
mm ;
m
m il
m mm
■ A 1
White to play and mate in five moves.
No. 580.
By H. F. L. Meyer.
W,,„„ „jll^ Jill vs5tJ™^'»
mk wk
■ ft ■ g L
i MP tt Bl
* «^ mi im
- ill «i m
'w%m} WW§ HP^ ° w
if §1111 Hm
wzz
wM,..
W
WHITE.
White to play and mate in five moves.
No. 581.
By H. F. L. Meyer.
W. %Si
mm m
a: ""K~ww4
HIP iPiP 5 X ^^P A ^^#
* PI Pllf
m*Jm£A
White to play and mate in five moves.
No. 582.
By H. F. L. Meyer.
1 m , WB, mm 4m,
mm, ...:,;
llP^iii
W. mm. mm. ^ Si
vkMh mm
ww IP i ill W
P& ; 'i JOjl
IS.
m w//////m.
White to play and mate in five moves.
No. 583.
By H. F. L. Meyer.
m m
m
■i i Bl
..mw/'M...^.... W//////A
1 i Pi
///////////a
&J *■ 4M>, mm t
mm iCl iSJ
m H
' y 4£A ^^mk, fa^
'WW*
m
■M>////#M ,
5*
IS in
PL,,
.'//////Mb
iai ■§■ is
w ■ ■
€#J nn i«
nn
■r Hi ii
IS mi Hi m!
WHITE.
White to play and mate in four moves.
No. 588.
By Kohtz and Kockelkorn.
iH
w/////.'/^.
WWi & mm
Mi HJ^W
JIJ ml ■ *
CJ §■
H mi
ill
','/<>, .-■
■ *■
*%^
K Km 1
'ft—///:
w,
Wm m
1- PS
^^^gjgjg^^^jgjjg^^^^^ ^
L White to play and mate in five moves.
No. 589.
By Kohtz and Kockelkorn.
HP -
mm mm mm
I ill
'"■;•'
; < *
:.^; * t«
i & I*
' ._ '
4m fij ci
IP Plf ~§r
White to play and mate in five moves.
No. 590.
By Kohtz and Kockelkorn.
wf WWM^'W" ~~'
ifli llfl
i
ill _ ill .jbI h I
1
^
I il
i fa
i ■ if"
'mm
u Ill
White to play and mate in four moves.
No. 591.
By Kohtz and Kockelkoen.
WM. j*v WSM
%msm — mm. :
I
m a IB! A ^^
■ ■!
m wm*m
§1 ^»
White to play and mate in five mores.
No. 592.
By Kohtz and Kockelkoen.
JM ■ fi
^,,.,,,,,..,0^, ills
r i
si
nn a fill
PS pal
w&m.
mm
a
. i -
^ W/////M&
wm, mm,<
White to play and mate in five moves.
No. 593.
By Kohtz and Kockelkorn.
ml
:
m m
o
n
«i i ^'^M 'Wm.
'Sag' SO
ms pip
Si
;jl;
;.,.
2 '.;
as «1
HH ^H
WHITE.
White to play and mate in four moves.
No. 594.
By Johann Berger of Graz.
WMk^%
fe
(■L iH Hi . Bl
til
.,.., ...
■ JiL'#By
V///fS77ssA
■-'
Wife}}. ;4MM '
BL Hi. HLJH
White to play and mate in three moves.
38
No. 595.
By J. Beegee.
%
ftp?
i ^^
m m ii
■ iBdill H
J is „
■ HI
I Ill
I
'//////M„
Wi, mi
White to play and mate in four moves.
No. 596.
By J. Beegee.
r
«v, —
I ■ ■ill
in lii
V///////A.,
" /a " wm ^mtm.
. Hit wm,,£jvLdi ■■ £#J
8H H«
^1 ISpl
«L iK
,;<
I H
HHP P
I ^
"White to play and mate in four moves.
No. 597.
By J. Berger.
YW'% i fp
mmV^ mm. mm,
WW, W a !i
W//////Y//. V///////'/i V///S//J/Y/.
'ft W///MW. V^/////////). ?//////////.
'Mm WM
mm wm
• m isi
"" j ■ m^m
Y A w 'wW,
mm mm mm
WHITE.
White to play and mate in four moves.
No. 598.
By J. Berger.
BLACK.
lit"
1H
m ii*..
Ill
"■81
m -wJF A m
m §^-» ?
l» gii urn
WHITE.
White to play and mate in five moves.
No. 599.
By J. Berger.
BLACK.
w£m, wzmm. h. ^HH
ill
"m k I : ^
mm A W; mm wm
':. if 8 IO
•sJJU/I
HHF 'HP
"" i
ft///////?/.
White to play and mate in four moves.
No. 600.
By J. Berger.
V
i
1,3. .3
M„,„ m ,jmA „„„„
X s :
m. ■ m. Jm
i
H^i H&H n 1
w»k llll , * Jill „,,,,„„ ilK
H IHI
SB ^i
gig
White to play and mate in four moves.
No. 601.
By J. Berger.
Ky
™ ' m "wd'
w.
■ ill
i H H * 111
■■ ■ R R A
ll^Jig^liJLlI
^A^'wM mm. ^.....jBl.
WW,
W//////A
t S 'l£m MM m.
kZ%^3.
White to play and mate in four moves.
No. 602.
By J. Berger.
BLACK."
J"LPLJPLJ"
4— Aj^LA
WW
% asmr/,
in
m
White to play and mate in five moves.
No. 603.
By R. Bkaune.
/A w
itm
mi
y?>-/////yy/. j
wa 'mm. w/m wm
, i «^\H MM
'MM *^^=» V/y/////7//. / M7/X//M -ifT^TPli ,
«gg% ^i-^ M^ ^
mm, mm.
HH E#l ... IS.
III
m -mm m f%
m mm.
i s
White to play and mate in three moves.
No. 604.
By E. Brauke.
sT« — pTr
_ r J« -JBBL
>; 's/s/ss/s/A
mm tfH "m^r&m.
■?'
mm m :
Hill wmi IS^I
kW% HP i v
H III
i ■ a ■
nm lllJl iUfa ^^
^ JgHi.
§
^.**??T ■x^^i.
im%.
White to play and mate in four moves.
No. 606.
By J. MlNCKWITZ of Leipsic.
111 ^wi iO ill
■i Ml Pff
mm Wm
■-pJM^M.
wm&
1
MBH H
Wfr WW/-
i»-///////^ WWyyV/A
White to play and mate in three moves.
No. 607.
By J. Minckwitz.
my/./////.
m WW/
"■ *mr$
;#! 11 B ft
H S 1
is iWJ Wwm m.
^■mm, mm...
H Wmk /Wk%
White to play and mate in four moves.
No. 608.
By J. Minckwitz.
■AAa —
1 fij J» J|| Jfai.
iHH JUl
r#$l
W/////A , „„„„^^« ,,„,,,,,WWw/,
"«H«45W ,
«S: Eli
■ IHIiviM * 1
fll
.»,;
yJ~'y/mm^ //////A
wm*. m
White to play and mate in four moves.
No. 609.
By Rosa Fessler of Vienna.
...m 'mm
I ill III iil
H i
.3, §M * ^m\^\
HH fet. « * « *Z-J i
iil w lfil m
I
-_— «
m w J
V////////A
White to play and mate in three moves.
No. 610.
By Bela Szerenyi of Pesth.
,„.^» ^»__^»
ipm 4mi ' * A 'wm,.,x f'.m
W/.
4qA wm% %Q)A
White to play and mate in three moves.
39
No. 615.
By Franz Della Toeee.
■■ m
"" \ 'WW,
m
wk ww%, * Jilt.
'mm
: m% A ww wm f^>
%z£%g2i «^ S3 =* WW//, , v^M„,,,,,,,w///////A
m
iwfli mwl k
wm *** mm fm,*jww
■
mm
iszJU., %5zmM. . „ %BiJ 'Wwl.
V%77T7s//S. ws/ss/jm. 7y L, ,,,} /, VM.
WHITE.
White to play and mate in four moves.
No. 616.
By Constantin Schwede.
W% ^? 111 ;
HI ■
mi l
wm,
iH ll
1 m IS
z ' ''■■/////////%, ^m
j & mm
vww. %
H WW/,
"White to play and mate in three moves.
No. 617.
By A. Baybrsdorfer,
mm
M 'i
mm
■ ""•""Wf
wM,„-0M. *. IM
I HI
i iSi
W> V/7?7/77sJ,
W&
■ //m m,
ifil
<§&»:
H §1
mm
m A
White to play and mate in five moves.
No. 618.
By Oscar G-elbfuhs.
£3
*■«■*
~ -*RJPi
w,B'wM
H
pm^tiSm^
WHITE.
White to play and mate in five moves.
No. 619.
By WlLFRlED MOSEE.
W, y
67$&7& VOKMtA J^8st« %////////',
wwz 'mm 'WW/, _h iii
tmw mam
w^wi
iii
u * mm
iL,,„, mW>.' s *~m^M. 'mm.
W£M i HIP WiM
mm m,
WLfak
11
V/mi//V/. V77%77??i
"WHITE.
White to play and mate in four moves.
No. 620.
By C. Bosch.
i r
w-
v/M. mWk
wm m
m AMjp—
P
L5 r lil
White to play and mate in four moves.
o
No. 621.
By ^. Blumenthal.
mm mm mw
_JHI B WM hh
III SB <# *1if
k
Wm J8ji mk^Jm
JUJf wlmk wmk UPl
„■ flLJa
WHITE.
White to play and mate in three moves.
No. 622.
By H. Lehner.
...Hi
I!
Pil
til
imm IS i Is
M M
■ at.
White to play and mate in four moves.
No. 623.
By Ignace Calvi.
f.l
im
^m * wB, Is.
mm
■^11
B til
m W^m. Ww/A
mi mm mm & \
«&■ 111 ■
-'^,„„„„0^''.„„„„„
W&W WMfr. W%M>, WW/
^ 'mm
#1111 wwfr.
"Wliite to play and mate in four moves.
No. 624.
By Ignace Calvi.
BLACK. 1
mm Q WmZ^Jf^/A „liil
i
mm ° Hm
"1 ■
■//////■///■ ^^^
. if im 111
WHITE.
White to play and mate in four moves.
No. 625.
By Lamouroux.
^ nit i
H m
HIP HiP IPailP §HP
^p w.: §h| fill
^ B CSi ili
i
I „ H_ML H
'"Y///ZMi ^,^Sv?i waaam
White to play and mate in four moves.
No. 626.
By Lamouroux.
IB
mm
^^
|B , wmCwB '§
* ill ill * ill ita
§™ hhb ii« i§p
ISi IMl^
?:''
^ „ 'w/m.^. , warn
HI ^^«—J^^'- mW ^^ ! '': /
H ° isi
i i
mm
White to play and mate in four moves.
40
No. 627.
By Lamouroux.
BLACK.
f« Hi
wm r . Mm..
"mum %r« ''
4M. h WM
fa
M^mW,
WHITE.
White to play and mate in five moves.
No. 628.
By Schrufer.
Wml. ip
v mi
mm
Dil ■
WiM. WfM W
mm.
WM, i
I ife^i — mm,
. ■ B L
VWs/sy/yW. &ssy0yy/w. 'ffi/kb'001' /\
fMt mm mw, mm. $
V/////////ji
H IHi
wmz.
White to play and mate in three moves.
No. 629.
By L'Anonyme de Lille
( TH. HEELIN. )
BLACK.
*m
m mm
WW
',.,../;..,.';
! ■ I
BB~A\~W
WmJ—LWm.
m
m mr
I mm
WHITE.
White to play and mate in six moves.
No. 630.
By L'Anonyme de Lille
•'•99 %f%
m b
H7///M-/-. P- : aPV
JM Wk IH
>■•■■
x im x
Pj^ BSE?™
«# !.®; #R#
Jul . #^. , ISI
. '/)■//////&
W//////A
mm
White to play and mate in sis, moves.
No. 638.
By A.. F. op Florence.
W"
JjLJai 4M mm,
■>.
/TV/////////
A,, .s..Y.
m, mm.
m
WKk x lip 1
i iw ill ; W
%% V////S////JI V/777777/A
White to play and mate in seven moves.
No. 639.
By a Shagird,
OR NATIVE CHESS PLAYER OF INDIA.
BLACK.
fo&l fm& fmm.
SggM
Hi
W JF m wm
tM%Z%i 'iw/z/z/zZ.
HHP ^
m WfJ
H mm Hi
White to play and mate in three moves.
No. 640.
By Moonshee Waeis Ali, or India.
db
i M :
£1 1^1 ^H
1
VMM W
,: WzZm
Wz^tfz. WzzWz,
m 4M, y , mm wM
pj mi
'////zY/////. / /'//////z//. 'zt—^TTTsz.
v//'zW'z
'/•///Z/7777A
1
\ i III
■zW W
m
Either party playing first is to give checkmate in ten moves,
the last three of which are to be three successive checks with
three Pawns.
No. 641.
By Theodore Moeeis Brown.
JF-wi-
WM%- '%%%>-
m / mm,
m Mk '§§■ ■
" * m.
.„ HI
39 Hif 111 111 iuf
J f«i| llil llil -Jm£
wMk mm
/ /////M//,
WHITE.
White to play and mate in three moves.
No. 642.
By T. M. Brown.
'//////////// Wffiffift y////// / //// a ytffiffiffi
Will
WM I
%■////////. $%Wl_ .
HI iff
warn , A mtm A
^„ „ #£z5>7 4//////M.
w. mm, mm,
White to play and mate in four moves.
41
No. 643.
By T. M. Brown.
HHH *s. Iflll 'WmW it; fw
wl-S&k
m S 3
':<■■
m>
A m (m i
m.^'wM,,
wm mm mm*
w/m.
HHf "^ i'^J
mm
|a||
White to play and mate in six moves.
No. 644.
By T. M. Brown.
WM^
ill
JM iMl mm. mm.
lili
Kf
1
1 K
HI 111
m,
H ,„ mmm
m
n "pi % p
m-////m. fy////////}. vi/MMyj, sfeesa*
White to play and mate in three moves.
No. 645.
By T. M. Brown.
'WZM
'/M '//,
|j|P |f§ip ~"p
W////>. r,„„„„„™^*m Wi%m
- wk P.
WmM /% W,
WW W -
w
Jif[
warn, *. r msm. ,™ mm.'"
WM. = :
\^1mM.1u^wm
w,
Hi jb,j wiw, HH I
WHITE.
White to play and mate in five moves.
No. 646.
By T. M. Brown.
BLACK.
JJkJkJ^m~jmm
w. mm. mm.
mm wjm mk
ill HI
ftp x
y/y//.
.....w
WB. WLA WM. wm wm.
mm w»i i
Smrmr
White to play and mate in five moves.
No. 647.
By T. M. Brown.
'■?'>w//7/. .. y 'v///mf: /j
m
wmwm
m WBBk
m
j wm * Ki ■
"'-;
* mm mm. §#i
J HUC
il IBJ
White to play and mate in five moves.
No. 648.
By T. M. Brown.
iH WHm y '//////////a
#,.* &i wm,^;\
»|
als^
'£:////'///,, 'M
■-"•3 •< WW - 'J faXa? ^
I „ffii
* wk
i H
W//////?//. W/////""
■ Wm~wm mm
White to play and mate in three moves.
No. 649.
By T. M. Bkown.
%; : » W.,„,
r i« iRH HHI
js&jl; mm wqm mi
v?S?h&. ifewi #m///,A
WHITE.
White to play and mate in two moves.
No. 651.
By T. M. Brown.
«§§!
W% W ■
m
k p
^
&
JH lii
IP
Hi
I
White to mate in eight moves by discovered check with the
King's Bishop (a8) , without moving the Queen's Knight's Pawn
No. 652.
By T. M. Brown.
h!
'■y^y/vm '•%%y'W / ', vt'MW/,
ill &*■ II 1111
iHl H
H "^ ISI KM
111 iM Hil
4.
£
*-~«*
*2i§, ^/m/Ay/.,,,, tv^^-A
w, p
m mm.
ill
• MrMI
=i <=&* im nm §b
"White compels Black to mate him in four moves.
No. 653.
By Charles A. Gilberg.
wm, Wm wm,
M. §1 1 i 1 ^
'uwA 4^m w,
4m
f@1
i
A
in
ms „ W///M.
IM.I1
^ i^
mm Wm, Wm^mmk
White to play and mate in four moves.
No. 654.
By C. A.. GlLBERG.
mm wm mm mm
JBiJk^JR— mm
mm. mm.,.
I i
mJULM i ._«
fSI
8 H ISI
u
! 'lip
zm%&, »//////%,
"White to play and mate in four moves.
No. 655.
By C. A. Gilberg.
M.
m ■ v A ^
'^^A...^....WMiZ^.'W^...^...W^..
Hra. jd^^B^jBj*
mm ■? tui
"White to play and mate in four moves.
No. 656.
By C. A. Gilberg.
...m
mmk°L±\
m
wok
mm mri wmhm
vfewM m^m #iH '- ,
^S A If
HP wm^^mm^EL
WHITE.
White to play and mate in three moves.
No. 657.
By C. A. Gilberg.
BLACK.
up hp mp
w/m.
W* ^p iS i""""™'
%////////,. ..mam WZM&
■ i
111 ill
H.
AYhite to play and mate in three moves.
No. 658.
By C. A. Gilberg.
■ J >ni
>r
■iJUi
HI ifei y=y ^
Hi
*l pi
A lilt
i * flj Hi S I
■H Hi ; WM-
M\
mm// m
"White to play and mate in three moves.
42
No. 659.
By Samuel Loyd.
W////M %.
mm
I, ..§fc
ifl ii
iAj J
§n^
n iwjs ^»,
§3
i
m mJ^wJ mm
White to play and mate in four moves.
No. 660.
By S. Loyd.
"...^LpJ""..
^mJ m 'm^ m 'm
T 1
111 i
m^Jmm i
dBi^SH^ ilHB
■ H ill & ill
White to play and mate in five moves.
No. 661.
By S. Loyd.
p£^l
ill wi ■ IIP
■ &■*
wow
m wm^^^'mm'
A^.JxmM,..
mjt wm
m w - «■-
• Hi
White to play and mate in three moves
No. 662.
By S. Loyd.
m J* m J&~J!R* m ^
. mm S 41 4 4tL
F^f ^^ 1I1P r-s-*, IP
»_^ *— "- 11
a III ■ijuhl
■ i
^
«*■_«
1 H
%l My///////.
White to play and mate in four moves. (
No. 663.
By S. Loyd.
1" %,
mmoam
B.^WAw.
:•: mm wwm
mm mm mm
'mm * r
fa
m wmk
1 " la HH S IHI
White to play and mate in three moves.
No. 664.
By S. Loyd.
'A
Ww
i mm i fSf
mm A US! ^^
pip » iip iip
- mm * Hi
M . mm
» wm, mm. mm,
White to play and mate in four moves.
No. 665.
By S. Loyd.
^^ ^^ IMi
g :.
m
mm
m mm...
4 "mr'
mm. . laS «1.
^m WBl.
mm
mm..
i
wm, III
White to play and mate in four moves.
No. 666.
By S, Loyd.
WH
ill
■«r-
I "J8 i till
^p PIP
i_*4
VST
wjm:
1 nil
White to play and mate in four moves.
No. 667.
By S. Loyd.
m mm
m
i * « ill fc
m1^ wm.,f^ T mm,
«1 §■ lUf fli
■'..'///////A
m mm
White to play and mate in three moves.
No. 668.
By S. Loyd.
ifii j
■ ■
r&i'
111
^^ » mm
P
w. wW,
Hi ■ fli
*■ Ymm w// ^
m ^ss^.
^m w jm m i
H
H
^i
White to play and mate in three moves.
No. 669.
By E. B. Cook.
PUP ■
mm. ^llli
i
Y///////A
©a®
^ Wl
Hi ^^
H
dllL
.... ,,,.
H III <=* IB fill
Mi § ill u I
lip A HH HH? ww
;
White to play and mate in three moves.
No. 670.
By E. B. Cook.
SI
^ «Bujm^m
tii:
;^ $/////////.;
191 6 i
Hi
Pi
m. ifc.
^^g
_„m ,.^»
W%@% WiM ww%>
m*m m
'■mm. w&>,
mm
w mm
WHITE.
White to play and mate in three moves.
No. 671.
By E. B. Cook.
Jill «J=y mn nm
m fll-M-iyi
nn iilli*
v/i'/ 7 ', yj%>,
mi ^-> mm mm
o VMM. A '9^%- ° W<
pi
WfflM//. ■,,,,..,.,., "^^y,,,//////
WHITE.
White to play and mate in four moves.
No. 672.
By E. B. Cook.
H ^ illi
HP iMS
^ wizy/yi »M,
% iHH iHH Warn
ill III HI
PfPP a PW; A W"'% 9
11
'•77— /, ' ~ TT -
s^S^L '/.TTTA.'^— ^//mM JlW
ppir
M
'KM
jus JMiiijj^ ^ ~
\mkm-
m mm.
w, wm
White to play and dra-sv.
No. 673.
By G. N. Cheney.
H \M m
p
HHI Mill nm
^P #JSl HHl
"Hi n
ftlJJ
« es * WW?A
m. mm
WHITE.
"White to play and mate in three moves.
No. 674.
By G. N. Cheney.
WKk 'wm, i
is
W// W wt
■ ■ p
■ _J(SlJ« JB
^-W - ii
iiiii§ inm w*sm,
#p^ n«
..M„ jrm jMm
White to play and mate in three moves.
43
No. 675.
By G. N. Cheney.
WM^/
A
«■»
; ,.„J11
.^Mj^mm
y/soyy, '/////■ "a
WmSLWrn ¥9 Wm
PI ■
WHITE.
"White to play and mate in three moves.
No. 676.
By G. N. Cheney.
wm wm. i
S Si
i II S (■
JSSSP
Pi
wAA:m,...jm,... w .m.
^m"™"
§1 111 I
..HH IM1„
White to play and mate in four moves.
No. 677.
By J. A. Potter.
BLACK.
wm P
\ i
mm
m
^wJ^wmFfwmt
mm
warn tM?A
ma WWk
m^%m^:. 111.
White to play and mate in three moves.
No. 678.
By J. A. Potter.
%
i
mi p
111 mm.
®m.k
i
M
«i,=
I
fi
m*jBm
w
Va*t
MM ^
m wx :m
m
m
mm
I ©
White to play and mate in four moves.
No. 679.
By J. A. Potter.
r-
III
OP i i
f
""P^P P
"White to play and mate in four moves.
No. 680.
By J. A, Potter.
WMM.
■Mm y/////// J? / '^\ /m
mm^h^Sm ,. ., wm?A
w¥ jJ^k _
P^f 4 Ml HP i*
/ /mmZM r mttXMi,. '#'//////%'/., , / if7^w».
-fs as".
White to play and mate in five moves.
No. 681.
By G-. E. Carpenter.
*m
1
m,Jm*
mm ifiJ
if 111
■i
WHITE.
White to play and mate in four moves.
No. 682.
By G. E. Carpenter.
...Jl nni
mm
ill
WM
mm %
m Jxm
m
mwF M m
\ ki-
rn
9/, A ..I^,J
«
White to play and mate in three moves.
No. 683.
By G. E. Carpenter.
mm Mm
iP ft »
WW wW> ** WwW, Ww
ill ■*■ A
i SI H Hi
* is H m
WW> Wffit HHP UPP
: PI
m ww/t.
fill Q WW
mm " wmj±jmk.
i mm,
%
vwwz
■ 154
4m,
White to play and mate in four moves.
No. 684.
By G. E. Carpenter.
L~t"J
.ill ii-;
Wwm
m..
■ iP
■——-
j lli HHI
Y//////////.
't W&M
Hi H ■
r ill ill wW^..,.
Cj | PLJPL
H ■§
i
WHITE.
White to play and mate in three moves.
No. 685.
By G. E. Carpenter.
JH
mm
mm m
tmL
V/, '///,
■WM y
1&T
mm
m mm
White to play and mate in two moves.
Wo. 686.
By G-. E. Carpenter.
'WW?, i§
HHl
"wmF'm
m
WW
4% ,,$/////////.
m wm y mm. wsm.
I mm mm
§
4M,
i M i
White to play and mate in three moves
No. 687.
By Jacob Elson.
mm mm wm
iH fiJ u
¥M ■ If I
WB HP k iP IP
■^^//i Swhyrf. J///////& MtwW.
■:^& mmmr ^^ ^ ^^. ^^
»§^ - warn
m jk $M §M m
White to play and mate in five moves.
No. 688.
By Jacob Elson.
wm„ mm
mm
7 K^^\^ M w,J M
€k
HI.
Pi
%
H ■ULH
/ stt5777& W/////AY/.
_^"U
mm
White to play and mate in four moves.
No. 689.
By Jacob Elson.
m if 111 § iH
„Wi^-wSi & Wm. HI
mm,
i
is il
iH..
I
JKm r _JSSa
r,^ [ j^T^l' P' IIP
'■mm
WHITE.
White to play and mate in four moves.
No. 690.
By. Jacob Elson.
H W ^
...
■«W^:-wJ m -- WM *
i:4^ii
^ ^WJI JS^ ^ ^^^
i ■
HP
Jill IHi ^H
P
;'
Iflfl'^'^P HBH
II
baa
siHI ess iHp HP Hi
JillllP ^^
% 111 III fill
S%3 W//7//V/A
White to play and mate in four moves.
44
No. 691.
By Jacob Elson.
I
HIP WSF
'"if
H m "
I ^» illf
"J i ■
1 ml * HH
hi' 1b"W"W "
isy nm inn iHH
White to play and mate in three moves.
No. 693.
By Jacob Elson.
PH lip
wfflk §u is nm
ill "^ mm =■ vMa Warn
mm
■ HP ■
i
Hi
m mmF m& ^
WZ/.. tMxs/Zi.
IJJi H
ip
■v.
JI vM^A
White to play and mate in three moves.
No. 693.
By F. W. Maetindale.
mm jjj-.^P lip
ill mm mm. mm
**,,.:,,:, m Hi
41 #
;
t.^.mm mm wt
4 HlH
«L -™ 1
H
mm
White to play and mate in three moves.
No. 694.
By F. W. Maetindale.
mm
WW
mm....... s >
1!
iml IK «j
. Ml _«„_!&:
iBAJll
§ s
m
w.
"i. m
■i
M
2^5?i wm^m
1 il
■ ■ III
HP I
„,,l!l„„iii
n
H #1111
White to play and mate in four moves.
No. 696.
By F. W. Martindale.
4m
™
vm
lit
b
1 *.J
"■■■■ii
Si SHH<
White to play and mate in three moves.
No. 697.
By F. W. Martindale.
IP HP
wk„ iSlf
I §||p
g
White to play and compel Black to mate liim in
four moves.
No. 698.
By Joseph Wainwright.
mm
J
» i mm
Wm
wm., ,,wMk.
i'>&3 X S
l8p X
p
pi
■ay////.''//.
HHP x HIP US
MAM ■
mm
■T^
i
m : mi
WMte to play and mate in four moves.
No. 699.
By W. A. Shinkman.
B*il«
WW) 'tMm> w:
m mm,
wm A 'WW WmSM
MfLM m
m iESy WW.
V.
■ 4 I ^J^'
^y* WW). §j|Sf
mm, ww, * ww, IS
m m
WW. w^'m^z *to*L--Jf™.
'ww%,
mm ip IF
$Mr4. WW, WW, WW,
WHITE.
"White to play and mate in two moves.
No. 700.
By W, A. Shinkman.
W wm
mm
wfomm mm H
mm, i„»i
'WW
w,^^wW,,.^...^^B,..^..^w%.
.W///W,
m I
€qJ <■- • w,
HI ■
89!
////v//////.
WHITE.
White to play and mate in three moves.
No. 701.
By W. A. Shinkman.
A
j^W.^jM.ihB » 1
; jp*1 A*. HIP WM 'WM
% ■ JM& ?dc3l mm iffifl
. ii J3 " B I ,,JB Bi
PIP ^# HP Hi
H ■ i ■ Hi
.. ms .'
181
WHITE. .
White to play and mate in three moves
No. 702.
By W. A. Shinkman.
%
f % iiiP i^
ySsStt lllllP ^^ 1
HHy HlH ^%54%%5
"--—ail ™
<%v, .■■■ //
i
A ^~ A m
P
W i%^
BL-JBLaa^
WHITE.
White to play and mate in two moves.
No. 703.
By W. A. Shineman.
x/ wm. y mm r wm.
I mm. 4fm r wit
fit
^S ww. mm, JsSm
mm, . w-
mm w
iii..AJlB *
mt
i HP
5
Pi
iH 'WtB, Wmk*L IS
w.
White to play and mate in three moves.
No. 704.
By Denis Julien.
ill Awmf-^mah 1
W^~\TWm
WW? %WM t WW/ W;
. W//////A
1 «#H
i
. ,._
^^
HM ■ PI w
■
IB
"White to play and mate in five moves.
No. 705.
By Joseph N. Babson.
BLACK.
...m
PPf fa HI
pgg
m m
mkSLmk
—m m.
wm w
in WJF-
III m. H .. m
'ml mm, mm mm
White to play and mate in three moves.
No. 706.
By Joseph N. Babson.
'mM
mm
WW,
l "'W /M m- M wM'""""w
* mm). wLM.
mm „„„ itt_™. !M8 «j™
■////; ■////#■;
mm i mm mm mm k
i m ■ m m
* »e@ mmi. fmw. 4&^m
m mm m
§mi mm ■ wm
m
White to play and mate, sui-mate, stalemate, and
sui-8talemate ; each in two moves.
45
No. 707.
By B. M. Neill.
m 1
I m mi H
4 ■ if
PIP Pi^ P-
ill ill III,
if fi ■ ■ i .
i^ a ^p p^p mm
\~mk lu iSi
■ HP
J «n
White to play and mate in three moves.
No. 708.
By B. M. Neill.
■LJ-l
ii in in
ifil
ggH
'towZwzfr %%%VZM>. fflfi™"
m.
H
WHITE.
White to play and mate in three moves
No. 709.
By V. N. Pobtilla.
MP.tlr.IH WM
^ ^p
lJi_M
m&^ . e
J
m
JBm wm mm.
iSi
^wmf^wr^^
i til
White to play and mate in three moves.
No. 710.
By V. N. Portilla.
BLACK.
** IP
HIP Prl
'Wk HI P
™- «LJ"L
'Zy//////A,
£t^4M
Wm> eQ~\ maim a®ai
■; ■ -ate:- ; '*%/>,' ■
wwk. ^^ ^jS„
HP
White to play and mate in four moves.
No. 711.
By S. Tybbell, of Adelaide,
(SO. AUSTRALIA.)
BLACK.
m
i iBI Ill
i
mf'.'Wwf^
mm * iO
pfirt b ■
1
^ ifc .
4k mmr m m
mm. n
ll„,„„,JliJ
'mm
m
White to play and mate in two moves.
No. 712.
By S. Tybbell.
m
-mm ■
vffi 4
Htt" ■'""
is
%.
%m^x. A m
HI i
i H H ta
White to play and mate in three moves.
No. 713.
By S. Tyerell.
- ij. wm.
III +A
n ^»
til
HI 4 fill UMI
Hii
H« Wwm §81
,w//mWl
White to play and mate in five moves.
No. 714.
By H. Chaelick, of Adelaide.
™mw^^sBh^!F>
■//.■■ -/■■/' W///// y ///, oa. y/y/xs:////.
is m.^m
Wm. Hi
^ 'W?:*
i K-
^ J/^7777^/,,
'{/:■
m W
» WW,
WKk
White to play and mate in three moves.
No. 715.
By H. Charlick.
■mm. mm,
.ma. mm ^ mm wm
HP 6 lH P
White to play and mate in four moves.
No. 716.
By W. Crane, Jr., of Sydney.
■ HP HP i
fa
fel H i .
\<^jfj g Jjil
H * HLJHJ I
IP^f 1^ PW ,t. ^
W3
WW
k<
J ^H $JH1
WHITE.
White to play and mate in two moves.
No. 717.
By J. Willis, of Bogan.
/ %'//////A, mmmw, 'Wmm,, . , wwo'///,,.
m mm
: - hi * if
i Hi,
i H
^^^■^aFP^r-
SP Hi IMP ^#
MP H H Pi
White to play and mate in three moves.
No. 718.
By E. B. James, of Molong.
m wmp
mSM
mm
B mm
miM 1
m mm
up up mf up i
m 4m
wm m
w, mm
White to play and mate in three moves.
No. 719.
By D. W. Clark, op Barnaul,
( SIBERIA.)
W/4P///'/. VW////,
wm mm
M m^m Warn
4mi fUJ www. Jam
www, www. , iu mm.
m I
ill* 181
mmf^mm7Tmmf m m
m
a wm,^ www, mm
WHITE. '
"White to play and three ia-Jetrr moves.
No. 720.
By D. W. Clark.
White to play and mate in four moves.
No. 781. Ghuznee.
By Eev. H. Bolton.
*£$
i iUm ■
mm
^ "MW
Wma W//////A
wmm
|i
«m mm m
Mm mW mm..
WMuaiwr*^™' / / WMMXy '„,,,,/,,,,, ^^^' J^////0™m
White to play and mate in eleven moves.
No. 722.
By F. Hoffmann.
%-
''MW&- 'm
■P mtT
WK fm^Wm WU*
lb m
White to play and mate in three moves.
46
No. 723.
By Alphons Beck.
i nm mm,
^ 4 9K HP
« -J—jjm ma
WB i 01 1111 I
w mm,
wA w/////////.„
'//////.■//7s. ,
SI
'M*K#
■
White to play and mate in three moves.
No. 724.
By A. Bayersdorfer.
m
1
w/m, «
H il.fi.Mli
88 ^^
«# a^n Wfml r>. Wm
"White to play and mate in four moves.
No. 725.
By J. Berger.
■mm 1111 gg
WmJ^jtmf^ mm mm
mm j^v p
wm
m£//rfi — wm.....
fm WmJkJm mi.
wm^-'Mz
White to play and mate in three moves.
No. 726.
By J. Berger.
■ ■mU/HI i
mm mm.
«_. d ilit
myy/Ma
III
i
i
ri
•L-J""-
go J.®w
mm.
WHITE.
White to play and mate in four moves.
No. 727.
By F. v. Geyersstam.
HI ■r^fli
w %
WtMfr W,
vMwk k~\ §§§lfp A
m §11 » nn mm
A HP i IIP ^P
S ■ * §■ ill " MR
■ ■ II P* 1
0%%%%% wzfflyZ't $9i=^v/. ;^aBHl^ ^si
IfeM H3t "^ Wwk ...mam
White to play and mate in three moves.
No. 7S8.
By C. Bayer.
I PI HP
IP v™!!"*
_jskJk is, —
WHITE.
White to play and mate in three moves
No. 729.
By H. F. L. Meyer.
■ ■ ■
■
■ fill
w.
B^j§ ju
111
H B H 1
w
w
H^l 11
111 fci 181 4
ill a fHf -ML is
■mw
ww.
if
White to play and mate in three moves.
No. 730.
By P. Klett.
HHI 'WZW
m mm
W//4Z
i
w y/ ,,,,,,jwm>, *mwi
w. 4mMa wmi
Wm A mm i mm
m * ww, * eg
I m^
H €sl
>JI WW,
i AMI
.;«./.
JPL Wm^'mm.
mm. i
Wm JWm, y 3Ww>
w, mm,
White to play and. mate in four moves.
No. 731.
By P. Klett.
w
mm m
bI
i mm fnn wm,
t////Zt7'.
mi, 4M
ilfli
wi' ^hfwmfm
151 HP
JlSil *^* JHH ^H
«?
IIP 111/^
jm llli ^^ Jill
White to play and mate in five moves.
No. 733.
By J. v. Szirmay.
--W - ir"' 11
*^L, ^W^
iP
pp iw ^# ^ '
fit fj pi
mm .». ias8 if
%y////A ///////M-, , *~~", Kwvffl.
pl i H PI H
; . - 'TB -,
! hhb.
WHITE.
White to play and mate in four moves.
No. 733.
By C. W. op Sunbury.
a> y . vm
fe^S ?
White to play and mate ia four moves.
No. 734.
By J. A. Miles.
^wxT^imtf"***
m. je
! ill
"11 i 111
<' White to play and mate in three moves.
No. 735. The Topaz.
By G. Chocholous and J. Dobrusky.
-%- m m
w, ww,
WW,
4,
W, wW6. ww, ww.
Wi... r/ IfeU WW/,.,
m ww,
i" A wW ^kii A /j, 'wm""
mm, mm, mk ™
m
i |g| l«*
WHITE.
"White to play and mate in three moves.
No. 736. The Avalanche.
By A. Townsend.
'//-■;,■
mm ■ wm &
^fmJ^mm^MmfT'm
i\ i
mm.
W . ',.
w,
. 3 gg : -;-
m ww, = JHH
ww HI j
wm
I
HI iflp
-n-
11111 iiil
m i
v». '////,.
White to play and compel Black to mate him wa
a Book in eighteen moves.
SOLUTIONS
X signifies takes
+ „ check
K ,, King moves
Cov. „ Covers
Where Black's moves are compulsory they are in most cases omitted.
No. 1.
WHITE.
BLACI
1 E.
b7 +
2 P.
h5 +
3 E.
b6 +
4 E.
a6 +
5 P.
XP. +
6 P.
b6 +
7 E.
a8 +
8 P.
c6 +
9 P.
b7 +
10 Kt
b5 +
11 Kt
c7 +
12 B.
c5 mate.
No.
s.
1 B.
g7 +
2 E.
h8 +
3 E..
xQ. +
4 E.
f6 +
5 E.
d6 +
6 E.
d4 +
7 E.
f4 +
8 E.
f2 +
9 B.
fl +
10 E.
f4 +
11 E.
d4 +
12 E v
d6 +
13 E.
f 6 +
14 E.
f 8 +
15 E.
h8 mate.
No.
8.
1 E.
a4 +
Q. a5
2 Kt.
d8 +
E. X B.
3 E.
a7 +
Q. X E.
4 P.
mates.
No,
4.
1 E.
c8 +
2 E.
3 B.
a6±
o6# •
4 E.
a!8vidate.
47
No. 5.
■WHITE.
BLACK.
IE. h5 +
2 R. g3 +
3 Q. g7 +
4 P. f7 +
5 P. xE. mate.
No.
6.
1 E. g4
2 E. h4
3 E. h5
4 Q. f7 +
5 Q. g6
6 P. h7
7 K. g6
8 K. h6
9 P. g6
P. g7 mate.
No.
7.
1 B.xKt.+
E. xB.
2 Q.xP. +
K. x Q.
3 E. al +
4 E. a8 +
K. xE.
5 Kt. b6 +
6 E. d8 +
E. c8
7 E.xE, +
8 E. a8 mate.
No.
8.
1 Kt.xf5 +
P. x Kt. (A)
2 B. e5
K
3 Kt. c5 mate.
(A)l.
E. x Kt.
2 B. e5 +
E. X B.
3 P. X E. mate.
No.
9.
1 Kt. f5 +
b7 b6
2 P. X P +
K. a8
3 P. queens +
E. xQ.
4 E.xE. +
5 Kt. d6 mate.
No. 10.
WHITE. BLACK.
1 Q. h8 + E. X Q.
2 Kt. f6 + K. g7
3 Ktxd7 + Q. cov.
4 B. XQ. +
5 E. f8 +
6 E. xh8mate.
No. 11.
1 Kt. c6 + B. X Kt.
2 E. d8 + K
3 Q. xB. + K. X Q.
4 B. e4 + K. c5
5 Kt. d3 + Kt x Kt.
6 E. d5 + K. c6
7 E.xKt+ K. c5
8 P b4 + K. c4
9 B. d5 mate.
No.
1 Q. h7 +
2 Kt. g6 +
3 B. mates.
12.
Kt x Q.
No. 13.
IE. f 8 + K. h7
2 Q. g6 + B. X a
3 P. XB. + K. X P.
4 B. d3 mate.
No. 14.
1 Q. d6 + K. a8
2 Q. c6 B. x Q. (A)
3 E. d8 + Q. c8
4 E. xQ. mate.
(A) 2. Q. c8
3 E. d8 anything.
4 Mates.
No. 15.
1 Ktxc7 + B. x Kt.
2 Q. x B. + K. x Q.
3 B. e4 + K. b8
4 Kt. a6 mate.
No. 16.
1 Kt. e7 + K
2 Q. g8 + E. x Q.
3 Kt xB. + P. xKt.
4 E. h.4 mate.
No. 17.
1
a gs +
K. xQ.
2
Kt. e7 +
K -
3
Kt. f7 +
E. x Kt
4
E. X E. +
E. f8
5
E. X E. mate.
No.
18.
•WHITE.
BLACK.
1
2
3
4
5
3
4
Q. d7
Kt. g5
Q. h3
Q.X h6 +
E. b.7 mate.
(A) 2.
Q. f5
Q,. h3 mate.
E. g8
b8 f8 (A)
P. h6
P. xQ.
P. h6
P. XKt
No.
19.
1
2
Q. f8 +
E. xQ. +
Q. x Q.
K
3 B. h.6 + K. X B.
4 E. g8 anything.
5 E. h5 + B. X E.
6 P. g5 mate.
No. 20.
1 Q. e5 + K. a8
2 Kt. c7 +
3 KtxE. +
4 Kt. c7 +
5 Kt. a6 +
6 Q. t8+ E. X Q..
7 Kt. c7 mate.
No. 21.
1 Q. a6 E. b8
2 f3 d4 Q. c8
3 d4 b5 E. b7
4 bl c3 a b8
5 c3 e4 Q. c8
6 e4 g5 Q. b8
7 g5 e6 Q. c8
8 e6 d8 Q. b8
9 P. xE. + Q. X P.
10 Q. xQ. mate.
1
2
3
4
5
6
7
8
9
10
No. 22.
1 Q. e8 +
2 B. h6 +
3 Q. f8 +
4 E. f5 +
5 Q,. g7 mate.
No.
Kt. f5
KtxB.
Ktxh5 +
Kt. f6 +
Kt X Q. +
Q. f6
E. gl +
E. X Kt +
Q. f3 +
Q. g4 mate.
K. X
B.
P. X
E.
23.
Q.
P.
g8
h5
K.
h7
K.
h6
K —
K.
Kt.
S 4
g2
h3
No. 24.
WHITE. BLACK.
1 Q. g4 + B. X Q.
2 E Xh6 + P. x E.
3 B. f7 mate.
No. 25.
1 b3 xc5 + K. c4 +
2 B. b3 + E. X B. +
3 P. xE. + Q. x P. +
4 Q. xQ. mate.
No. 26.
1 Q. f6 + K. h.6
2 Q. h4 + K
3 Q. d4 + K. h.6
4 Q. f4 + K
5 Q. e5 + K. h6 (A)
6 E. h.5 + P. x E.
7 Q. f6 mate.
(A) 5 K. f8
6 Q xE. + K. g7
7 Q. f8 mate.
No. 27.
1 0. d6 + P. X Q.
2 E. f4 P. d5
3 Kt. e4 + P. x Kt.
4 B. e3 P. x P.
5 Kt. f2 P. x P. mate.
No. 28.
1 E. f8 + E. x E.
2 E. X E + K
3 B. g8 + K
4 B. f7 + K
5 B. g6 mate.
No. 29.
1 Q. X h7 + K. x Q.
2 E. hi + K. g8
3 E. h8 + K. f7
4 E. h7 + K. g8
5 E. g7 + K. h.8
6 Kt X g6 mate.
No. 30.
IE. e8 + Kt. c8
2 E.xKt + K. x E.
3 E. a8 + Kt. b8
4 P. f6 + K. d8
5 E. X Kt mate.
No. 31.
IE g8 +
2 B. g5 +
3 Kt. gl +
4 B. e3 +
5 B. f2 +
No. 82.
WHITE. BLACK.
IB. e8 +
2 E. x h.6 +
3 Q. e7 + K. g8
4 Q f7 +
5 Q. f8 +
6 B. g6 4-
7 Q. g8 mate.
No. 33.
1 Q. X Kt + Q. x Q.
2 b7 x g7 + K. h8
3 E. g8 + B. x E.
4 B. d4 4- K. b.7
5 E. g7 + K. h8
6 E. a7 -(- Q. cov.
7 B. x Q. mate.
No. 34.
1 P. f6 P. g3
2 Q. h6 Q. g8
3 K. g5 P. b3
4 B. b2 P. a4
5 B e5 P. a3
6 B. d6 P. b2
7 B. 18 P. queens.
8 B. g7 +
9 Mates.
No.
35.
1
2
3
4
5
b4
E.X
E.
c6
e5
c6 4-
c7 +
d7 +
e7 +
c6 mate.
K
K. x E.
K
K
No.
36.
1
2
3
4
5
B.
Q.
E.
B.
P.x
d4 +
a5 +
a3 +
c5 +
P. mate.
K
K. x Q.
K. b4
K. c4
No. 37.
1 Q. h3 + Kt. h6 (A)
2 Kt. f7 4- K. g7
3 Kt. h5 + K. g8
4 KtxKt+ K. h8
5 P. mates.
(A) IK. g7
2 Kt. e8 + E. X Kt.
3 Q. h.7 + K. f6
4 Q. h.8 + K. g5
5 P. mates.
K.
K
h4
g 4
XB.
No.
38.
Kt.
1
Kt.
dl
+
K.
c4
K.
h4
2
E.
e4
+
Kt.
cov.
Kt.
XB.
mate.
3
P.
e3
+
9
10
11
WHITE.
P. X Kt +
P.xP. +
P.XB. +
R. b4 +
R. a5 +
P. X R. mate.
BLACK.
No. 39.
R.
P.
R.
R.
B.
P.
P.
P.
P.
Kt+
P. mates
b8 +
g6 +
h5 +
h8 +
c8 +
e3 +
d4 +
c5 +
b6 +
K. X R.
No. 40.
1 R.
2 Q.
h7 + B. x R.
g8 + B. x Q.
P. g 6 B
P. mates.
No. 41.
R.
K.
Q.
R.
a7 +
f7
f6 +
a8 -f
K
B. al
B. x Q.
B. GOV.
P. mates.
Q.
Q.
Q.
R.
No. 42.
White mates.
f6 +
f7 +
e7 +
+
B. cov.
P. mates.
Compels Black to mate.
IB. h6 +
2 R. a8 +
3 Kt. d2
4 K. g2
5 Kt, ft
6 Kt. h2
7 K. hi
8 P. b7 8. P. mates.
No. 43.
Kt. f7 4-
Q. f8 +
R. X h6 +
Q.X P. +
Q. f8 +
6 R. h3 4-
7 P. mates.
B. x Kt.
B. cov.
P. xR.
B. cov.
B. cov.
Q. xR.
WHITE.
BLACK.
No.
b8 +
c8 +
b6 -4-
d2 +
Q.xKt +
Kt. c6 +
R. a4 +
P. mates.
R.
Kt.
R.
B.
44.
Kt xB.
R. X Q.
B. x Kt.
B. xR.
No. 45.
d8 f7 +
Ktxh6 +
e5 ■ f7 +
KtxB. +
Kt. g5 +
R.xKt +
Kt. f7 +
Kt. e5 +
Kt. g6 mate.
K. g S
K. h8
B. x Kt.
K
K. h8
Ktx R.
No. 46.
1
2
3
4
5
6
7
8
9
10
11
3
4
5
6
7
8
9
10
11
12
13
14
15
16
R.
B.
Q
Q.
Q.
R.
Q.
Q.
R.
Q.
a4 +
e5 +
c4 +
c7 +
d7 +
f4 +
e8 +
h8 +
f6 +
h6 +
P. mates.
No. 47.
Q.
Q.
R.
R.
Q.
Q.
Q.
Q.
B.
Q.
B.
B.
Q.
R.
P.X
e7 +
e6 +
d7 +
a7 +
d6 +
c6 +
c7 +
d7 +
h6 +
f7 +
g7 +
e5 +
f6 +
h7 +
P. +
K. c8
P. X Kt mate
KtX P.
No. 48.
1 Kt. g2
2 Kt. el
Kt. g3
Q. dl
Q.
Kt.
3
4
5
6
7 Kt. cl
8 R. f8 -f
al
e2
"WHITE. BLACK.
9 f6
f7 +
10 E.
h8 +
11 B.
h4 +
12 B.
f5 +
13 cl
d3 +
14 Q.
b2
15 K.
f2
16 E.
a7
17 E.
a6
18 K.
f3
19 B.
g4
20 K,
g2 dis. +
21 P.
c4 dis. -j-
22 Q.
cl +
23 B.
f3 +
24 Q.
g5 +
25 B.
g4 +
26 B.
h5 +
27 E.
h6 +
28 Q.
d8 +
29 P.
c5
30 Q.
a8 +
31 B.
d8 +
32 P.
mates.
No. 49.
1 Kt
h-7 +
2 e4
f6 +
3 h7
f8 anything.
4 Q.
or Kt. mates.
No. 60.
1 Q.
d4 + P.
e5
2 Q.
X P. + K.
x Q.
3 B.
e7 P.
b5 +
4 K.
e5 Q.
gl +
5 P.
d4 + Q.
X P. +
6 P.
X Q. mate.
No. 61.
1 P.
f7 P.
e6
2 P.
X Kt. aQ+ B.
d8
3 a.
XP. + K.
b8
4 Q.
d6 + B.
cov.
5 Q.
xB. +
6 a
c8 mate.
No. 5S.
1 Q.
c5 + K.
a5
2 Q.
b4 + K.
b6
3 Q.
d4 + K.
a5
4 B.
b4 + K.
a4
5 B.
el dis. + K.
b3
6 a
b4 + K.
c2
7 a
bl mate.
No. 53.
1 Kt.
d6 + K-
2 E.
h8 + B.
g8
3 E.X B. + K.
K E.
WHITE.
1
2
3
4
5
6
7
8
9
10
1
2
3
4
5
6
7
8
9
10
11
1
2
3
4
5
6
7
8
9
10
11
12
13
14
15
16
17
18
19
Q.
P.
Kt.
Kt.
Q.
c4 +
g6 +
f7 +
h6 +
g8 +
Kt. mates.
(A)<
Kt. f7 +
Q. h4 +
Q. h8 +
P. mates.
(B)4
Kt. b.6 -4-
Q. g8 +
Kt. f7 +
P. mates.
No.
Q. X f6 +
Q. g7 +
Kt. d5 +
Kt. f6 +
KtxQ. +
Kt. f6 +
KtxE. +
Kt. f6 +
Kt. d5 +
Kt. e7 mate.
BLACK.
K. h7 (A)
K. h8
K. g 8
K. h8
E. x Q.
K. h8
K. b7 (B)
K. g8
K. x Kt.
K. g8
K. h8
e. x a.
K. h7
54.
Kt.
Kt.
Kt.
E.
Kt.
No.
e6 +
c7 +
a6 +
d3 +
b4 +
55.
Q. x E.
KtxQ. +
Kt.
Kt.
Kt.
Kt.
P.
b4 +
a6 +
c7 +
e6 +
c4 mate.
No. 56.
E. x P. +
KtxP. +
B. f5 +
Q. h.6 +
B. h7 +
B. x P. +
B. hi +
B. x E. +
B. b.7 +
B. x E. +
B. h7 +
B. xKt +
B. h.7 +
B.
Q.
Q.
Q.
B.
d3 +
H7 +
h8 +
d8 +
X c3
Q.XP.+ K
K. x E.
P. X Kt.
K
K-^-
K
K
K
K
K
K
K
K
K
K
K
K
K
Q.XB. (A)
K-
BLACK.
20 Q. c6 mate.
(A) 18 B.
19 Q. xP. + K-
20 Q. f5 mate.
a4
1
2
3
4
5
6
7
8
9
10
11
12
13
14
15
16
17
Q.
Q.
Q.
Q.
Q.
Q.
Q.
Q.
Q.
B.
Q.
Q.
Q.
Q.
Q.
Q.
Q.
i2 a
No. 57.
d7 +
d6 +
g6 +
g4 +
d4 +
d2 +
el +
gl +
hi +
el +
h4 +
f2 +
d4 +
g4 +
g6 +
d6 +
d7 mate.
(A) UK. f5
g4 + &c.
No. 68.
K. e3 (A)
P. a4 + K. X P.
Kt. c3 + K. h4
Kt. a6 + Q. x Kt.
Kt. d5 + K. a4
Kt. c3 + &c.
Drawing by perpetual cl:eck.
If Black play
4 K. b5
then
Kt. c7 + winning the Q,. and
the game.
No.
E c7 +
Kt. b6 +
E. e7 +
Either P. +
69.
K. d5
K. e5
Kt. cov.
e2 X P.
Other P. mates.
No. 60.
1 e3
2 B.
3 R.
d5 +
c3 +
el
4 E. al +
5 P. mates.
Q.
E.
Q.
Q.
Q.
Q.
No.
a6 +
b7 +
c6 +
c7 +
d7 +
e7 +
K. a5
Q, cov.
Q must take B.
at c3
Q. X B.
61.
BLACK..
7 Q.
8 Q.
9 E.
10 E.
11 E.
12 Q.
13 E.
14 E.
15 Q.
16 Q.
17 E.
18 E. "
19 Q.
20 Q.
21 E.
22 Q.
23 Q.
24 Q.
25 E.
26 Q.
27 P. X
16 E.
17 Q.
18 Q.
19 E.
20 Q.
K.
K.
f7 +
f 6 +
g7 +
g5 +
h5 +
g6 +
f 2 +
f7 +
d6 +
e6 +
f8 +
c8 +
c6 +
a8 +
c6 +
b7 +
b3 +
b6 +
c4 +
63 +
P. mate.
A. 15 K.
c7 +
b6 +
c6 +
c8 +
a8 +
d8
e8 or (A)
K. a4
c8
&c. as before.
No. 62.
Q. c8 +
K. d7
P. b6
B. c5
K. x Q.
P. mates.
P. a6
P. a5
a. X Q. +
Kt.
Kt.
Q.
Kt.
Q.
P.
+
No
e3 +
c4 dis
f7 +
d2 +
e7 +
c4 + mate.
63.
P. X Kt.
Q.
Q.
B.
E,
P.
P.
No.
d8 +
a8 +
c6 + +
a8 +
a5 +
X Kt. mate.
64.
K. x Q.
Kt xP
Kt. b8 +
G. xE. +
B. xKt.
B. d2 +
Kt. c3 +
P. mates.
(A)
K. c3 +
B. X Q. +
P. mates.
No. 65.
K
K. x Q.
K. b4.
(A)(B)
3 Q. +
Q. cov. +
BLACK.
WHITE.
WHITE. BLACK.
(B)J
! B.
xP.
+
4 B. h5
4 K. c3 +
B.
cov.
5 B. +
5 B. xB. +
6 P. mates.
6 P. mates.
(B) 2. K.
g8
3 E. + K.
f7
No.
66.
.
4 B. h5 +
5 E. a8
1 Q.X d6 +
K.
e4
6 B. +
2 P. b3 +
Kt
X P.
7 P. mates.
3 E. cl +
Kt
X E.
(0) 2. K.
g7
4 E. e4 +
Kt
X E.
3 E. e8
5 Q. d3 +
Kt
x Q.
4 B. h5
6 P. X Kt +
K.
d5
5 E. a8
7 P. x Kt mate.
6 B. +
7 P. mates.
No.
67.
(D) 3. K.
g«
IP. g6 +
4 E. + K.
g7
2 P. g7 +
5 E. a8
3 Kt. g5 +
P.
X Kt
6 B. +
4 E. a4 +
P.
X E.
7 P. mates.
5 B. g6 +
6 P. g4
anything
No. 70.
7 P. g5 mate.
1 Kt. f3 + Q.
2 Q. d5 + K
X Kt.
No.
68.
3 Q. d6 + K
4 Q. e6 + K-
1 B. x P. +
P.
XB.
5 Kt. g6 4- E.
X Kt.
2 Q. g8 +
K.
X Q.
6 Q. e7 +
3 E. e8 +
K.
g7-
7 Q. e8 +
4 Kt. f5 +
K.
g6-
8 P. x E. mate.
5 Kt. f4 4-
B.
X Kt.
6 P. h5 +
K-
No. 71.
7 P. h4. mate.
1 B. x g7 +
No.
69.
2 Q. x h7 +
3 Kt. f6 +
1 E. e2
K.
f7
(A)
4 E. gl + K.
f5(A)
2 K. g5
K.
f8
(B)(0)
5 Kt. h4 +
3 B. Ii5
K.
g7 ()->'
6 Kt. h5 +
4 E, e8
7 B. d3 +
5 B. +
8 P. mates.
6 P. mates.
(A) 4. Kt.
g2
(A)l.
2 K. g5
3 B. h5
K.
K.
K.
g7
f8
g»
g7
(E)(F)
5 E. xKt +
6 Kt. h4 +
7 Kt. h5 +
4 E. +
K.
8 P. mates.
5 E. a8
6 B. +
No. 72.
7 P. mates.
1 Q. e8 +
(E)2
K.
f7
2 K. f6
3 B. h5 +
K.
f8
G)
3 Kt. iS
4 E. +
4 Kt. g5
5 E. a8
5 Q. e6 +
6 B. +
6 K. f7
7 P. mates.
7 Q. e8 +
(G)3
K.
g8
8 P. mates.
4 E. +
K.
g7
5 E. a8
No. 73.
6 B. +
1 E. +
7 P. mates.
2 Q. x E. +
(F)2.
K.
h7
3 B. e4
3 E. e8
4 E. e7
■WHITB. BLACK.
5 Kt
. c4 K. x Kt. (A)
6 E.
e5
7 E.
h5
8 E.
d5
9 P.
X P. mate.
(A) 5. P. a5
6 B.
b7
7 E.
e5
8 E.
d5
9 P.
X P. mate.
No. 74.
First Solution.
1 Q.
a8
2 E.
+
3 Q.
d8
4 Q.
f 8 +
5 Q.
f4 +
6 K.
h3
7 Q.
d6 +
8 Q.
e5 +
9 P.
mates.
Second Solution.
1 R.
e8
2 Q.
b7 +
3 Q.
e7 +
4 E.
f8 +
5 E.
f7
6 K.
f3
7 E.
f 6 +
8 E.
f5 +
9 P.
h5 +
10 P.
g4 mate.
Third Solution.
1 E.
e8
2 Q.
d7 +
3 Q.
e7 +
4 E.
f8 +
5 E.
f7
6 K.
f3
7 E.
f6 +
8 E.
f5 +
9 E.
h5
10 P.
g4 +
11 P.
h5 mate.
No. 76.
1 Q.
a7 +
2 Q.
a8
3 Q.
b7
4 B.
c3 +
5 Q.
c8
6 P.
d3 P. X B.
7 P.
XP.
8 Q.
c4
9 Q.
c6
io P.:
34
35
36
37
38
1
2
3
4
5
6
7
8
9
10
11
12
1
2
3
4
5
6
7
8
9
10
11
12
13
14
E.
P.
E.
Q.
Q.
e6 +
b6 +
d3 +
c6 +
c7 +
P. mates.
No. 92.
Q. c8 +
P. e6 +
Q. f8 +
Q. X E. +
a. f5 4-
d5 +
b3 -r
c4 +
d5 +
f5 +
f7 +
f8 mate.
Q.
a.
Q.
Q.
Q.
Q.
Q.
E.
K.
No. 93.
To mate with K. at e8.
cl d3
d3 eo
eo f7
el g2
d2 el
g2 e3
17 go
el f2
e3 f5
g5 e4 -f
f2 g2
e4 f2 +
f5 e3
e3 f 1 dis. mate.
To mate with E. at c8.
The Play must be made on the other
side of the board.
1
2
3
4
5
6
7
8
9
10
11
12
13
14
Kt.
B. d5 +
Kt. f7 +
KtxP. +
Kt. f7 +
Kt. d8"+
No. 94.
e5 + K.
P.
Kt.
Kt.
Kt.
E.
e8 queens +
f7 +
g8
B. cov.
e5 +
g6 +
hi +
E. X B. +
Q. X P. +
Q. x B. mate.
P. X Kt.
Q. B. cov.
P. X E.
B. cov.
No. 95.
a.
c5 +
f8 +
WHITE.
liL
A.0K.
3
B.
c3 dis.
+
1
Q.
d6 +
5
B.
e2 +
6
E.
XP. +
7
Q.
a6 +
8
U.
b5 +
9
E.
b6 +
10
E.
a7 +
K.
d8
11
Q.
d5 +
K.
e8 (A)
12
Q.
f7 +
13
a
c7 +
14
B.
h5 +
15
a.
e7 +
16
Q.
£7 +
17
K.
e7 dis
■+
!8
Q.
f8 +
19
B.
g6 +
Kt.
COV8.
20
P.
xKt. mate.
(A)
K.
c'8
12
Q.
c6 +
13
a.
o7 +
&.C
as
before.
No. 96.
1 E. c5 + E. x E.
2 E. b8 dis. + E. X B. +
3 P. XE. P. c5
4 Q. b4 + P. X Q. ■+■
5 P. X P. mate.
No. 97.
1 E.
2 E.
3 P.
4 P.
5 B.
6 E.
7 E.
8 Q.
9 Q.
10 Q.
11 B.
12 B.
13 B.
14 B.
15 B.
16 B.
17 B.
18 B.
19 B.
20 B.
21 Q.
22 a
23 Q.
24 Q.
25 Q.
26 a
27 d7
28 Q.
29 Q.
30 B.
31 Q.
al +
xKt.+
b5 +
b4 +
c3 +
b2 +
a2 +
dl +
X a4 +
b3 +
b2 +
X e5 +
b2 +
Xf6 +
b2 +
Xg7 +
b2 +
Xh8 +
b2 +
d4 +
b2 +
c2 +
d2 +
dl +
gl +
hl +
f6 +
h4 +
h3 +
e3 +
g3 +
K. b-3
K. f4
K. e6
WHITE.
B LACK.
32 Q.
g4 +
K. e5
33 Q.
g5 +.
34 Q.
d5 +
35 a.
d7 +
36 a
e8 +
37 a
f7 +
38 Q.
f8 +
39 P.
mates.
No.
98.
1 P.
be
2 Kt
. e7
3 K.
c4
4 K.
d5
5 K.
d6
6 K.
d7
7 Kt
c6
8 K.
c7
9 ?,
+
10 P.
queens +
11 Q.
mates.
*
No.
99.
1 Q.
eo -4-
2 f5
e7 +
3 Q.
f6 +
4 Q.
h8 +
5 Q.
c8 +
6 Q.
c6 +
7 Q.
f6 +
8 Q.
f4 +
9 Q.
e3 +
10 Q.
xP. +
11 a
c6 +
12 P.
mates.
No.
100.
1 Q.
a8 +
2 P.
d5
3 B.
el
4 Q.
b8 +
5 E.
Xd6 +
6 K.
a3
7 B.
b2
8 Kt
d7
9 Kt
xP.
10 Q.
a7 +
11 E,
b4 +
P. X E. mate
No.
101.
1 Q.
g6
2 P.
g3
3 Q.
g5 +
4 E.
c8 +
5 P.
g 4
6 Q.
e5
7 Q.
h8
8 B.
d2
9 B.
cl
10 B.
a3
WHITE. r ^.BLACK.
11 Q.
g8 +
12 K.
d5
13 E.
d8
14 K.
e4
15 E.
f8 +
16 Q.
g7 +
17 B.
el P. mates
No. 102.
With B. at al.
1 Kt
f7
2 Kt.
c7
3 Kt.
g5
4 K.
f6
5 c7
e6
6 K.
f7 dis. mate
WitliB. at hi.
1 Kt.
d7
2 Kt.
c7 K. c7
3 Kt.
b5 K. d8
4 Kt.
15 K. e8
5 K.
e7
6 Kt.
e8
7 Kt.
d6 +
8 Kt.
b7
9 K.
d6
10 K
d7
11 K.
c6
12 Kt.
d6
13 K.
c7 dis. mate.
No. 103.
With P. at c3.
1 Kt.
e3
2 E.
bl
3 K.
e2
4 E.
b5 K. c3
5 K.
dl
6 K.
d2
7 E.
g5
8 E.
f5
9 P.
f3 +
10 P.
c3 mate.
With P. at f3.
1 to 6 as before.
7 E.
i5
8 P.
c3 4-
9 'P.
f3 mate
No. 104.
1 Q.
d3 +
2 E.
c2 +
3 Q.
b3 +
4 Q.
c3 +
5 E.
cl +
6 Q.
e2 +
7 E.
al +
WH
ITE. BLACK.
"WHITE.
BLACK.
8 E.
a4 +
32 E.
a3 +
E. x E
9 Q.
c4 +
33 Q.
a6 +
10 E.
a6 +
34 P. X E. mate
11 B.
c8 +
12 E.
b6 + K. a7
No.
106.
13 Q.
c7 +
1 B.
dl
14 Q.
c6 +
2 E.
g8
15 E.
b7 +
3 E.
g 5
16 E.
b5 +
4 K.
e7
17 Q.
c7 +
5 E.
h5 +
18 B.
b7 +
6 P.+
19 B.
g2 +
7 P.+
20 Q.
c6 +
8 P. mates.
21 E.
b7 +
22 E.
c7 +
No.
107.
23 E.
c8 +
24 Q.
a8 +
1 K.
h4 *
25 E.
c6 +
2 Kt.
fo
26 Q.
a6 +
3 Kt.
e7
K. h8
27 E.
c4 +
4 E.
f8 +
28 Q.
a4 +
5 E.
f7 +
29 E.
c2 +
6 P.+
30 Q.
b3 +
7 P.+
31 Q.
c3 +
8 P. mates.
32 E.
b2 +
33 E.
f2 +
No.
108.
34 Q.
c2 +
1 B.
do
35 Q.
dl + Q. mates.
2 K.
3 Kt.
o7
c4
No. 105.
4 Kt.
eo
1 Q.
ho +
5 Kt.
d7
2 Q.
g4 +
6 P.+
3 Q.
f3 +
7 P. mates.
4 E.
g2 +
5 E.
d2 +
No.
109.
6 Kt.
e2 +
1 K.
f6
K. g8
7 Kt.
cl +
2 B.
c5
8 E.
g2 +
3 B.
f8
9 E.
e2 +
4 B.
g7
h4
10 E.
el +
5 P.
11 Q.
12 +
6 P.
h5
12 E.
hi +
7 P.
he
13 E.
b4 +
8 P.
g3
g 4
g5
g6 +
h7 mate
14 Q.
f4 +
' 9 P.
15 E.
h6 +
10 P.
16 Q.
f6 +
11 P.
17 Q.
h8 +
12 P.
18 E.
f6 +
19 Q.
f8 +
No.
110.
20 E.
d6 +
21 Q.
d8 +
1 Kt.
e6
22 E.
b6 +
2 B.
f7
23 Q.
c7 +
3 B.
e8
K. h8
24 Q.
c6 +
4 P.
g6
25 E.
b7 +
5 P.
g5
26 E.
b3 +
6 P.
g 4
27 Q.
c7 + K. a8
7 P.
g3
28 Q.
c8 +
8 K.
hi
29 Q.
b8 +
9 K.
h2
30 Q.
t>7 +
10 K.
h3
31 Q.
b6 +
11 B.
f7 +
WHITE
BLACK.
WHITE. BLACK.
12 P.
+
,'A) 1 K.
h7
13 P.
+
2
P.
g 4 * K.
g«
14 P.
+
3
K
e7 K.
g7
15 P.
mates.
4
P.
f4 &c. as befo:
(B) 4 K.
'6.
h6
No.
Ill
.
5
K
f7
With P
. at e4.
6
7
P.
P.
g5
f6
1 E.
eo
8
P.
f7
2 E.
e7
9
P.
e4 &c. as befoi
■e.
3 Kt
. d4
(C) 4 K.
h7
4 Kt
. e6
5
K.
f7 K.
e6
5 K.
g6
6
P.
e4 K.
h7
6 K.
ho
7
P.
g5 &c. as before.
7 K.
h6
(D) 5 K.
h7
8 E.
e8 +
6
P.
go K.
8' 7
9 Kt
g7
7
P.
e4 K.
gH
10 P.
g 5 +
8
P.
f6 K.
h7
11 P.
g6 +
9
K.
f7 &c.
12 P.
e5 mate.
6
K.
(E) 5 K.
f7
h8
WithP
. at
g4.
7
8
P.
P.
g5
n4
1 K.
e6 +
K.
e8
9
P.
e5
2 E.
d5
10
P.
e6
3 Kt
d4
K.
g8
11
P.
e7
4 E.
h5
K.
g7
12
K.
g'G
fe
5 Kt
c6
K.
g6 or g8
13
P.
6 Kt
e7 +
14
K.
he
7 E.
hi
15
P.
g6
ho
8 E.
h7
10
K.
9 K.
d6
.17
K.
g5
h6
10 E.
hi
18
K.
11 E.
h8 +■
19
P.
f7 +
12 E.
g8
20
P.
g7 mate.
13 P.
e5 +
(F)
If Black play K
to h8
14 P.
e6 +
h.7 White plays K. to e
7 &c.
15 P.
go mate.
No. 113.
No.
US
1
E.
b6 + K.
d8 (A)
2
a.
e8 + K. x
Q.
1 K.
to
K.
g8(A)
3
E.
Xg6 K.
X e7 + K. X
f8
2 K.
e7
K.
g7
4
B.
B.
3 P.
g 4
K.
g6
5
E.
e6 + K. —
4 P.
f4
K.
87(B)(0)
6
E.
e8 mate.
5 P.
15
K.
h6 (D) (E)
(A) 1 K.
X f7 + K.
f8
6 K.
f6
K.
h7
2
B.
f'7
7 P.
g&
K.
g»
3
B.
X g6 + K. g8
e8 + Kt. cov.
8 K.
e7
K.
g7(F)
4
a
9 B.
e4
K.
g8
5
Q.
X Kt. mate.
10 p.
16
K.
h7
11 K.
f7
No. 114.
12 P.
eo
1
Kt
b3 +
13 P.
eG
2
Kt
c5 +
14 K.
g 6
3
B.
c3 B.
e5 +
15 P.
e7
4
E.
cov. + B. x
B.
16 K.
h5
K.
h7
5
P.
b4 + B. X
P.
17 P. +
K.
g8
6
P.
X B. mate.
18 K.
g5
19 K.
he
No. 115.
20 P. +
1
a
c7 B. x
Kt.
21 P. i
nates.
2
B.
ti3 + P. cov.
P.XP.f^6
d7 +
5
c8
b6 +
6
E.
a8 +
Kt. x E.
7
Kt.
c8 mate.
No.
119.
1
Q.
a4 +
2
Kt.
d7 +
3
Kt.
e5 +
4
E.
b.5 +
.5
E.
c5 +
6
E.
c6 +
7
E.
d6 +
8
R.
d7 +
9
E.
e7 + +
10
Kt.
g6 +
Q. x Kt.
11
Q.
e8 +
Q. X Q. +
12
E. x Q. + +
K. x E.
13
P.
f7 mate.
No.
120.
1
R.
e8 +
E. cov. dis. +
2
E.
f 3 dis X
Kt. e5
3
E. X
E. +
Q. x E.
4
B.x
Kt +
Q. cov.
5
E.
f8 +
B. cov.
6
E.x
B. mate.
No.
121.
WHITE.
BLACK.
1
Kt.
d7 +
K. e6
2
Q.
b5 +
K. x Q.
3
Kt.
d4 -f
K. x E.
4
Kt.
CO +
K. a3
5
Kt.
b5 +
K. b2
(i
Kt.
d3 +
K. x B.
7
Kt.
a3 mate.
No.
122.
1
Kt.
a2 +
Kt x Kt,
o
B.
a3
B. c2
3
B.x
Kt. dis -1-
B. cov.
4
B.x
B.
K. dl
5
B.x
b2
E. xE.
6
B.
e4 dis. + E. cov.
7
E.x
E. mate.
9
10
11
12
3
4
5
fi
7
8
9
10
11
12
13
14
No. 123.
IP. b7 +
2 B. f4 +
K.
1.8
E. cov.
K.
B.
K.
K.
K.
o-fi
eo
L7
g8
f8
B. xh2
B. e5
K. e8
K. d8
B. X E. mate.
P.
P.
P.
P.
P.
P.
P.
P.
ho
h.4
g« +
h.3
h2
g2
P. queens.
No. 124.
Q. f5 +
c5 x d7
P. g4 mate.
(A) 1 K.
Q. Xd7+ K.
Q. e6 mate.
No. 126.
K. x Q. (A)
1 Kt. e4
K.
e6
g 5
f3
d4
id
K.
f4
e4
g6
g7
f6
f5
e.j
i'3
d4
fo +
g7
U
g 6
f6
f4
fo
g4 +
g3 mate.
K.
K.
K.
K.
K.
K.
K.
K.
K.
K.
K.
K.
K.
e7
f6
fo
eo
15
g4
h4
g 4
hi
prl
h.3
h2
hi
h2
hi
No. 126.
IE. d3 +
2 Kt. e6
3 Ktxb2 +
K. X E. (A)
E. x Kt.
K. c2
No. 149.
This Problem and " The Hero " (No. 141) ,
as well as Ghuznee, appear never to have
been correctly solved. They were published
by Lewis, in 1844, without Solutions, and
copied into Le Palamede, and afterwards by
Alexandre in his " Beauties of Chess," but
the Solutions of the three Problems in both
these works are incorrect.
1
2
3
4
5
6
7
8
9
10
11
12
13
14
15
"WHITE.
E. xP. +
E.xKt +
E. g7 +
E. X d7 +
E. g7 +
E. X Kt +
E. g7 +
E. c7 +
Q. xE. +
E. g7 +
E. g4 +
Q. b7 +
a. X B. +
Q. XE. +
Kt. f8 mate.
BLACK.
Q.
d8
B. cov.
E. cov.
Q.XQ.
1 E.
2 B.
Kt.
Kt.
Kt.
KtX
Kt.
8 KtX
9 B.
10 Kt.
11 KtX
12 Kt.
13 P.
14 Kt.
15 P. x
16 E.
17 Kt.
No. 150.
a7 +
a4 +
e6 +
c7 +
a6 +
b6 +
c7 +
B. +
d7 +
c7 +
Kt +
e8 +
g7 +
f8
B. (a Kt.) +
h7 +
g6 mate.
P. XE.
Q. c3
d3 c5 +
Q. X P. +
Kt. d3 +
P. b3 +
P. bl +
Kt. mates.
No.
+
x P.
a6
161.
P.
K
K
K. x Q.
K. x a4
K. a5
K
No. 152.
E.xKt +
Q.
Q.
K.
Q.
E.
c3 +
c7 +
f5 +
f4
g5 +
K.
K.
K.
K.
E.
K.
X E.
e7
f6
g 6
g7
h7
WHITE. BLACK.
7 Q.
h2 E. x E.
8 B.:
< E. + K
9 Q.
h.6 mate.
No. 153.
1 Q.
al
2 B.
e4
3 E.
d2
4 Q.
dl
5 E.
a2
6 B.
bl
7 P.
b4
8 a.
d7 +
9 P.
b6
10 K.
b5
11 K.
c4
12 K.
b3
13 E.
c2
14 K.
a2
15 K.
al
16 Q.
c8 +
17 E.
a2 +
18 E.
a6 +
19 Q.
c6 +
20 E.
b6 +'
21 Q.
c5 +
22 E.
b4 +
23 E.
e4 +
24 Q.
b5 +
25 a
b2 + P. x Q. mate
No. 154.
1 Kt.
d5 Kt x B.
2 Ktxb6 + K
3 Q.
a7 + K
4 Kt.
a8 anything.
5 Mates.
(A) 1 P. x Kt.
2 B.
d6 dis + Kt. b8
3 B.
f5 + E. d7
4 Q. x Kt. mate.
No. 155.
1 P.
b7 + B. x P.
2 E.
d7 KtxKt.
3 R. xB. Kt. d8
4 E.
bl dis + Kt. b7
5 E. X Kt. anything.
6 E.
bl dis mate.
No. 156.
1 Kt.
d6 + K. d3
2 P.
a3 E. d8
3 P.
g8 queens E. x Q. (A)
4 Kt.
cl + K
5 K
d2 P. c4
6 a3 X b4 anything.
7 Mates.
WHITE.
BLACK.
(A)
3 E. x Kt.
4
Kt. cl +
K. d4
5
Kt. e2 +
K. d3
6
Q. b3 mate.
No.
157.
1
Q. e5 +
K.xE. (A)
2
Kt. d6 +
K
3
Kt. d3 +
K
4
Kt. e4
B. x a2
5
Q. d4 +
B. cov.
6
Q. xB. +
K
7
Q. b4 +
K
8
Kt. c6 mate.
(A) If lK.xQ.
2
Kt. d3 +
K
3
E. d4 mate
No.
168.
1
KtxB. +
K. Ii7
2
Kt. f8 4-
K. g8
3
Kt. c!7 +
K. b7
4
E.x B. +
K.xE.
5
KtxE.
P. xE.
6
Ktxb3
P. h4
7
Kt. d4
P. h.3
8
Kt. c6
P. h2
9
Kt. eo
P. queens.
10
Kt x f 7 mate.
No.
169.
1
KtxE. +
E. x Kt.
2
B. a7 +
3
Q. f3 +
4
K. b6
5
K. b5 +
6
K. c5
7
K. c4 +
8
K. d4
9
K. e4 +
10
B. e3
P. X B.
11
K.XP.
12
Q. dl mate
No.
160.
1
Q. e7 +
K. d5
2
Q. d8 -4-
K. c5
3
0. a5 +
P. b5
4
E. c4 +
K. d5
5
E. c5 +
K.xE.
6
Q. b4 +
K. d5
7
Kt. e7 +
K. e5
8
Q. X f4 +
K. XQ.
9
Kt. g6 mate.
No.
161.
1
Kt. e7 +
K. e4
2
Kt. f5
P. X P.
3
K. h4
E. e6
4
Q. g2 +
K. X Kt.
5
Q. g4 mate.
No.
16S
WHITE.
BLACK.
1
Kt. c6 dis. + K.
b5
2
E. a6
E.
XE.
3
Q. e2 +
K.
xKt.
4
Q. xB. +
E.
d7
5
Q. e4 +
K.
b5
6
Q. c4 +
K.
c6
7
Q. xE. +
K.
d5
8
Q. c4 +
K.
c6
9
Q. e4 +
K.
b5
10
Q. a4 mate
No.
163
1
Q. g6
B.
XB.
2
Q. xP. +
B.
cov.
3
Q. f8 +
B.
cov.
4
Kt. f7 +
K.
h7
5
Kt. go +
K.
h8
6
Q. h6 +
B.
cov.
7
Q. xB. mate.
No.
164.
I
Q. a4 +
K.
XKt
2
P. c5 +
K.
e6
3
Q. g 4 +
P.
f5
4
g5xP. +
K.
f7
5
P. XP. +
K.
g8
6
Q. e6 +
E.
x Q.
7
P. f7 +
K.
h8
8
Kt. f6
E.
X Kt.
9
P.xE.(aQ)+
E.
cov.
10
Q. x E. mate.
No.
166
.
1
Q. d6 +
Q.
eo
2
Q. b4 +
Q.
e4
3
Q. b8 +
Q.
eo
4
E. c7
Q.
d6
5
Q. b2
B.
g7
6
Q. XB.
Q.
c5
7
Q. b2
Q.
d5
8
E. c4 +
Q.
XE.
9
Q. b8 +
Q.
c7
10
Q. X Q. mate
No.
166
.
1
Q. f3 +
K.
a7
2
Kt. c6 +
3
Kt. d8 +
4
P. XP. +
K.
XP.
5
Q. c6 +
6
Q. c5 +
1
Q. d5 +
8
Q. d4 +
9
Q. e4 +
10
Q. e3 +
11
Q. f3 +
12
Q. 12 +
13
Q. xP. +
"WHITE.
BLACK.
14
15
16
17
18
19
20
21
22
23
24
23
24
1
2
3
4
5
6
7
8
9
10
11
12
13
14
1
2
3
4
5
6
7
8
9
10
11
12
13
14
15
16
Q.
Q.
Q.
Q.
Q.
Q.
Q.
Q.
Q.
Q.
Q.
Q.
Q.
f2 +
f3 +
e3 +
e4 +
d4 +
d5 +
c5 +
Xc8
Xa6 -f
c6 +
X B. mate.
(A) 22.
c6 +
mates.
P.
B.
E.
B.
f6
eov.
cov.
COT.
(A)
No.
e4 +
f6 +
h6 +
g3 +
KtxB. +
E. f6 +
E. Xb6 +
E. f6 +
E. c6 +
Kt.
E.
E.
Kt.
167.
K.
f5
g3 +
e2 +
g6 +
E. X c5 +
P. mates.
Kt.
Kt.
Q.
No.
Q. Xd6
KtxQ.
Kt. c7
Kt. c5
P. e4
P. XE.
+
mate
168.
Q. d5
B. x Q.
B. x Kt.
Kt. d5
E. xP.
No. 169.
KtXa6 +
KtxE. +
Kt. a6 +
Kt. c5 +
Q. a7 +
KtxKt +
B. x f4 +
B. £5 +
B. g5 +
K. X Q.
Kt.
Kt.
P.
B.
Kt.
Kt.
Kt.
d6 +
d7 +
h7 +
e6 +
f7 +
e5 +
g6 mate.
E. x P.
No. 170.
P. queens + K —
P.Q. h8 + K—
Q. g7 + K—
WHITE. BLACK.
4
Q. e 5 + K
5
P.Q. g8 + K
6
Q. h8 4- B. cov.
7
P.Q. X B. + K
8
Q. g7 + Kt. cov.
9
Q. xKt. + K
10
Q. e4 + K
11
Q. e5 + Q. cov.
12
P.Q. x Q. + K
13
P.Q. f8 + K. g6
14
P.Q. e8 + K. h7
15
P.Q. f7 + K
16
Q. is + K
17
P.Q XE. + K
18
P.Q. e5 + K
19
P.Q. e4 + K
20
Q. g7 + K
21
P.Q. f5 + B. cov.
22
P.Q. 1l3 + B. cov.
23
P.Q.xh2 P.xP.Q
24
P. g4 mate
No. 171.
]
Kt. £4 P.xKt.
2
R. gl K. e8
3
E. g8 + B. cov.
4
Q. d6 K. f7
5
E. g7 + B. X E. (A
6
Q. e7 4-
7
Mates.
(A) 5 K. e8
6
E. e7 + B. X E.
7
Q. x B. mate.
No. 173.
1
KtXg7+ K. f6
2
E. f8 + K. g5
3
Q. f4 + K. x Q.
4
E. X B. + Kt X E.
5
Kt. e6 mate.
No. 173.
1
Q. d3 + K. g2
2
Q. g3 + K. fl
3
Q. gl + K. e2
4
Q. dl + K. e3
5
Q. d3 + K. x Q.
6
P. X Kt (aE.) anything.
White is stalemated.
No. 174.
1
Kt. a6 + K
2
Kt. d6 + K. x Kt.
3
Q. xKt+ P XQ.
4
E. h4 anything.
5
B. mates.
No. 175.
1
B. xP. dis. + Ktx Q.
2
B. e8 dis. + Q. X E.
WHITE.
P.
BLACK.
g5 + K. g6
4 P. f8 (a Kt.) + + and mate.
No. 176.
1 Kt. f6 + KtxKt.
2 KtxKt+ K. Ii8
3 Q. h4 E. g8
4 B. f8 E. g7
5 Q. h6 E. X B.
6 Ktxh7 P. f5
7 Kt. f6 + E. h7
8 Q. x E. mate.
No. 177.
1 P. f6 Q. gl +
2 E. xQ. P. xQ.
3 P. f7 + E. cov.
4 P. f8 (a Q.) mate.
No. 178.
1 E.xKt. E. X E. (A)
2 Q. f5 K. g8
3 B. b3 + a7 f7
4 Q. xE. + E. X Q.
5 E. e8 mate.
(A) Kt. e7
2 E. xKt. g8 X f8
3 Q. f5 E. X Q.
4 E. + E. cov.
5 E. x E. mate.
No. 179.
1 E.X f7 B. e6 (A)(B)
2 B. X B. Q. X Kt.
3 E. X E. + K. L.7
4 E. h8 + K. x E.
5 Q,. e8 + K. h7
6 B. g8 + K. h8
7 B. f7 +
8 Q. mates.
(A) E. x E.
2 Q. XE. + K. Ii8
3 (J. g6 &c.
(B) Q. e8
2 E. e7 + Q. f7
3 Q. xQ. + E. X Q.
4 E. mates.
No. 180.
1 B. h6 K. f7
2 Q. g7 + K. e6
3 0. g8 + K. d6 (A)
4 P. + K. X P.
5 P. + Q. X P. *
6 B. f 8 + and wins the Q,
*K. X P.
6 B. d2 + and wins the Q.
WHITE.
BLACK.
(A)3K. e7
4 B. g5 + K. d6
5 Q. f8 + K. anywhere.
6 Q. orB. mates.
No. 181.
1 Q. g5 + K. X Kt.
2 Q. XB. + K. XQ.
3 B. xb3 mate.
No. 182.
1 Q. h5 + K. e6 (A)
2 Q. e5 + -K. X Q.
3 P. XE. + K. e6
4 P. xE. (aKt.)mate.
(A) 1 Kt. cov.
2 Q. xKt+ K. e6
3 P. x R. mate.
No.
183.
Kt.
1 Q. d6 Kt. e6
2 E. X c7 Q. e5
3 E. c8 + K. X E.
4 Kt X a7 mate.
No. 184.
1 Kt. e6 + Q. X Kt.
2 E. X Kt. Q. X E.
3 Q. a5 + K. d4
4 Q. c3 mate.
No. 185.
1 Q. b3 B. d5
2 Q, dl P. f4
3 Q. h5 Kt. d7
4 Q. e5 + Kt.xQ.
5 Kt. c5 mate.
No. 186.
1 P. g4 + B. X P.
2 bl c3 B. X Kt.
3 B. d5 K. e5
4 Q. f 6 + K. X B.
5 Kt. mates.
No. 187.
1 Q. g5 E. f8
2 E. c5 E. f5
3 Q. f4 + E. or B. x Q.
4 E, or Kt. mates.
No. 188.
1 B. d4
2 B. e3
3 Kt. c4
4 B. a4
5 B. mates.
cl el
E. x B.
P. X Kt.
anything.
No. 221.
WHITE. BLACK.
1 E. cl Q.xKt+(A)(B)
2 K. dl anything.
3 E. or B. mates.
(A) I E. X E.
2 K. dl &c.
(B) 1 E. g8
2 K. bl &o.
No. 222.
1 Kt. d2 B. X Kt. +
2 Kt. e4 B. moves.
3 E. mates.
1 Q.
2 B. d4
3 Mates.
No. 223.
g5 Q. xKt. +
anything.
No. 224.
1 Q. a2 . E. XKt.
1 P. c4 anything.
3 Mates.
No. 225.
1 Q. b4 E. b2
2 B. b3 anything.
3 Q. mates.
No. 226.
1 Q. al+ P. d4 (A)
2 Q. b2 either E. +
3 ft. xE. mate.
(A) c2 b2
2 Q. X E.+ E. x Q.
or anything.
3 Kt. mates.
1 Q.
No. 227.
b2
No. 228.
1 E. d6 either E. xE.
2 B. d5 or e6 anything.
3 Mates.
No. 229.
1 Kt. c5 " P. X Kt.
2 E. e5 anything.
3 E.orB. mates.
No. 230.
1 B. d5 Q. gl
2 B. g3 Q. a7
3 B. f2 anything.
4 Mates.
No. 231.
1 Q. f8 Q. cl +
2 B. c2 dis. + Q. X E.
3 Q. f5 + Kt X ft.
4 B. mates.
No. 232.
WHITE. BLACK.
1 Kt. f5 Kt X e5 dis. +
2 P. c4 + Kt X d dis. +
3 Kt. e3 + KtxKt.dis.+
4 P. c4 + Kt X P. dis. +
5 Q. e4 mate. (A)
(A) 4 P. X P. en pass.
dis. +
5 Q. d3 mate.
No. 233.
1 E. x P. + P. X E.
2 E. c6 Q. X E. (A)
3 Kt. g8 anything.
4 P. or Kt. mates.
(A) 2 B. g6
3 Kt. f 7 + If B. x Kt.
4 P. mates.
If Kt.xKt.
4 E. X B. mate.
No. 234.
1 Kt. c5 + K. d6
2 B. e7 -f K. x E.
3 E. g5 anything.
4 f 5 moves, mate.
235.
1 Kt. d2 + K. e5
2 Kt. c6 + K. d6
3 R. dl
4 Mates.
236.
1 E. g6 B.xKt+(A)(B)
2 K. d2 + E.xQ.*
3 E. e6 +
4 One of the Bishops mates.
*IfK. f4ore5
E. + and B. mates.
(A) IB. d4 +
2 K. d2 B. c3 +
3 K. e3
4 Mates.
(B) IB. e3 +
2 B. cov. B. d4 +
3 K. d2
4 Mates.
No. 237.
1 P. d4+ K. x P. (A)
2 Q. a7 + P. cov.
3 Q. al Q. X Kt.
4 Q. gl Q. X Q. *
5 Kt. mates.
* 4 If any other move,
5 E. e4 mate.
(A) 1 K. b6
2 Q. b7 + K
3 Q. Xb4 + K. X Q-
4 E. a3 anything.
5 P. mates.
No. 238.
WHITE. BLACK.
1 Kt. f5 + K. do
2 Kt. e4 E. X d4 +
3 K. e3 E. d3 +
4 K. f4 B. d6 -j- (A)
5 KtxB. mate.
(A) 4 Q. X Kt.+
5 P. xQ. mate.
1 Kt.
2 Kt. d3 +
3 P. x Kt+
4 B. c2
No. 239.
f5 + K. c5
K. d5
K. xE.
anything.
5 Mates by discovery.
No. 240.
1 E. c7 Kt.xE.
2 B. f3 + P. X B.
3 P. x Kt. E. x E. (A)
4 P. X E. (a Kt) anything.
5 P. mates.
(A) 3 P. d6
4 P. c8 queens E. X Q. (B)
5 E. X P. mate.
(B) 4 R. X E.
5 P. mates.
No. 241.
1
2
3
4
5
Q. a2 R. f5
Q. d2 E. e6
Q. d5 + Kt X Q.
KtxB. +
Kt. b.5 mate.
No. 242.
1
2
2
4
5
Kt. d4 a3 c4
K. g3 Kt. dl
Q. x Kt. P. X Q.
K. f4 anything.
K. moves, mate.
No. 243.
1 P. h4 P. X Kt.
2 Kt. f 1 Q. X B.
3 •■ E. c4 P. b5
4 B. X f3 + P. X E. +
5 B. b7 + P. X R.
6 P. g3 mate.
No. 244.
1 B. d5 + K. X B.
2 Kt. c7 + K. e4
3 0. X f4 + K. X Q.
4 Kt. d5 + K. e4
5 Kt. g5 + K. X Kt.
6 P. e4 mate.
No. 245.
1 E. e4 E. X B.
2 KtXf6 + K. e6
3 Kt. d4 + K. e7
4 B. xKt+ K. X B.
50
WHITE. BLACK.
5 E. e5 anything.
6 Mates.
No. 246.
1 E. b8 either E. X Q. (A)
2 Kt. f5 + K. d8
3 B. b7 dis. mate.
(A) 1 B. d7
2 B. X B. P. X Kt.
3 E. e8 mate.
If 2E. X Q. orE. a8
2 Kt. f5 mate.
No. 247.
1 B. ho B. f5 (A)
2 B. X f3 B. X Q.
3 B. d5 + K. d3
4 Kt. cl mate.
(A) 1 P. e2
2 P. b3 + P. X P.
3 Q. x e2 + P. X Q.
4 B. X P. mate.
No. 248.
1 Kt. e7 E. x Kt. (A)
2 K. f2 Kt. e5
3 K. e2 Q. X P.
4 KtxB. + E. xKt.
5 E. d5 mate.
(A) 1 Kt. +
orKt. orB.xKt.
2 K. e2 and mates in two more
moves.
If 1 B. a5
2 KtxP. +
3 E. mates.
If 1 B. x B. or
Ktx P. or
B. a5 or
E. e7
2 P. +
3 B. mates.
No. 249.
IB. cl + E. e3
2 E. x P. E. X E.
3 Q. h8 If Black take
either Q. or Kt. he is mated at once.
3 anything else.
4 Kt. g6 + E. X Kt.
5 Q. mates.
No.
250.
1
2
3
4
5
Kt. f5
Q. e5
B. b5 +
Kt. e6 +
B. mates.
KtxKt.
P. X Q.
K. d4
K. XP.
No.
251.
1
2
3
el
B.
B.
d3 +
e4
f 4 mate
B. x Kt.
B.x B.
(A)
WHITE. BLACK.
If 2 E. X B.
or E. X Kt.
3 Q. x d5 mate
If 2 K. X B.
3 Q. f4 mate.
(A) 1 E. X Kt.
2 B. f4 + anything.
3 Mates.
No. 252.
1 Q. c3 E. f8
2 Q. h8 E. f7
3 Q. g7 anything.
4 Kt. or Q. mates.
No. 253.
1 E. d4 + K. x E. (A)
2 P. g4
If he play P. X E. he cannot win
in four !
2 E. x P.
3 Kt. g3 E. x Kt.
or anything.
4 Q. matea.
(A) 1 K. fo
2 Q. f7 + K. g5
3 Q. f6 + K. X Kt.
4 E. X E. mate.
No. 254.
1 Kt. d3 + P. X Kt.
2 B. g4 Kt X B. (A)
3 E. f5 +
4 Q. mates.
(A) 2 Kt. e4 (B)
3 Q. f5 + K
4 Q. co mate.
(B) 2 B. X E. (C)
3 Q. c3 + K
4 Q. mates.
(C) 2 K. x E. (D)
3 Q. c7 mate.
(D) 2 B. X Kt.
3 Q. c7 +
4 Q. e5 mate.
No. 255.
IE. e4 + K. x E.
2 E. e6 -4- K. fo
3 B. c8 anyihing.
4 Mates.
No. 256.
1 B. d2 P. c5 (A)
2 B. g5 anything.
3 Mates.
(A) 1 P. e6 (B)
2 B. b4 + anything.
3 Mates.
(B) 1 K. x P
2 E.xP. P. e6dis.+(C)
3 B. b4 mate.
WHITE. BLACK.
( C) 2 K
3 B. mates.
No. 257.
1 Q. f8 Kt x Q-
2 P. g 6 P. fe
3 P. g7 anything.
4 Mates.
No. 268.
1 Q. e2 anything.
2 Mates.
No. 259.
1 E. g5 Q. e4 (A)
2 E. e5 Q.xE.orK-
3 Q.xhS + K
4 Q. mates
(A) 1 Q. a6
2 Q. d5 + anything.
3 G. dl -4- anything.
4 Mates.
No
1 E. e5
2 Kt. e4
3 Q. or Kt. mates.
260.
P. xE.
anything.
No. 261.
1 Kt. e7 E. g8
2 Kt. a8* anything.
3 Q, or Kt. mates.
*The object of this move is far
from being obvious.
No.
262.
1
2
3
3
H.
Q.
B.
Q.
b4
e4 +
c6 mate.
(A)
c6 mate.
P. X E.
K. x Q. (A)
K. x Kt.
or K. co
No.
263.
1
2
3
4
Kt
Q.
Kt.
E.
Xf4
a 5
d6 +
co mate.
P. X Kt.
P. X Q.
Kt x Kt.
No. 264.
1 G. c4 B. g.3 (A)
2 B. b6 K -* .
3 B. c7 + anything.
4 Q. cl or Kt. f 1 mate.
(*) 2 B. el
3 Q. cl + B.cov.orK.xP.
4 Q. x B.
or Kt. c5 mate.
(A) 1 K
2 Q. cl +
3 Kt. f3 +
4 Q. c4 mate.
No. 265.
•WHITE. BLACK.
1 K. C2 B. X E. + (A)
2 K. c3 B. X Q. (B)
3 B. mates.
(A) 1 E. X E.
2 Q. b3 + K. x P.
3 Kt X P. mate.
(B) 1 K. X E.
2 K. c3 di8. 4-
3 Q. mates.
No.
1 Q. b6
2 Q. x c7
3 Kt e8
4 Mates.
266.
E. c6
E. X Q. (A)
(A) 2 P. d4
3 Q. X c6
4 Q. mates.
No. 267.
1 Kt. fl + K. e2
2 Q. d6 If Black take Q.
with either P. then Kt. e6, and
mates next move.
2 E. d3 (A)
3 Kt. e6 E. f8
4 Q. x E. + B. x Q.
5 Kt. d4 mate.
(A) 2 B. d3
3 Q. e5 + B. cov.
4 Q. x B. + E. cot.
5 Q. x E. mate.
No. 268.
dl E. hi (A)
t B.
2 R. d4 + B. c4
3 Q. h3 anything.
4 Mates.
(A) 1 E. x Q.
2 P. c3 dis. + B. cov.
3 E. d4 + Q. cov.
4 E. X Q. mate.
No. 269.
1 Q. X Kt. a8 X Q. (A)
2 E. X a4 anything.
3 KtorE. mates.
(A) 1 e8 X Q.
2 E. b6
3 Kt. mates.
No. 270.
1 Q. Xa7 + P. b6
2 Q. g7 B. x Q.
3 K, g4 anything.
4 B. or Kt. mates.
No. 271.
1 Kt x c2 P. e5 (A)
2 E. X P. + anything.
3 Q. or Kt. mates.
WHITE. BLACK.
(A) 1 P.xKt. (B)
2 Q. x Kt + K. X E.
3 Q. d5 mate.
(B) 1 E. X Kt.
2 E. f4 +
3 Q. mates.
No. 272.
1 E. x e4 P. c5 (A)
2 E. c4
3 Mates.
anything.
A 1 P. x E. dis. -f
anything.
2 B. x E.
3 Mates.
No. 273.
1 d5 e3 K. xE. (A)
2 Q. b4 + K. x E. or Q.
3 Kt. mates.
A 1 E. x B.
2 Q. c3
3 Q. mates.
No. 274.
1 B. c7 B. x B. (A) '
2 Q. h8 anything.
3 Q. mates.
A 1 P. f4 (B)
2 Q. e5 + K
3 Kt. mates.
(B) 1 Q. f6
2 P. e4&c.
Note. If White begins
1 P. b5 he will be defeated by
1 B. a5
No. 275.
1 B. e2 B. x B.
2 Q. c5 P.orKtxQ. (A)
3 Kt. d4 dis. + K
4 Kt. mates.
(A) 2 Q. d8 4-
3 Kt. d2 dis. + K
4 Q. mates.
No. 276.
1 Q. b6 B. orE.xQ. (A)
2 E. Xd7 dis. 4- K. g6
3 P. e5 anything.
4 E. or B. mates.
(A) 1 P. f4 (B)
2 P. e5 4- K. f5
3 B. xP. + K. X P.
4 Q. d4 mate.
(B) 1 B. X c3
2 E. X d7 dis. x
3Q. gl +
4 Mates.
No. 277.
1 Kt. g6 P. X Kt +
2 K.xP. anything.
3 (J. mates.
No. 278.
WHITE. BLACK.
1 Q. f5
2 Q. bl
3 Mates.
Kt. e5
anything.
No. 279.
1 B. 16+ K. X B.
2 B. bl R. x B.
3 Q. h8 + K. g 5
4 Q. x i'6 mate.
No. 280.
1 Q. b5 P. x Kt. (A)
2 B. h2 anything.
3 Q. or Kt. mates.
(A) B. c4
2 Kt. f3 dis. + B. X Q.
3 Ktxd6mate.
No. 281.
1 B. b.4 Q. g8 (A)
2 Q. X e5 -f Kt X Q.
3 B. el and although Black can
gwe check with four different pieces,
yet White mates next move.
(A) 1 Kt c5 +
2 Kt x Kt-f- and mates in 2 moves.
No. 282.
1 E. g3 P. K 5
2 Q.xKt. Kt X Q. (A)
3 E. b3 anything.
4 Kt. mates.
(A) 2 B. X P.
3 Q. xB. anything.
4 Mates.
No. 283.
1 Kt. c8 B. d4 (A)
2 Q. f7 E. h.7
3 Q. a2 Kt X Kt.
4 B. xP. + K. XB.
5 Q. c4 + K. X Q.
6 Kt. d6 mate.
(A) 1 Kt. d4
2 Q.xB. E. h8
3 E. g7 E. x Kt.
4 Q. f7 E. c7
5 Q,. xE. anything.
6 Q. mates.
No. 284.
1 B. h6 dis. + B. cov.
2 B. e8 Q x d5
3 B. xB. Q. dl + (A)
4 B. cov. dis. + Q. x E.
5 B. a2 a. e6
6 E.xKt+ E. X E.
7 B. x Q. mate.
(A) 3 P. X B.
4 E. xP. + K. h7
■WHITE. BLACK.
5 E. b7 + K. X B.
6 Kt. f5 + K. h5
7 P. mates.
No. 285.
1 Q. fl + K. h2
2 E. g2 + K. h3
3 E. xKt, dis.+ Q. cov.
4 E. a4 dis.+ B. cov.
5 E. Xa3 B. X B.
6 Q.xB. + Q. cov.
7 Q. fl + Q. cov.
8 E. a4 Q. X Q. mate.
No. 286.
1 B. f8 Kt. c4 (A)
2 P. e4 anything.
3 Mates.
A 1 E. x P. + (B)
2 Kt. g5 + K
3 Q. mates.
B. 1 a X P. (C)
2 Ktxf4 + E. x Kt.
3 Q,. d7 mate.
C. 1 Kt. d6 (D)
2 Q.xKt + K. x Q.
3 KtxE. mate.
DIE. f5 (E)
2 P. xE. + anything.
3 Q. f7 mate.
E 1 K. d5
2 Q. e5 + K. c4
3 Q. c5 mate.
No. 287.
1 Q. g6 K. xKt. (A)
2 Q. bl anything.
3 Q. b5 mate.
(A) K. X B. (B)
2 Kt. b7 anything.
3 Q. mates.
(B) 1 K. c4 or c6
or P c2
2 Q. d3 anything.
3 Q. or Kt. mates.
No. 288.
1 B. xP. Kt.xB. (A)
2 Q. el B. d4 + *
3 P. X B. + anything.
4 Q,. mates.
* 2 B. X P. or
K. x Kt.
3 Q. g3 + K
4 Q. mates.
(A) 1 Kt.xKt. (B)
2 Kt. f6 dis. + Kt. b5
3 Q. x Kt + anything.
4 Q. or Kt. mates.
(B) 1 P. queens (C)
2 B. f6 + K. X Kt.
3 Q. d8 + anything.
4 Mates.
WHITS.
(C)
B. f6 +
Q. b6 +
Q. eO mate.
BLACK.
1 B. b8
K. xKt.
K
No. 289.
B. el
Q. d4
B. h4 +
Mates.
(A)
P.xQ.
B. h4 +
Mates.
(B)
h4 +
8'5 +
X Q. mate.
(C) 1
h2
h4 +
B.
Q.
Q.
Q.
B.
P. e2 (A)
f4 X Q.
anything.
Q. X f7 (B)
Kt.xQ.
anything.
Q. e4 (C)
Kt. x B.
Q. e7
Mates.
B. g2
Kt. x Q.
anything.
or g4
No. 290.
1 Q. h3 R. + (A)
2 Kt. cov. dis. + K. f7 dis. +
3 Kt. e5 mate.
(A) 1 Q. or E. d7 (B)
E. h7
Q. f8 or h8
or E. X Q.
2 c4 e5 + anything.
3 Q., Kt., or B. mates.
(B) 1 K. d7 dis. +
or B. x Kt.
2 f4 e5 dis. + K. c7
3 B. mates.
No. 291.
1 B. d4 B. e4 or c6 (A)
2 B. e5 Q. X B.
Q. xh5 + Q. X Q.
Kt. X B. mate.
(A) B. h3
B. f7 + K. d7
P. queens + K. eS
Q. mates.
No.
E-
P.
E.
B.
cl
h5
hi
b2
5 Mates.
292.
bl b3
P. a6
P. a5
anything.
No. 293.
Q. b6
Kt. d6 +
Q. f2
B. mates.
Q. a6 (A)
B. x Kt.
anything,
WHITE.
(A) 1
P. XE.
Kt. g7 +
Q. f6 mate.
BLACK.
E. X P.
Q. a4
Ktx Kt.
No. 294.
(B)
1 B. f5 a8 d8
2 B. dl B. g7 (A)
3 B. a2 anything.
4 Mates.
(A) 2 E x P.
3 B. e4 + Ktx B.
4 Q, x E. mate.
(B) 2 f8 e8 (C)
3 P. x E. (a Kt.) anything.
4 Kt. mates.
(C) 2 f8 g8
3 P. x E. queens + &c.
If 1 f8 d8
2 P. queens. B. x Q.
3 Q. g8 &c.
No. 296.
Q. c7 + E. x Q.
E. d6 Q. x B.
P. d4 + K. x E.
P. c5 mate.
No.
Kt. e4 +
Q. e5 +
E.xE.
E. d7 mate.
296.
K. d5
P. X Q.
anything.
No. 297.
B. xQ.
K. x E. or
anything.
B. or Kt mates.
E.
E.
g7
a6
(A)l
Q X d7 +
Q. d3 mate.
(B)2
Q. a4 mate.
B.
B.
X B.
cov. (B)
K. c4
No. 298.
1 Q.
2 K.
P. X Q. (A)
anything.
g6
c2
Mates.
(A) 1 Kt. e6
E. c3 anything.
Mates.
No. 299.
1 E.
2 E.
h2
cl
Kt. h4 +
E. c2
B. mates.
Kt. g6
E. x E.
Kt x Kt.
anything.
WHITE
BLACK.
No.
300.
1
Q. el
B. e5 + (A)
2
K. d4
B. X E. +
3
Q. c3
B. orP.xQ. +
4
K. b3
anything.
5
Kt. mates.
(A)
1 P. f2
o
Kt. d7 +
E. X Kt.
3
E. do +
P. X E.
4
Q.Xb4 +
K
5 Q. b6 mate.
No. 301.
1 E. ol Kt. e4 (A)
2 Q. b3 P. X Q.
3 P. o4 mato.
(A) 1 B. eo
2 E. dl + anything.
3 Q.orKt. mates.
No. 302.
1 B. g6 B. g4 (A)
2 B. e8 Q. e7
3 Q. x c4 Kt. c2
4 Q. a4 + P. X Q.
5 Kt. mates.
(A) 1 B. f5
2 KtxP. + K
3 Q. dl + Kt. c2
4 Q.xKt + B. X Q.
5 B. xB. mate.
No. 303.
K. b5 (A)
K. x Q.
1 R. f4
2 Q. a4 +
3 Kt. mates.
(A) 1 P. X E.
2 Kt. c5 + anything.
3 Q. or Kt. mates.
No. 304.
1 Kt. f6 Kt X f6 (A)
2 E. d7 + Kt x R.
3 P. f6 anything.
4 B. or Kt. mates.
(A) 1 E. d8
2 Kt. g4 ' E. dG
3 B. c6 + R. X B.
4 E. mates.
No. 305.
IE. d7 + K. xKt.
2 B. f 5 + K. X B.
3 E. e7 P- moves.
4 P. X P. mate.
No. 306.
1 Q. h2 + B. cov. (A)
2 Q. h8 B. _e5 +
3 E. e3 mate. v v
WHITE. liLAOK.
(A) 1 Kt. COV.
2 Q. b2 anything.
3 E. c6 mate.
No. 307.
1 B. g7 E. e5
2 Q. el E. X Kt.
3 ft. XP. mate.
No 308.
1 E. g2 B. X E.
2 Q. a2 anything.
3 Q.orKt. mates.
No. 309.
1 B. e3 E. e5
2 Kt. b6 R. x Q.
3 B. co dis. mate.
No. 310.
1 E. f7
No. 311.
1 B. xP. B. c8
2 Kt. f3 + K. X P.
3 B. g7 anything.
4 Kt. mates.
No. 312.
1 B. el E. f6 (A)
2 Kt. e7 + K. e5
3 E. e4 mate.
(A) P. h3
2 B, g3 &c.
If 1 E. f5
2 Kt. b8 dis. +
If 1 Either Kt. moves
2 B. c3 &c.
No. 313.
1 B. g7 E. x B.
2 E. f2 anything.
3 E. g5 or f4 mate.
No. 314.
1 Q. d3 B. e8 (A)
2 Q. h7 anything.
3 Q.orKt. mates.
(A) 2 Kt. c7
2 Kt. e6 + Q. or Kt. X Kt.
3 Q. mates.
No. 315
1 B. b4 anything.
2 E.xb6 anything.
3 b7 mates.
No. 316.
1 B. a2 B. X B.
2 Q. bl Q. X P. +
3 B. xQ. anything.
4 Q.orKt. mates.
No. 317.
■WHITE.
BLACK.
1 Q. d2 E. b4
2 B. e6 B. d6
3 E. g~ E. b.5 (A;(B)
4 Q. f4 + . IfE. XQ., -
5 P. mates. If B. X Q
5 E. g4 mate.
(A) 3 B. f4
4 P. go + anything'.
5 Q. -mates.
(B) 3 B. e7
4 P. g3 + K
5 B. i'7 mate.
No. 318.
1 B. b7 + Rt X B.
2 E. a4 B. X E. (A)(B)
3 Q. a7 -f E. X Q.
4 E. c7 anything.
5 E. mates.
(A) 2 P. X Q.
3 E. XE. + B. X K.
4 E. c7
5 Mates.
(B) 2 b7 c5
3 E.xE. + B. X E.
4 Q. x o 5
5 Mates.
No. 319.
1 Kt. f5 + K. g4
2 Kt. h6 + K. g5
3 Kt. e4 + E. X Kt.
4 Q. h4 + K. X Q.
5 B. b6 anything.
6 B. d8 mate.
No. 320.
1 E. f4 + K. X E.
2 KtXd5 + K. XE.
3 Q. e2 P. X Q.
4 P. f4 mate.
No. 321.
1 B. dl K. X E. at cl
(A)(B)
2 Kt. c4 anything.
3 B. b3 dis. mate.
(A) 1 K. X E. at el
2 Kt. e4 anything.
3 B. f3 dis. mate.
(B) 1 B. X Kt.
2 B. b3 anything.
3 cl dl mate.
No. 322.
1 E. b2 B., P., or Kt x E.
2 Q. a2 anything.
3 Q. mates.
■\VJIITE. BLACK.
No. 323.
1 Q. g8 . P. X Kt.
2 E. c2 anything.
3 Q. mates.
No. 324.
1 Q. a4 P. f4
2 Kt. e3 + P. X Kt.
3 Q. XP. + K. X Q.
4 B. mates.
No. 325.
1 Q.xE. E. bl
2 E. b4 B. X K.
3 Q. d2 + Kt. or B. X Q.
4 Kt. mates.
No. 326.
1 Kt. g5 K. g7
2 Kt. g8 B. X Kt
3 Q h8 + K. X Q.
4 B mates.
No. 327.
1 B. e4 K. XB.
2 Kt. f3 dis. +K. X d3 (A)
3 Q. h7 mate.
(A) 2 K. X f3
3 Q. x Kt. mate, &c.
No. 328.
1 B. c7 B. X Q.
2 Kt. e3 anything.
3 Mates.
No. 329.
1 B. e6 E. xKt.
2 Q. d7 + K. eo
3 B. d5 anything.
4 Mates.
No. 330.
1 B. d4 K. p4
2 Q. h7 + K. X B. (A)
3 Q. d3 + B. X Q.
or anything.
4 Kt. mates.
(A) 2 K. X Kt.
3 Q. h4 4-
4 Q. g4 mate.
No. 331.
1 Kt. b2 P. x Kt.(A)
2 Q. a7 4- K. e5
3 Kt. c4 + P. x Kt.
4 Q. mates.
(A) IE. h6 +
2 P. X E. P. x Kt.
3 Q. f6 K. co
4 Q. b6 mate.
■WHITE. BLACK.
No. 332.
1 Kt. f4 P. x Kt. (A)
2 B. e4 anything.
3 Mates.
(A) 1 B. g2
2 Q. Xb6 + P. X Q.
3 Kt. e6 mate.
No. 333.
1 E. e4 1 K. X E.
2 Kt. f 6 mate.
1 B.orKt.xE.
2 Kt. f4 + + mate.
1 E. X B. +
2 KtxE. dis +
1 P. X P. +
2 B. X P. mate.
1 Q. e6
2 E. d4 mate.
I E. x P. +
2 Kt. cov. dis. mate.
1 Kt. f4
P. d6 E. e7
Q. e8 or
Ktx P.
2 Kt. f6 mate.
1 P. x Kt.
R. g6 &c.
2 E. e5 mate,
No. 334.
1 Kt. f4 1 K. d6
2 Kt. e6 mate.
IK. f6
2 Kt. e6 mate.
1 E. xKt. +
2 Kt. f3 mate.
IB. c4 +
2 Kt. e2 mate.
1 P. +
2 Ktx P. dis. mate.
IB. c4 +
2 Kt. e2 mate.
I K.xKt.
2 Kt. e6 + + mate.
.1 E. a7 or d7
or E. g2 or Kt. e2
2 Kt. f3 mate.
No. 335.
1 Q. g4 1 K. c6
2 B. d7 mate.
1 K. c4
2 B. X P. mate.
1 K. e5
2 B.X P. dis. mate.
1 Kt. moves.
2 Q,. e4 mate.
1 P. moves.
2 B XB. mate.
WHITE. BLACK.
No 336.
1 P. h3 1 K. X E. or
E. X e6
2 B. h2 mate.
1 B. X E.
2 P. e3 mate.
1 Q. X B. or
Q. X dl &c.
2 E. f5 mate. &c.
No. 337.
1 Kt. d6 E. X Kt. (A)
2 K. X B. anything.
3 Mates.
(A) 1 B. X B.
2 Kt. e5 + K
3 P. d3 mate.
No. 338.
1 E. c7 1 K. X P.
2 E. e4 mate.
1 E. X E.
2 Q. do mate.
1 Kt. e4
2 E. XE. mate.
1 Kt. d3
2 P. e3 mate.
1 Kt. anywhere
2 Q. gl mate. else.
No. 339.
1 B. b7 K. x Kt.
2 Q. xP. + K. x Q.
3 E. c4 dis. mate.
No. 840.
1 Q. dl P (A) (B)
2 P. d3 +
3 Mates.
(A) 1 K. d5
2 Q. a4 K. X P.
3 Kt. d8 mate.
(B) 1 K. d3
2 K.XP. + K. d4
3 Q. al mate.
No. 341.
1 Kt. h2 E.. f4
2 Q. f7 Kt X Q.
3 E. g4 + K. x E.
4 Kt.Xf3mate.
No. 342.
1 Q. h7 E. xE. (A)(B)
2 Q. a7
3 Mates.
(A) 1 P. x Kt.
2 Q. + &c.
(B) 1 K. XKt.
2 0. + &c.
WH^TE.
BLACK.
2
Q.
(C) 1 Kt. e7
X Kt.
3
Q.
mates. •
No. 343.
1
2
3
Q.
Q'.
Q.
g3 P- f4 (
g6+ K
c6 + K. X Q.
4
Kt
mates.
(A) 1 K. d5
2
Kt
b4 4- K. e6
3
4
Q.
B.
g6 + K. e7
d.6 mate.
No. 344.
1
E.
e3 4- P. X E.
2
E.
e7 + K. f6
3
4
Q.
Q.
fl anything.
orB. mates.
No. 345.
1 Kt. f8 Kt.xKt.
2 Kt. g5 anything.
3 Kt. or B. mates'.
No. 346.
K. g5 (A)(B)
K. xQ.
1 Q. hi
2 Q. b.4 +
3 Kt. mates.
(A) 1 K. e3
2 B. xE. anything
-3 Q., Kt., or B. mates.
(B) l P—
2 Q. el anything.
3 Mates.
No. 347.
1 Q.
f3 B. gl (A)
or E. g3
2 Q. fl anything.
3 Kt. d3 anything.
4 Kt. or P. mates.
(A) 1 E. g4
2 Q. xP. E. xKt.
3 Q. a7 anything.
4 Q. mates.
(B) 2 B. gl
3 Q. d3 + B. X Q.
4 P. mates.
No. 348.
Kt. e4
E. a7
Q. Xd8
Qi. mates.
(A)
Kt. f6 +
E. g4
Mates.
P. X Kt. (A)
B. xE.
anything.
P. XB.
K
anything.
No. 349.
■WHITE. BLACK.
1 B. b3 B. X B. (A)
2 B. c3 Kt. c4
3 Kt. d5 + P. x Kt.
4 K. g8 anything.
5 Mates.
(A) 1 B. x P.
2 B. c3 Kt. c-4
3 B. X Kt. B. X P.
4 B. x P. -I- K. f5
5 P. mates.
No. 350.
t Q. e3 E. i2
2 Kt. b5 anything.
3 Mates.
No. 351.
IB. f8 IE. anywhere.
2 Kt. at c6 mates.
1 KtxE
2 Q. X Kt. mate.
1 h5 g7 or
Ktx f4
2 Q. h7 mate.
No. 353.
Numbered 351 in error.
bl K. d5 (A)(B)
b4 + K
c3
1 Q.
2 Kt.
3 Mates.
(A) 1 K.
2 E. XB. + K
3 Q. el mate.
(B) 1 Kt
2 Kt. e3 + anything.
3 Q. mates.
(C) 1 B. b6 or a7
2 Kt. a3 + &c.
(D) 1 B. d6 &c.
2 Kt. e3 + &c.
No. 353.
IE. c4 - Q. .1-!
2 Q. e4 +
3 E. mates.
K. orQ. X Q.
No. 354.
1 Q. f6 Q. a2
2 Q. X f5 Q. or B. +
3 P. b5 + Q. or B. X P. +
4 Q. retakes, mate.
No. 355.
1 B. d6 E. f8 (A)(B)
2 Kt. d2 anything.
3 Q.orP. mates.
(A) 1 E. e6
2 B. X R. + .Q. X B.
3 P. mates.
51
"WHITE. BLACK.
(B) 1 P. or KtX Q.
2 E. f6 + Q. X E.
3 P. mates.
(C) I Q.,E.,orKtxB.
2 Kt. retakes + anything.
3 P. mates.
(D) 1 B. X d5 or
E. g8
2 P. g4 + K. ±'4
3 E. f 6 mate.
No. 356.
1 Q. b4 Ktx Q. (A)(B)
2 Kt. e3 dis. + K. X Kt.
3 B. mates.
(A) 1 K. d5
2 Q. c4 + K. X Q.
3 Kt. mates.
(B) 1 K. X P.
2 Q. f8 + K
3 Q,., B. or Kt. mates.
No. 357.
Q. n3 E. XP. + (A)(B)
Q.XE. + K. X Kt. or
KtX Q.
E. or Kt. mates.
(A) 1 Kt. e6 +
K. X Kt.
Q.xKt +
Kt. mates.
(B) 1 Q.
f3 or
E. e3 or
P. X Kt.
Q. orE.xKt.
d5orP.xP.
X Kt. or
e5
Kt. e4 +
Q.orE. mates.
(0) 1 Q.
Kt. b5 + K
K
Q. orB. mates.
(D) 1 K. X Kt.
Q. c3 + K. d6
Kt. mates.
No. 358.
E. h2 Q. c2 (A)
E. X Q. E. X P. (a)
E. c5 E. X Q.
E.xKt+ P. X E.
Kt. c5 mate.
If 3 R. X B.
Q. g2 +
Q.orKt. mates.
If 3 P. d2
Q. bl + Kt. gov.
Q. hi mate.
(a) 2 Kt. f3 (b)
KtxKt. P. d5
Kt. c5 + K. f5
Kt. h4 mate.
If 3 P. d2
KtxP. +
Q. mates.
AVHITB.
Q.
Kt.
If 3
f7 +
d2 mate.
(b)2
E. c5
Q. d5 +
B. xB. mate.
(o)2
Q,. e6
Kt. c5 +
Q. X Kt. mate.
(d)2
E. c5
B. d5 +
Q. or P. mates
(A)l
KtxKt
B. h7 +
Kt. f6 +
Kt. b.4 mate.
BLACK.
K. f5
K. e4
R a5 (e)
Kt. f3
B. X Q.
f8 g6 (d)
Kt X Kt.
P. X Kt.
e5 g6
KtX h4
anything.
Kt.
Q.
P.
K.
f3
cl,e2 or c3
g6
15
Several other variations on Black's
second move.
No. 359.
1 B. b5
1 Q. X B. or
Kt. d2
2 Kt. f2 mate.
1 Kt.
P.
2 KtxP. mate.
No. 360.
c3 or
d4
e5
1 Kt. g3 K.
2 Q. d6 mate.
IK. e3
2 Q. b6 mate.
1 K. c4
2 Q.. d3 mate
1 K. d5 or
B or
P
2 Q. e4 mate.
No. 361.
1
E.
e8 +
E. X
E.
2
E.
a6 +
E. X
E. (A)
3
Q.
XE. + !
md draws by per-
petual
check.
(A)
2 K.
d5
3
B.
d6dis.
+ K.
e4
4
Q.
e2 -+-
K.
f5
5
Q.
h5 +
K.
e6
6
B.
e5 dis.
+ &e. as before.
No
. 362.
1
E.
g6 +
K.
h3
2
E.
gl
P. X
E. (aQ.)+
3
K.xQ.
K.
g3
4
K.
fl
K.
f3
5
P.
e8 (aQ
.) winni
as:.
No. 363.
WHITE. BLACK.
1 E. el P. queens. (A)
2 Kt. d6 + P. X Kt.
3 B. XP. + K. XB.
4 Kt. d4 mate.
(A) Black has other moves, but he
cannot prevent the mate in four.
No. 364.
1 K. gl P. x Kt.
2 B. d3 B. x E. (A)
3 Q. X h8 anything.
4 Q. mates.
(A) 2 E. X B.
3 Q. c3 &c.
No. 365.
1
Q.xKt. p. x a
2
B. h7 anything.
3
Mates.
No. 366.
1
Q. al Kt. c6+(A)(B)
2
E.xKt. P. e5 (a)
3
Q. c3 K. e6 or
anything.
4
or
Q.XP.) .
Q. ci \^^
(a) 2 If K. X E.
3
Q. a5 &c.
2 If P. X E.
3
Q. a4 &e.
(A) 1 Kt X E.
2
Q. c3
3
Q. orKt. mates.
(B) 1 P. e5
2
R. c5 + P. X E.
3
Q.XP. &c.
No. 367.
1 E. d2 E. x R.
2 KtXf4 E. d5 (A)
3 Q. dl anything.
4 Q. or P. mates.
(A) 2 Kt. e6
3 Q.xKt. +
4 P. mates.
No. 368.
1 E. el K (A)
2 Kt. d5 anything.
3 Mates.
(A) 1 B. +
2 KtxB. + Kt X Kt. or
K. c6
3 Q. c5 mate.
No.
369.
1
Q.
cl
1 B.
b5
K. e6 (A)(B)
2
b7
co
2 Q.
f5 +
K, x Q. or
3
P.
e3
K j
4
Kt.
mates
WHITE. BLACK.
3 B.or Q. mates.
(A) 1 K. d6
2 Q. f7 &c.
(B) 1 K d8
2 Q. xh7 &c.
(C) 1 R. a6 or
E. fl
2 Q. XP. &c.
No. 370.
1 E. xKt P. x R. (A)
2 Kt. e5 P. x B.
3 Q. d2 anything.
4 Q. mates.
(A) 1 Q. al +
2 E. X Q. E. a5
3 E. X R. anything.
4 E. a8 mate.
No. 371.
1 Q. X B. Q. x E. (A)
2 P. f 4 + K. e6
3 B. b7 anything.
4 B.or Q. mates.
(A) 1 Kt X Q.
2 E.xKt + K. e6 (B)
3 K. f8 anything.
4 Kt. g7 mate.
(B) 2 K. e4
3 E. h4 + K. f3
4 E. d2 dis. mate.
No. 372.
1 Q. f4 E. x Q. (A)
2 Kt. e3 + K. e4, e5, or e6
3 E. e7 mate.
(A) E. X Kt. at d8
or moves.
2 Q. f5 + E. cov.
3 Q. X E. mate.
If 1 any other move.
2 Q. d6 mate.
No. 373.
1 B. e5 Q. f8 (A)
2 Kt. e3 + K. x B.
3 P. mates.
(A) 1 P. c5
2 Q. c4 + K. X Q. (B) (0)
3 Kt. e3mate.
(B) 2 K. x B.
3 Q. X c5 mate.
(0) 2 K. c6
3 Q. x E. mate.
No. 374.
E. x Q.
P. f3
anything.
No. 375.
3 Q.
WHITE.
E. g2
Q.XP.+
P. mates.
If 2
h3 mate.
BLACK.
E. g5
K. X Q.
K. x Kt.
376.
KtxE. (A)
Ktx Q.
No.
1 Q. c8
2 Q. h3 +
3 P. mates.
(A) 1 E. moves.
2 E. h3 + Kt. X E.
3 Q. X Kt. mate.
No
1 B. f8
2 Kt. c8
3 Kt. mates.
(A) 1 E. x B
2 Kt. g4
3 Kt. mates.
377.
B. x B. (A)
anything.
anything.
No.
Q, c2
E.X d8
P. e6
P. e7
Mates.
378.
P. x Q.
Ktx R.
P. x.Kt.
anything.
No. 379.
B.
E.
g4 +
e8 +
E.Xg6
Kt.
B.
fo
f3 +
K. e4
B. x B.
B. x E.*
B. x Kt.*
K. x B.
Stalemate.
•If Black make any other move he will
lose the game.
No. 380.
E. h6
Kt. e2 +
Q. X Kt+
B. mates.
(A)
Kt. e6 +
E, h5
Mates.
Q. X E. (A)
K
K
Q f6
Kt X Kt.
anything.
No. 381.
Q.
Q.
Kt,
2 Q.
c8
fo
b5 mate.
(A) I
fo +
K. d4 (A)
anything.
Q. X B.
K. X Q. or
K
3 Kt. mates.
No. 382.
1 Kt. c4 dis. + B. cov.
2 Q. a6 Kt. f7
Kt. e5 +
Q. XE. +
K. al
Kt x Kt.
Kt x Q. +
P. b2 mate.
No. 383.
1 E. c7 Q. c3 + (A)(B)
2 E. xQ. K. or KtxE. or
anything.
3 Q. b2)
Q.xP. > mate accordingly,
or Q. e5 )
(A) 1 Q. X Kt.
2 E. c4 + P. X E.
3 Q. e5 mate.
(B) 1 K. e3
2 E. c3 + Kt X E.
K. X Kt. or
K
AH
3 Q.
Q.
or Q.
g3 )■ mate accordingly.
e5j
No. 384.
B.
Q.
3 bl
4 E.
E.
Q.
8 Ktx P.
g'
a8 +
b2
a2 +
a4 +
d8
e7 +
B. b8
B. cov.
Kt. f2
K. xE,
K. c5
B. b8
B. cov.
B. X Q. mate.
No. 385
1 Q. el P. x Kt. (A)(B)
2 Q. XP. K. f5
3 Q,. g5 mate.
(A) 1 K. X Kt.
2 Kt. it P
3 CJ. c4 mate.
(B) 1 K. d6
2 Q. cG + K. e5
3 Q. xP. mate.
(C) 1 K. fo
2 Q. f4 + K. g6
3 Q. f7 mate.
No. 386.
1 K. gl P. g2 (A)(B)
2 Kt. c6 K. X either Kt.
3 Q. c2 or c8 mate.
(A) 1 K. b4
2 a bl + K. x P.
orK
3 Q. b3) ,
or a b6 | mate -
(B) 1 K. d4
2 Q. f6 + K
3 Q. mates.
No. 387.
WHITE. BLACK.
"WHITE. BLACK.
No. 391.
1 B.
a4 + K. d5
1
K. f4 K. d4 (A)
2 E.
hi anything.
2
B. fl K. d5
3 E.
dl anything.
3
B. c4 + K. d4
4 B.orB. mates.
4
B. e5 mate.
No. 388.
(A) IP. e5 +
2
B. XP. P. h5
1 E.
d8 + ' K. e6
3
K. f5 P. h4
2 d8 X P. + K. e5
4
B. g2 mate.
3 E.
h4 dis. + E. cov.
4 B.
d3 E. a3 or
No. 392.
5 E.
6 Kt.
5 E.
anywhere, or
B. f7 (A) (B)
h5 + B. X E.
f3 + B. X Kt. mate.
(A) 4 B. g6
e4 + B. x E.
1
2
3
or
E. c3 K. f3 (A)
P. queens -4- K. e5 or
K. xB.
a x g 7 l mate -
6 Kt.
f3 + B. X Kt. mate.
(B) 4 B. d7
2
3
or
(A) 1 any other move.
Kt. f3 anything.
B. f6 ->
P. f8 (a Kt.) Vmate.
5 Kt.
6 Kt.
g4 + B. X Kt.
f3 + B. x Kt. mate.
(C) 4 Kt x Kt. or
or
P- g3 J
5 Q.
Ktx e7
g5 + Kt. cov.
No. 393.
6 Q.
g3 + Ktx Q. mate.
1
Kt. e2 P. x Kt.
(D) 4 Kt x f 6
" 2
E. d7 B. x E.
5 Q.
e3 + Kt. cov.
3
Kt. f7 anything.
6 Q.
g3 + Kt X Q. mate.
4
B. or Kt. mates.
No. 389.
No. 394.
1 b3
a5 + K. a8
2 Kt
b6 + Kt X Kt.
1
e8 X B. P. x E.
3 P. h8 queens + Kt. c8
2
Kt. g5 P. x Kt.
4 Q.
xKt + Q. x Q.
3
B. e8 B. x B. (A)
5 K.
e3 dis. + K
4
Q. h7 B. X E. *
6 K.
e4 dis. + K
5
Q. a7 mate.
7 K.
d4 dis. + K
* 4 K. x E.
8 K.
d5 dis. + K
5
Q. e7 mate.
9 K.
c5 dis. + K
(A) 3 If any other
10 K.
c6 dis. + B. i-2
move,
11 B.
XB. + K
4
E. +
12 K.
b6 dis. 4- B. g2
5
P. mates.
13 B.
XB. + Q. cov. +
14 B.
xQ- mate.
No. 395.
No. 390.
1
Q. a7 B. x Kt.
1 B.
2 B.
3 Q.
4 Q.
5 Q.
f2 dis. + K. g2
a 7 dis. + K. f3
f2 + K. e4
X P. + K. d3
e3 + K. X P.
2
3
4
5
6
Q. gl + B. fl
E. a8 P. g3
E. a2 P. g2
K. c4 K. X Kt.
Q. h2 K dis. mate
6 Kt
7 Q.
a3 + E- X Kt.
d4 + K. b3
No. 396.
8 P.xKt.(aB.)+E. c4
1
B. g8 R. x B.
9 P.
b8(aE.)4-B. b7
2
Kt. b2 Q. x P. +
10 Q.
al P. g6
3
Kt. d3 mate.
11 B.
gl P. X E.
12 K.
Xh5 P. f4
No. 397.
13 K.
g4 P. f3-dis. +
14 K.
Xh3 P. f2
1
E. c7 E. e8
15 Q.
b2 + K. X Q. dis.
2
Q. el anything.
mate.
3
Mates.
No. 398.
WHITE. BLACK.
1 R. f 8 E. x R. (A)
2 P. f4 R. X P.
3 Kt. g3 mate.
(A) 1 P. e5
2 R. f4 + P. X R.
3 Kt. f 6 mate.
No. 399.
1 Kt. g6 Q. b8
2 Q. XB. Q. orKt. X Q.
3 Kt. mates.
If 2 Q. e5
3 Q.. e5 mate.
No. 400.
IP. c4 + K. e6
2 g6xKt+ B. X R.
3 KtX P. + R- X Kt.
4 Q. b3 anything
5 Mates.
No. 401.
1 Kt. d4 1 Kt X Kt.
2 P. mates.
1 K. X Kt.
2 Q. d6 mate.
1 K. d5
3 CJ. c5 mate.
1 Kt.
2 Kt. c6 mate.
No. 402.
1 Kt. c5 1 K. f6
2 Kt. d7 mate.
1 K. d6
2 0,. e6 mate.
1 K. d4
2 KtxKt. mate.
1 K. f4
2 B. c7 mate.
1 Kt x Kt.
2 Q. d5 mate.
No. 403.
1 Kt. g7 1 Kt X P. +
2 B.xKt. mate.
I Kt. anywhere
else.
2 P. d4 mate.
1 K. e4
2 R. e6 mate.
No. 404.
1 B. b6 K. e7
2 Kt. e5 Kt X Kt.(A) (B)
3 Q. f6 + K. X Q.orKt.
4 B. or Q. mates.
WHITE. BLACK.
3
4
3
4
(A) 2 K. X Kt
Q. f8 + K. X Kt
Q. f4 mate.
(B) 2 B. X B.
Q. f8 + K. X Q.
Kt. g6 mate.
No. 405.
1 B. f4 P. X B.
2 P. d4 R. X P.
3 R. e5 + K. X R.
4 R. e7 mate.
No. 406.
1 Kt. b6 P. f3
2 B. cl Kt X B.
3 Q. XP. P. X Q.
4 Kt. g6 mate.
No. 407.
1 Q. b5 Q. a5
2 hi f2 P. X Kt.
3 Q. e5 + Q. X Q.
4 B. mates.
No. 408.
1 R. h7 Q. f4
2 Q. cl anything.
3 Mates.
No. 409.
1 B. c8 Q. X P, (A)
2 Kt. iS Kt X Kt. *
3 Q.. c3 anything.
4 Q., B., or P. mates.
*2Q. c3
3 Q. d4 + anything.
4 Kt. mates.
* 2 Q. X c4
K. X Kt. or
B. X Kt.
3 P. f 3 + anything.
4 Kt. or P. mates.
(A) 1 Kt X f2
2 Q. e5 + B. X Q.
3 B. X P. + K. e3
4 Kt. c2 mate.
No. 410.
1 Ktxb5 Q. d7 + (A)
2 Kt d6 Q. X Kt. +
3 K. e4 dis. + Q. cov. +
4 B. X Q. mate.
(A) 1 Kt. f2 +
2 K. c3 Kt. e4 +
3 Q. X Kt. B. X Kt. +
4 Kt. cov. mate.
(B) 1 Kt. b2 +
2 K. c3 ' Kt. dl +
WHITE.
BLACK.
3
K. d2
Q. d7 +
4
Kt. cov. mate.
No.
411.
1
Kt. e6
P +
2
K. e4
P. g 3
3
E.xP
P. g5
4
B. c4
P. g4
5
K. d3
anything.
6
Mates.
No.
412.
1
K. c3
R. d7 (A)(B)
2
Q. X Kt.
B. x Q.
3
B. d6
anything.
4
Mates.
(A)
1 Kt X B.
2
Q. h8
K. xE.
3
Kt. f6 +
anything.
4
Q. mates.
fB)
1 Kt X P.
2
KtxB.dis. + Kt xB.
3
KtxE.
anything.
4
Kt. mates.
No. 413.
1 E. h3 K. f5 (A)
2 Q. g4 + K. X Q. *
3 B. e6 mate.
* K. e4
3 E. e4 mate.
(A) 1 B
2 Q. e2 + &c.
No. 414.
1 Q. g3 + P. X Q. (A)
2 Kt. d6 dis. + P. X E.
3 B. g2 anything.
4 Mates.
(A) 1 K
2 Q.X P. + K
3 R. x P + Q. X E.
4 Q. mates.
No. 415.
1 Q. f8 Kt. a4 (A)
2 Q. f4 Q. X Q. *
3 Kt. d6 + Q. X Kt.
4 Kt. mates.
* 2 Kt. c3 +
3 JTt X Kt. anything.
4 B. mates.
(A) 1 Kt. d3
2 P. X Kt. + K. X P.
3 Q. fl + K
4 Q. mates.
No. 416.
1 KtxP. 1 B. X P. +
2 Kt. f6 mate.
■WHITE.
BLACK.
1 B. X Kt.
2 B. f7 mate.
1 Kt X Kt.
2 Q. a2 mate.
1 K. e6
2 Kt. c5 mate.
No. 417.
1 B. c2 B. X B.
2 E. e8 E. X E.
3 P. d4 anything.
4 Kt. mates.
No. 418.
1 Q. e8 E. f4
2 Q. e7 anything.
3 Q. or Kt. mates.
No. 419.
1 Kt. d3 E. b5
2 Q. e7 + E. c5
3 Q. X E. + E. X Q.
4 B. mates.
No. 420.
1 B. b6 1 P. X B,
2 Q. d3 mate.
1 P. X B. or
P. c6 or c5
2 Kt. d6 mate.
1 e3 moves.
2 Q. d5 mate.
1 a3 moves.
2 B. c2 mate.
1 B. e3
2 E. X B. mate.
1 P. h.6 or h5
2 Q. h7 mate.
I E. f4
2 Kt. g5 mate.
1 R. g4
2 Q. X E. mate.
1 E. anywhere
else.
2 Q. g4 mate.
No. 421.
1 E. el 1 P. e2
2 Kt. c6 + anything.
3 Q. mates.
1 K. d6
2 Ktxb5 +
3 Q. mates.
1 K. X B.
2 Q. f5 + K. X Kt.
3 E. mates.
1 P. f4
2 Q. e7 +
3 Kt. or E. mates.
WHITE. BLACK.
1 B. x E.
3
Q. e6 +
2
Q. or Kt. mates.
No. 422.
1
B. e7 Kt. d6
(A)(B)
2
Q. c4 Kt. X Q.
#
3
E. f5 + K. x Kt
4
B. mates.
* 2 B. X E.
3
Q. c5 + K
4
Kt. e2 mate.
(A) 1 E. x B.
2
Q. g5 + K. d6
*
3
E. f6 +
3
Kt. mates.
* 2 K. X Kt.
3
P. c3 + K
4
Q. e2 mate.
(B) 1 B. X E.
2
Q. b5 + K. f4
*•
3
Kt. e2 + K
4
B. mates.
* 2 P. f5
3
Q. x e4 + K
4
Kt. mates.
(C) 1 K. x Kt.
2
Q. b5 E. x B.
3
P. c3 + K
4
Q. mates'.
No. 423.
1
G. x c6 B. X Q.
(A)
2
B.XP. + K. X B.
3
Kt. mates.
(A) 1 K. X E.
2
Q.XP. + K
3 Q. e3 mate.
No. 484.
1 Kt. d2 dis. + K. g3
2 K. Ill P. X P.
3 Q. g8 E. g5
4 B. d6 + either E. cov.
5 B.xE. + E. X B.
6 Kt. f5 + E. X Kt.
7 Kt. fl + E. X Kt.mate.
No. 425.
B. xE. (A)(B)
Kt X Q.
1 Q. al
2 Q. a8 +
3 B. mates.
(A) 1 K. X E.
2 B. c4 + K
3 Q.xB. mate.
(B) 1 R. h8
2 Q,. e5 anything
3 Mates.
(C) IB. cl
WHITE. BLACK.
2
Q. d4 anything.
3
Mates.
(D) 1 B.
g2
2
E. e3 + anything.
3
Mates.
(E) 1 B.
h3
2
Q. hi + anything.
3
Q. or B. mates.
No. 426.
1
B. f6 E. x
B. (A)
2
h5 g7 E. x
Kt. *
3
Q. X P. + K —
4
Kt. mates.
* 2 P.
e3
3
Q. c5 + K —
4
Kt. mates.
(A) 1 P.
e3
2
e6 X P. + K.
e4
3
Q. c6 + K —
4
Q. mates.
No. 427.
1 Kt. d7 + K. d4
2 Q. hi Black has choice
of five moves, but in any case
3 Q. or E. mates.
No. 428.
1 E. f4 B. X B. (A)
2 Q. h8 E. X B. *
3 Q,. a8 anything.
4 Q. mates.
* 2 K. X E.
3 Ktx P. + K
4 Q. mates.
(A) 1 E. x B.
2 Kt. f3 + P. x Kt.
3 0. h8 + K
4 Q. mates.
No. 429.
1 B. h3 K. d5 (A)
2 Q. c5 + E. x Q. *
3 Kt. e7 + K. c4
4 Kt. d6 mate.
If 3 K. X Kt.
4 B. f5 mate.
* 2 K. X Kt.
3 Kt. e7 B. c4
or E. x Q.
4 B. f5 mate.
If 3 Kt. g3 or g7
4 CA. d5 mate.
(A) i a x Kt.
or B. x Kt.
or B. fl
2 Q g8 + K. d7
3 Q. f7 + K. c8
4 Kt. d6 dis. +
No. 430.
WHITE. BLACK.
I
lit. f5 1 K. X Kt.
2
E. X 1'. mate.
1 R. e8
2
B. X Kt. mate.
1 E. X Kt.
]'. X Kt.
Kt
orB
2
Q. c3 mate.
No. 431.
1
Kt. d3 E. c4 (A)(B)
2
Q. d4 + K. X Q. *
3
E. X Kt. mate.
41 2 E. x Q.
3
E. c5 mate.
■ * 2 K. X E.
3
Kt. e5 mate.
(A) 1 B. X Kt.
2
Q. d7 anything.
3
Q. X Kt. mate.
(B) 1 B. X E.
2
Q. e5 + K. c4
3
Kt. b2 mate.
(C) 1 Kt. f7
2
Q. X Kt + anything.
3
Q. mates.
(D) 1 K. x E.
2
Q. c7 +
3
Q. mates.
(E) 1 Q. X f2
2
KtX f4 + K. x E.
3
Q. c7 mate.
No. 433.
1
P. go + 1 K. x P.
o
Q. X f7 2 B. X Q. *
3
E. eo + 3 K. f6
4
Kt. d7 mate.
If 3 K. f4
4
Kt. d3 mate.
* 2 E. X Kt.
3
Q. e7 + 3 K. X B.
4
Q. X P. mate.
If 3 K. f5
4
B. g4 mate.
* 2 B. X B.
3
E. e5 + K. g4
4
Q. f5 mate.
* 2 K. X B.
3
Q. f3 + K. g5
4
Kt. e6 mate.
* 2 B. f5
3
E. g4 + B. X E.
4
Kt. e4 mate.
* 2 Q. X B.
3
Kt. e6*+ K
4
Q. x P. mate.
WHITE
BLACK.
* 2 Kt X Kt.
or Kt. d8
3 E. ej + B. cov.
4 Q. x B. mate.
* 2 P. queens,
or Kt. d6
or Kt. c6
3 Kt. e6 + K. x B.
4 Q. f3 mate.
No. 433.
1 Q. a7 K. d4 (A)(B)
2 E. e2 K. e5 *
3 Q. g7 mate.
*2 KtX Kt.
3 P. xKt. dis. mate.
* 2 anything else.
3 Kt. b5 mate.
(A) 1 Kt x Kt.
2 Q. g7 + K. f5
3 E.XE. mate.
(B) 1 Kt X P.
2 B. x E. + K. f6 or d4
3 Q. orKt. mates.
(C) 1 Kt x B.
2 Q. g7 + E. f6
3 Q. X E. mate.
(D) 1 K. fC
2 B. X Kt.
moves.
3 Q.orKt. mates.
K., P., or E.
No. 434.
1 B. a4 Kt X E. (A)
2 Kt. e5 K. X Kt.
3 B. g7 mate.
If 2 K. X P.
3 Kt. b3 mate.
If 2 K. c3
3 Kt. f3 dis. mate.
If 2 B. X Kt.
3 Q. X Kt. mate.
(A) 1 B.xE.
2 Kt. b3 + K. e4
3 Q. hi mate.
No. 435.
1 Q.xP. E. g6 (A)
2 Kt. c8 anything.
3 Q. , R., or Kt. mates.
(A) 1 B. X E.
2 B. e4 + K. c5
3 Kt. c8 dis. mate.
(B) 1 Kt. f5
2 Q. b7 + K. x Kt.
3 B. e5 mate.
(0) 1 Kt. c5
2 B. e4 4 Kt X B.
3 Q. e5 mate.
.52
No. 436.
WHITE. BLACK
1 Q. o4 B. X Q. (A)
2 K. d7 + K
3 P. mates.
(A) 1 K. x E.
2 Q. c5 +
3 Q. mates.
(B) 1 B. X E.
2 Q. X f i + K. c-5
3 Q,. b4 mate.
No. 437.
1 Q. g4 1 B. x Q.
orB
2 E. d4 mate.
1 K. xKt.
2 Q. f3 mate.
1 K. x E.
2 Kt. e3 mate.
1 Kt
2 Q. x B. mate.
1 K-
2 E. d.4 mate.
1 P. f3
2 Kt. b4 mate.
No. 438.
1 Q. 08 + K. h7
2 R. c7 + K. h6
3 E. g6 + K. h5
4 E. a6 dis. + K. g5
5 Kt. f3 + K. f5
6 B. c2 + E. cov.
7 Q. f7 + Q. cov.
8 K. x P. Q. X Q. (forced)
9 E. c5 + Q. cov.
10 B. bl Q. e5
11 E. d5 Q. X E. mate.
No. 439.
1 Kt. b3 K. X Kt. (A)(B)
2 Kt. a5 dis. + K
3 Kt. or B. mates.
(A) 1 K. e5
2 B. g3 + K. f6
3 Kt. d5 mate.
(B) 1 E. X E.
2 Kt. d5 + K
3 Kt. or B. mates.
No. 440.
1 Q. g8 Q. x Q. (A)
2 KtxP. + Kt. or P. X Kt.
3 Kt. mates.
(A) 1 B. f7
2 Q. x Kt. anything.
3 Q. a2 mate.
No. 441.
WHITE. BLACK.
1 E. f4 B. orP.xE.(A)
2 Q. f5 + K. X Q.
3 Kt. mates.
(A) IK. d7
2 Q. c8 + K. X Q.
or K. X Kt.
3 Kt. mates.
No. 442.
IP. d3 + K. d4
2 Kt. f5 + E. X Kt.
3 Ktxc7 K.xE. (A)(B)
4 B. e3 mate.
(A) 3 E. X e5
4 E. c4 mate.
(B) 3 E. f6
or E. e8
orB. X P.
4 R. d5 mate.
(C) 3 P. f2
or E. X b5
4 Kt. e6 mate.
No. 443.
1 f5 x B. + K. e8
2 Q. X b4 B. X Q.
3 B. b5 mate.
If 2 E. X Q.
3 K. X P. dis. +
If 2 Q. X Q.
orQ. X B.
3 P. c8 queens +
If 2 Q. X P. +
3 K.X Q. dis. mate.
No. 444.
1 B. h4 + E. X B.
2 Q. a7 K. X R.
3 Kt. e6 + + K. has five
squares open to bim, but where-
ever he goes
4 Q. mates at g7
No. 445.
1 Q. f2 E. b7 (A)
2 Q. X B. Q. d4
3 Q. d6* Q. gl
4 Kt. f 2 + Q. x Kt.
5 Q. h2 mate.
(A) 1 B. d7
2 Q.xB. Q. d4
3 Q.xQ. E. fl
4 Q. gl + E. X Q.
5 Kt. f2 mate.
* If white here play 3 Q. X Q. he
will be foiled by 3 E. b2.
No. 446.
WHITE. BLACK.
IE. d7 + Kt X E.
2 Kt. e6 anything.
3 K.orKt. mates.
No. 447.
1 E. h4 B. X E. (A)
2 Q. d6 P. c6 *
3 Q. f8 + R. x Q.
4 Kt. d6 mate.
* 2 Kt X P.
3 Q.xKt+ K. d8
4 Q. c8 mate.
(A) 1 B. IB
2 E. X E. anything.
3 Q.orE. mates.
No. 448.
IB. b3 +
2 E. a4 +
3 E. c4 +
4 Q. b5 +
5 Q. c6 +
6 Kt. ib +
7 Q. d6 + K. e8
8 Q. d7 +
9 Q. e7 +
10 E. h4 dis. +
11 Q. e8 + B. cov.
12 P. mates.
No. 449.
1 Kt. g6 + P. X Kt.
2 Q. h2 + Kt. h6
3 Q. X Kt + P. X Q.
4 B. X d4 + K. h.7
5 Kt. g5 + P. X Kt.
6 E. hi + Q. X E.
7 E. X Q. mate.
No. 450.
1 Q. X a7 +
2 Kt. b6 +
3 B. d6 +
4 E. c8 +
5 E. a8 +
6 Kt. d5 +
7 E. c8 +
8 E. X Q. mate.
KtX Q.
K
P. X B.
KtX E.
K. c7
K. c6
Q. cov.
No. 451.
1 B. X c7 + K. X B.
2 P. . b6 + K. b8
3 E. a8 + K. X E.
4 E. a4 + K. b8
5 E. a8 + K. X E.
6 Q. a I + anything
7 Q. mates.
No. 453.
WHITE.
BLACK.
1
E.
f8 +
B. XE.
2
Q.
g8 +
K. xQ.
3
Kt.
f6 +
K. h8
4
E.
XE. mate.
No.
453.
1
E.
b8 +
B. cov.
o
Q.
h5 +
K
3
P.
e7 +
K-
4
Q.
g6 +
K
5
B.
f6 +
Q. x B.
6
d7
Xd8 +
Q. cov.
7
P.
d6
E- +
8
K.
fl
Q. xE.
9
P.
c5
Q. g 8
10
P.
ffi
Q
11
0.
h6 +
K. g8
12
P.
b4 mate.
No. 454.
1 Q. a6 + K
2 E. b5 + K
3 Kt. c2 + K
4 E.xg5 + K
5 Q. e2 + K
6 R. g3 + Kt X B.
7 Q. fl + K
8 Q. el + K
9 K. f4 + P. x E.
10 B. fl mate.
No. 455.
1 Kt. c6 + K
2 E. b6 + P.xE.
3 P. xP. + K
4 Q. a7 + K.xKt.
5 B. d7 + K
6 Q. a2 + K
7 Q. g2 + K
8 Kt. g6 mate.
No. 456.
1 Kt. b5 + K
2 Q. a7 + B. x Q.
3 Kt. c7 + K
4 Ktxa6 + K
5 Kt. c7 + K
6 Ktxd5 + K
7 Kt. b6 + B. X Kt.
8 P. xB. + Kt. cov.
9 E. XKt.mate.
No. 457.
1 R. g8 + E. cov.
2 Ktxd6 + P. x Kt.
3 Q. f7 + K, x Q.
4 E. g7 + K. e8
5 B. d7 + Kt X B.
6 P. x Kt. mate.
Wo.
468.
WHITE.
BLACK.
1 B. g8 +
K
2 B. f7 +
K
3 B. S 6 +
E. x B.
4 P. XE. +
K. X P.
5 Q Xh6 +
P. xQ.
6 E. g8 +
K
7 Kt: mates.
No.
459.
1 E. xP. +
Kt x E.
2 Q. x Kt +
Kt X Q.
3 Kt. c6 +
K. b7
4 E. b8 +
K. x Kt.
5 E. X b6 mate
No.
460.
1 KtxE. +
.K. b7
2 K. 13 +
3 B. aG +
4 B. x B. +
SB. d5 +
6 B Xe6 +
7 B. xE. +
8 E. b8 +
9 E. e8 +
10 E. e6 +
11 E. xg6 +
12 E. g4 +
13 E. xB. +
14 E. g4 +
15 E, gl +
1G Kt. mates.
No.
461.
1 Kt. go +
K
2 E. h8 4-
B. xE.
3 K. g8
Kt. d6
4 K.xB.
5 Kt. mates.
No.
462.
1 Kt. c3
P. a2
2 E. al
K. a3
3 Kt. c2 4-
K. b3
4 E. X P.
B. f6
5 E. a3 +
K. b2
6 Kt. dl 4-
K. bl
7 B. x c4
P. XB.
8 Kt. c3 4-
K. b2
9 E. a2 +
K. b3
10 K. ct
K. X Kt.
or anything.
HE. a3 mate
No.
463.
1 B. c3
K. d3 (A)(B)
2 Kt, e5
K. x B.
3 Q. b2 mate.
WHITE.
BLACK.
(A) 1 B. X P-
2 Kt. e5 4- E. f4
orB
3 Q. f3 mate.
(B) 1 K. f4
2 Kt. e5 ' K. f5
3 Ct. g4 mate.
No. 464.
1 Q. d8 Q. f3+(A)(B)
2 B. g4 +
3 Q. or Kt. mates.
(A) 1E.XP.+
2 B. f5 +
3 Q. or Kt. mates.
(B) 1 E. d7
2 B. b7 +
3 Kt. mates.
(C) 1 B. X Kt.
2 B.X a6 +
3 Q. X E. mate.
No. 465.
I
2
3
4
Q. c5
B. e6 -f
Q. X E. +
Q. mates.
1
2
3
4
N<
Q. d6
Kt do
Q. X Kt.
Q.orKt. m
3
4
2
3
4
2
3
4
2
3
4
Kt. g6
KtX E.
466.
Kt. e6
B. x Kt.
anything.
No. 467.
Q. h.4
Kt. c6
E. X P. 4-
E. b8 mate.
^ x> (A ) l
Q.xB. +
Kt. a6 +
e8 mate.
(B)l
c6
d8 +
B. X B. mate.
(0)1
KtXaG
E.xB.
KtX c5 mate.
E. x Q. (A) (B)
P. a6
B. xE.
Q.
Kt.
Q.
B. X
K.
K.
E.
B. x
E. X
B.
b8
cS
dG
Kt.
Q.
P. a6
K. X Q.
E. X B.
No. 468.
Kt.
Q.
Q.
e7 +
d5 +
f7
Q. X h7 +
B. f8 mate.
B. x Kt.
K. hS
E. g8
k. x a
No.
469.
No. 478.
WHITE.
BLACK.
WHITE, BLACK.
1
2
3
4
5
Kt. h5 +
E. d5 +
P. X P. +
E. gl
E. dl mate.
P. X Kt.
K. e6
K. xE.
anything.
1
2
3
Q. f4 + K. x Q.
Kt. do + K. x P.
B. g6 mate.
No. 477.
No. 470.
1 Q.x P. + P. x Q.
2 1>. c7 Kt. e7
or Q. x h3
3 B. b7 + K. X B.
4 P.xKt(aQ)+K c6
5 Q. c7 mate.
No. 471.
1
Q X Kt + E. X Q.
2
E. f7 + K
3
Kt. c6 + K. X Kt.
4
P.XP. + K. X Kt.
5
P. d8(aKt)+B. X Kt.
6
P. b8(aKt) mate.
No. 472.
1 Kt. b5 + Kt X R. +
2 K. a6 Kt X b6
3 P. xKt. E. b8
4 P. b7 + R. X P.
5 P. queens. + E. eov.
6 Q. c6 + E. eov.
7
Q. yE. mate
No.
473.
I
K. e5 +
K. b4
2
P. a3 +
K. a5
3
P. b4 +
K. a6
4
Kt. e6
B. d8
5
Kt. d7
Kt X P.
6
P. XP.
R. c8
7
P. f5
E. a8
8
p. m
E. c8
9
P. f7
E. a8
10
P. f8(aQ.;
E. c8
11
Q. c5
KtX Q.
12
KtxKt +
Kt X Kt.
14
KtxKt mate
No.
474.
1
Q. d8 +
Kt x Q.
2
Kt. d6 +
K. f8
3
P. e7 +
K. g8
4
Kt. f7 +
Kt X Kt. mate
No.
475.
1
P. f4 +
K -
2
Q. d5 +
K. XQ.
3
Kt. g5 mate.
1 Q. & + K. x eo
2 Q. f6 + K. X Q.
3 Kt. e4 mate.
No. 478.
Q. g3 +
Kt. e3 +
Q. X P. +
B. b6 mate.
K
K
Kt x Q.
No. 479.
1 Kt. b6 + B. x Kt.
2 Q. b8 + E. X Q.
3 E. a7 + B. X E.
4 Kt. c7 mate.
No. 480.
d7 +
1 B.
2 Kt, c4 +
3 Kt. f6 4- K. c5
4 P. b4 mate.
K. X E.
E. x Q.
No. 4S1.
1 Q. c6 + K
2 B. a5 + K. x B.
3 P. c3 anything.
4 P. b4 mate.
No. 482.
1 E. d5 + Kt X E.
2 Q. c6 + K. x Kt.
3 Kt. f3 + K. x P.
4 Q. c2 + K. X Kt.
5 Q. g2 mate.
No. 483.
1
Kt.
a7 +
K. d7
2
P.
e6 +
K. d6
3
Kt.
b5 +
K. e5
4
K.
g5
Kt. d5
5
B.
c7 +
Kt. x B.
6
P.
f4 mate.
No.
484.
1
E.
al +
K
2
Kt.
d5 +
K
3
E.
cl +
K. X Kt
4
E.
e5 +
P. xE.
5
R.
c5 +
E. X E.
6
Q.
d7 mate.
No. 485
WHITE. BLACK.
1 Kt. C6 + K
2 B. e2 + K. X Kt.
3 B. f 3 + K
4 Q. f5 + K. x Q.
5 Kt. d4 + K. X P.
6 B.;X e6 mate.
No. 486.
1 Kt. bo + K
2 Kt. d6 + K. X B.
3 Q. a7 + K. X Kt.
4 b4 d4 + P. x E.
5 Q. e7 + K
6 B. e6 + K. X E.
7 B. x g8 + K
8 Q. e6 + K
9 Kt. f3 + K
10 P. mates.
No. 487.
1 B. el Q. x Q. (A)(B)
2 P. d4 c4 X P.
3 Kt. g3 +
4 Kt. or E. mates.
(A) 1 E. X Q.
2 P. d3 + E.,Kt.,orP.xP.
3 Kt. g3 +
4 Kt. or E. mates.
(B) 1 G. g7
2 Q. d4 + Q. X Q.
3 Kt. g3 +
4 E. mates.
No. 488.
1 R. b5 + R. X E.
2 Kt. c6 + K. fo
3 E. e3 P. e5
4 E. X P. + R. or Kt x E.
5
Kt.
d4 mate.
No. 489.
1
2
3
4
5
Kt.
B.
K.
K. •
E.
e3 P. X Kt.
f 1 P. X Kt.
c2 P. e7
1 Q. c2
2 Q. c3 +
3 Q. a5 +
4 Q. c7 mate.
(A) 1 P. f3
2 Q. h2 +
3 Q,. h5 +
4 Q. h8 mate.
■WHITE.
BLACK.
(B) 1 P. e3
2 Kt. d3 + K. e4
3 Kt. 1)4 dis. + K. f3
4 Q. g2 mate.
If 3 K. eo
4 Kt. c6 mate.
No. 699.
1 B.
K x E.
or B. d4
2 Q. e7 mate.
1 B. b2 or c3
2 Q. x B. mate.
1 Q. B
2 Kt. d7 mate.
1 Kt X E.
2 Kt. g6 mate.
1 Kt. elsewhere.
2 Q. d6 mate.
1 E. U
2 E. e6 mate.
1 E. elsewhere.
2 Kt. g4 mate.
1 g3 moves.
2 E, f5 mate.
No. 700.
1 Q. f7 1 B. x Q. (A)
2 P. c4
3 Mates.
(A) 1 K. X Kt.
2 P. a4 + K. X P.
or K. a6
3 Q. mates.
No. 701.
1 Q. b8 Kt x B.
2 Q.xb2
3 Mates.
No.. 702.
1 B. a4 K. X Kt.
2 B. b3 mate.
1 P. d6
2 bo c7 mate.
1 P. f6
2 d5 c7 mate.
IP. f5
2 Q. g8 mate.
1 P. e4
2 Q. X e4 mate.
No. 703.
1 B. dl K—
2 B. c2
3 Q. or B. mates.
No. 704.
1 Q. d6 B.
2 E. g3 + K.
e5 +
14
WHITE BLACK.
3
P. b.6 Q. p
4
E. a4 anything.
5
Mates.
No. 705.
1
E. g2 B. X P.
2
Q. gl anything.
3
Mates.
No. 706.
To mate.
1
E. X b3 + P. X E.
2
Q,. X B. mate.
To sui-mate.
1
E. Xa2 + P. xE.
2
Q. c3 + B. X Q. mate
To stalemate.
1
Q. X B. Kt
2
Kt X Kt. stalemate.
To sui-stalemate.
1
Kt. c2 + P. X Kt.
2
Q. c3 + B. X Q.
sui-stalemate.
No. 707.
1
Q. g2 K. f6 (A)
2
Q. hi K. e5 *
3
Q. al mate.
* If 2 P. e5
3
Q. c6 mate.
If 2 Kt. f4
3
Q. h8 mate.
If 2 Kt. e4
3
Kt xP. mate.
(A) 1 h3 (B)
2
KtxP. +
3
Q. X Kt. mate.
(B) 1 f2
2
KtxP. +
3
Q. e4 mate.
(C) 1 K. d4
2
Ktxb4 Kt. f4 *
3
Q. X Kt. mate.
* If 2 Kt. dl
3
Q,. e4 mate.
•
If 2 P. e5
3
Q. d5 mate.
If 2 Kt X Kt.
3
Q. e4 mate.
No. 708.
1
Q. a8 K. g4
2
Q. a4 anything.
3
Q. d7
01
Q. dl mate accordingly.
No. 709.
WHITE. BLACK.
1 Q. h5 K. do
2 Q. dl K. xE.
3 Q. b3 mate.
If 2 E. x Q.
3 Kt X e3 mate.
No. 710.
1 R. e7 E. x E.
2 Kt. f7 E. x Kt.
3 P. e4 -f P. x P. en pass.
4 E. d4 mate.
No. 711.
1 Q. h2 K. x Kt.
2 Kt. d3 dis. mate.
1 K. x E.
2 Kt. e6 mate.
1 Kt X Kt.
2 B. X B. mate.
No. 712.
1 Kt. d4 B. x Kt. (A)
2 E. e5 K. x E.
3 Q. d6 mate.
(A) Kt X Kt,
2 Q. d6 + K. f3
3 Q. xE. mate.
No. 713.
1 Q.xB. P. x Q.
2 B. b5 + K. X B.
3 Kt. b4 Q. x Kt.
or B. x E.
4 P. a4 + K. x Kt.
5 B. d2 mate.
No. 714.
1 E.X eo B. e7 (A)
2 R. x f3 P. X E.
3 KtxB. mate.
(A) 1 B. d5
,2 Q. b6 + K
3 Kt or P. mates.
(B) 1 B. X E.
2 Q. b6 + K
3 P. b3 mate.
No. 715.
1 B. bl
2 Q.xB.
3 Kt. g3 +
4 Mates.
(A) 2 E. X Q
Q. X B.
Kt. d7 (A)(B)
anything.
"WHITE, BLACK.
3 R. g5 + B. X E.
4 Kt. mates.
(B) 2 Q. e4
3 Q.XP/+ R. g4
4 Q. x E. mate.
No. 716.
1 Q. g5 K. x P.
2 Q. ft mate.
1 P. X P.
2 B. g7 mate.
1 K. c5
2 Q. bb mate.
1 K. e3
1 Q. d2 mate.
1 Kt. moves.
2 Q. X P. mate.
No. 717.
1 Q. d6 P. d4 (A)
2 B. f4 anything.
3 Mates.
(A) 1 Kt. b7
2 Q. b4 + P. d4
3 Q. xKt. mate.
(B) 1 Kt. c6
2 Q. xKt. + &c.
(C) 1 Kt. o4
2 Q. a6 &c.
No. 718.
1 B. h8 1 K.,E.,B., or h3
moves.
2 Q. al
3 Q. mates.
1 P. X B.
2 Q. e8
3 Q. mates.
1 P. b3
2 Q. d6
3 Q,. mates.
No. 719.
1 B. c3 B. X P.
2 Q. a6 + B. X Q.
or K
3 P. or Kt. mates.
No. 720.
1 B. c3 B. x, B.
2 B. g2 anything
except (A)
3 Kt. d3 + P. X Kt.
4 P. f4 mate.
(A) 2 Kt X P.
3 Kt. d6
4 Kt. mates.
No. 721. Ghuznee.
No. 148. Corrected.
WHITE. BLACK.
1 Q. X Kt. P. x Q.
2 R. b7 Kt. f4 +
3 K. f 1 (A) Kt x Kt.
4 R. e7 + K. d8
5 KtX P. + K. c8
6 P. c4 * Q. d8 (best)
7 P. queens. Q. x Q.
8 P. + K. c7
9 P. queens. + K. x Kt.
10 Q. d7 + K. b6
11 Q. b5 mate.
* This is the move which appears
to have remained nndiscovered for
so many years.
(A) If white play
3 Kt x Kt. which seems to be the
natural move, Black will escape,
thus —
3 Q. X P.
4 e5 g6 (best) Q. X Kt.
5 KtxQ. Castles!
and White is foiled.
It appears that in a letter from Bolton
to Lewis, dated 9th September, 1843, after
noting certain points of attack and defence
in this problem, he continues thus — " and,
if necessary, Black can afterwards Castle."
No. 723.
1 K.x e4 K. x E.
2 B. e5 K. X E.
3 B. c2 mate.
If 2 B. X E.
3 B. b5 mate.
No. 723.
1
K. a2 P. *b4
2
Q. ' f8 + B. b8 dis. +
3
Q. f3 mate.
No. 724.
1
E. b5 P. x B. (A)
2
Ktx P. K x P.
3
Kt. e4 + K. x Kt.
4
E. e5 mate.
(A) 1 K. X P.
2
B. e4 + K. c4
3
E. b6 B. X Kt.
4
P. X B. mate.
(B) 1 B. X Kt.
2
E. b6 + K. x P.
3
P. e4 + B. X P.
4
B. X P. mate.
No. 725.
1
B. c6 K. d3 (A)
2
B. xP. + K. e4
3 Kt. e7 dis. mate.
(A) 1 B. X P.
2 KtX e3 dis. + K. X Kt.
3 Q. a3 mate.
(B) 1 K. f5
2 Q. e8
3 B. d7 mate.
(0) 1 P. queens.
2 Kt X f4 dis. + K. X Kt.
3 Q. f8 mate.
No. 726.
1 E. g8 Kt X Kt. (A)
2 Q. g6 K. c5 v;.
3 Q. X P. + K. X Q.
4 E. c8 mate.
(A) 1 B. c3
2 P. e3 + ' K. e4
3 Kt. f 6 + E. X Kt.
4 Q. X P. mate.
No. 727.
1 E. b3 Kt X Q. (A)
2 B. e4 + K. X B. ;
3 Kt. mates.
(A) 1 Kt. d7
2 Q. X e6 + K. X Q.
3 B. mates.
No. 728.
1 E. g4 P. X E.
2 E. g5 P.orB. X R.
3 Kt. f5 mate.
If 2 K. X E.
3 B. cl mate.
No. 729.
1 Q. f6 + K. x R.
2 Q. b2 Kt X Q.
3 Kt. b3 mate.
No. 730.
1 Q. f2 P. e5 (A)
2 Q. fl P. c5
3 E. xP. + K.,B.,orP.xE.
4 Q,. h3, f8, or a6 mate.
(A) 1 P. c5
2 Q. X B. P. X E.
3 B. f4 +
4 Q. mates.
(B) 1 P. x E.
2 Q. e3 +
3 Q. d4
4 Q,. g7 mate.
No. 731.
1 Q. d2 h7 h6 (A)(B)
2 Kt. f2 K. g3 *
3 KtX e4 + K. f4
5 Kt.
4 Kt. g3 K. g3
or K. x P.
5 Kt. fl or Kt. g4 mate.
If 4 P. X e5
5 Q. f2 mate.
If 4 B. g3
5 Q. d4 mate.
* 2 P. X eo
3 f2 dl P. d6
4 Kt. c3 K. g3
5 Kt. fl mate.
If 4 anything else.
e2 mate
* 2 K. X P.
f2 X g4 + K. f4
Q. b4 anything.
Q. d6 mate.
* 2 P. g3
Kt. g4 dis. + K. i'5
KtXh6 + K. X B.
i'2 g4 mate.
(A) 1 P. g3
c4 dis. + K . g4
e2+ K
e3 anything.
g4 mates.
(B) 1 B. el
Q. X B. ' K. X P.
Q. fl K. d4
g4 anything.
c4 mate.
(C) 1 K. X P.
c3 + K. f4
f6 + K. g3
d6 mate.
(D) 1 B. f2
Q.XB. + K. X P.
K. c3 anything.
Q. f5 mate.
No. 732.
1 Ktx eo P. X B.
2 d4 e6 B. X B.
3 Kt. e7 + K
4 Q. b4 or b6 mate.
An extremely difficult Problem ;
the variations on which I leave as
an exercise for the reader.
Kt.
Q.
dl
Q.
Kt.
Q.
Q.
Q.
Q.
Q.
Q.
Q.
Kt.
No. 733.
fl K
b5 + K
b8 + K. X Q.
a6 mate.
No. 734.
1 Kt.
2 Q.
3 Kt.
3 Kt.
g5
c5 +
e4 mate.
If 2 K
d3 mate.
E. f6 (A)
K. X Q.
■WHITE.
BLACK.
(A)l
E. f8
2 Q. X E. +
K. x E.
3 Kt. d3 mate.
(B)l
K. XE.
2 Kt. d3 +
K. d6
3 Q. c5 mate.
(C)l
B. X E.
2 Kt. f7 +
K. e6
3 Q. f5 mate.
(D)l
P. d4
2 Kt. e4 +
K. XE.
3 Q. io mate.
(E)l
Kt. d4
2 Kt. f7 +
K. c5
3 Q. X Kt. matt
t
No.
735.
1 K. d2
K. hi
(A)
2 Q. a8
P. gl
dis. +
3 Kt. f2 mate.
If 2
K. gl
3 Q. a] mate.
(A)l
E. hi
2 Q. b8
anything.
3 Q. mates.
No.
736.
1 Kt. c5 + +
K. b5
2 Q XP. +
K. X Kt.
3 Q. b4 +
K. d4
4 B. XB. +
K. X P.
5 E. d2 +
K. e4
6 Q. XE. +
E. cov.
7 Q. X c6 +
; J
8 Q. Xe6 +
>i
9 Q. X c6 +
? J
10 Q. xKt +
} '
11 Q. c6 -4-
')
12 P. XB. +
K. XP.
13 B. XQ.
K g2
14 Kt. g5
K. XE.
(A)
15 B. c5
P. H3
16 B. Xh3
P. h4
17 Q. a8
P. c6
18 E. dl +
E. XE.
mate.
(A) 14 P. h3
15 B. X P. +
K. XE.
16 B. c5
P. M
17 Q. a8
P. c6
18 E. dl +
R. X E.
mate.
xE.
Solutions of the Problems on the Frontispiece
and the Vignettes.
Camtlla.
WHITE.
BLACK.
12 Kt
d6
K
a8
WHITE.
BLACK.
13 Kt
. c8
K.
b8
1 Q. f4 + *
K. e8
14 Kt
a7
K.
a 8
2 Q. a4 +
K. f8
15 P.
b4
K.
b8
3 Q. a8 +
E. d8
(A)
16 P.
b5
K.
a8
4 Q. xKt +
B. cov.
17 P.
b6
K.
b8
5 Q. X B. +
K. e8
18 Kt
c-6 4-
K.
moves.
6 Q. g4
K. f8
(best)
19 P.
mates.
7 Q. f 5 +
K. e8
8 Q. Ii5
Q. X R.
+
The Propeller.
9 Q. X Q. +
K. f8
10 Q. f7 mate.
i a
f3 4-
K.
a7 (best)
(A),
5 B. d8
2 Kt
c6 4-
4 Q. xKt +
K. e8
3 Kt
d8 dis.
+
5 Q. h5
E. x e6
(best)
4 P.
xP. 4-
K.
XP.
6 E. g8 +
K. d7
5 Q
c6 4-
7 Q. e8 +
K. c8
6 Q.
c5 +
8 Q. x B. +
K. b7
7 Q.
d5 4-
9 Q. a 8 mate
8 a
d4 4-
• * 1 Q. x Kt
. 4- will not win
9 Q.
e4 +
though it looks so
tempting.
10 Q.
11 Q.
e3 4-
f3 4-
f2 4-